You are on page 1of 264

Sno Exams Name Sub

1. Allahabad Bank Clerk Exam., 2010 Held on 14-3-2010 English Language : Solved
IstShift Paper

2. Allahabad Bank Clerk Exam., 2010 Held on 14-3-2010 English Language : Solved
IInd Shift Paper

3. Bank of India Probationary Officers Exam 2010 General Awareness on Current


Held on 24-1-2010 Developments : Solved Paper

4. Indian Bank Probationary Officer Exam., 2010 English Language : Solved Paper
(Held on 17-1-2010)
5. Baroda Rajasthan Gramin Bank Officers Exam., 2010 English Language : Solved Paper
Held on 21-02-2010
6. Union Bank of India Clerk Exam., 2010 General English : Solved
(Held on 10-1-2010) Paper

7. Punjab National Bank Clerk Exam., 2010 English Language : Solved


(Held on 4-4-2010) Paper

8. Allahabad Bank Probationary Officers Exam., 2010 Computer Knowledge : Solved


(Held on 21-2-2010) Paper

9. IDBI Bank Executive Exam., 2009 Reasoning : Solved Paper


(Held on 13-12-2009)

10. Oriental Bank of Commerce Probationary Officers Exam., 2009 Reasoning : Solved Paper
(Held on 13-12-2009)
11. Corporation Bank Probationary Officers Exam., General Awareness on
2009 Socio-Economic
(Held on 22-11-2009) Developments : Solved
Paper
12. Malwa Gramin Bank Clerk Exam., 2010 English Language : Solved
(Held on 31-1-2010) Paper

13. Rajasthan Gramin Bank Clerk Exam., 2009 English Language : Solved Paper
(Held on 29-11-2009)
14. Gurgaon Gramin Bank Clerk Exam., 2009 English Language : Solved
(Held on 1-11-2009) Paper
15. Punjab National Bank Agriculture Officers Exam., 2009 Professional Knowledge
(Held on 16-8-2009) (Economics) : Solved Paper

16. United India Insurance Co. Administrative Officers Exam., General Knowledge and Current
2009(Held on 24-5-2009) Affairs : Solved PapeR
17. Andhra Bank Probationary Officers Exam., 2009 General Awareness :
(Held on 5-7-2009) Solved Paper
18. United Bank of India (P.O.) Exam., 2009 English Language : Solved
(Held on 21-6-2009) Paper
19. Nabard Bank Officers Exam., 2009 Reasoning : Solved Paper
(Held on 29-3-2009)
20. Bank of Baroda Agriculture Officers Exam., 2008 Reasoning : Solved Paper
(Held on 14-12-2008)
21. United Bank of India Probationary Officers General Awareness :
Exam., 2009 Solved Paper
(Held on 21-6-2009)
22. Indian Overseas Bank Probationary Officers General Awareness :
Exam., 2009 Solved Paper
(Held on 5-4-2009)
23 Gurgaon Gramin Bank Officers Exam., 2009 General Awareness : Solved Paper
(Held on 12-7-2009)
24. Indian Bank (Clerk) Exam., 2009 English Language : Solved
(Held on 21-6-2009) Paper
25. Andhra Bank (Clerk) Exam., 2009 English Language : Solved
(Held on 14-6-2009) Paper
26. Canara Bank Probationary Officers Exam., English : Solved Paper
2009(Held on 15-3-2009)

Allahabad Bank Clerk Exam., 2010


English Language : Solved Paper
[Held on 14-3-2010(First Shift)]

Directions—(Q. 1 to 15) Read the following passage carefully and answer the questions given
below it. Certain words have been printed in bold to help you locate them while answering some
of the questions.

In a village in the East, there lived an old couple. They did not have any children and never spent
an extra paisa on themselves. They never repaired their house or cooked good food. One day, a
family moved into the village, it was their son’s second birthday and they sent two delicious
laddoos to the old couple. The couple ate the laddoos with great relish. For days after they could
talk of nothing else. Finally the old man could take it no more. He said to his wife, “Let’s buy the
ingredients to make just two laddoos.” His wilfe agreed but said, “If I make the laddoos at home,
the neighbours will know and will want a share. Let’s cook in the woods behind our hut. That
way no one will ever know.”

The next day the old man purchased the ingredients from the market and they set off to the
woods. Having never prepared food like this before, they ended up with three laddoos instead of
two. The old couple returned home with the sweets, dying to eat them. But there was a problem.
“It was my idea and I went to the market, so I ought to get two,” said the old man. But his wife
refused to give in so easily and said, “Since I prepared the sweets. I ought to have two.” The day
wore on and the argument continued.

At last, the old man suggested, “Let us not talk to each other. Whoever breaks the silence first
will get one laddoo and the other person two.” His wife agreed. Hours passed, but neither of
them spoke and finally they fell asleep with the bowl of sweets between them. That evening one
of the neighbours knocked on the door but the couple did not answer for fear of losing the bet.
Peering through the window, the man saw the couple lying on the floor with an untouched bowl
of food. “They are dead !” he shouted and alerted the other villagers. Soon the door was broken
down and the house was swarming with villagers. They discussed everything from the funeral to
where the couple had hidden their wealth. But the couple lay still.
Finally, the villagers carried them to the cremation ground. As the flames licked his feet, the old
man jumped up screaming “You win !” His wife jumped up too, happy that she had won. The
two then rushed home leaving the bewildered villagers present there behind. But when they got
home…Alas the door had been left open and they found a stray dog licking the last crumbs of
the sweets from the bowl.

1. Why did the old couple cook in the woods ?


(A) It was very convenient to gather firewood
(B) To avoid the risk of their hut burning down
(C) To keep the fact that they were preparing laddoos a secret from their neighbours
(D) To hide the fact that they did not know how to prepare laddoos from their neighbours
(E) They did not have the ingredients to make enough laddoos for their neighbours as well
Ans : (C)

2. What caused the old couple’s sudden longing for laddoos ?


(A) They hadn’t eaten good food for several days
(B) They had attended their neighbour’s party and enjoyed the laddoos served
(C) They saw the mouth watering laddoos being served to guests at the party
(D) They had tasted the laddoos prepared for their neighbour’s son’s birthday
(E) They thought that the laddoos would satisfy their hunger for days to come
Ans : (D)

3. Why did the old couple initially not reveal that they were alive to the villagers ?
(A) To learn the villagers’ true opinion of them
(B) They did not like the villagers and were mocking them
(C) Neither of them wanted to be the first to speak and lose the bet
(D) To test the honesty of the villagers
(E) None of the above
Ans : (C)

4. What did the old couple disagree about ?


(A) Where they would cook their meal
(B) Which of them was entitled to the first laddoo
(C) Whether to share the extra laddoo with their neighbours
(D) The quantity of laddoos that they had made
(E) None of these
Ans : (B)

5. What was the villager’s reaction to the old couple’s presumed ‘death’ ?
(A) They were upset and all came to pay their last respects
(B) They were worried about how to pay for the funeral
(C) They could not decide what to do with the couple’s wealth
(D) They disliked the old couple and demolished their house
(E) They dutifully arranged the couple’s last rites
Ans : (E)

6. Why did the neighbour peep into the old couple’s house ?
(A) To learn what they were being so secretive about
(B) To spy on them and discover where their wealth was hidden
(C) To complain as they had cooked in the forest
(D) To check up on the old couple as he did everyday
(E) Not clearly mentioned in the passage
Ans : (E)

7. Which of the following can be said about the old couple ?


(A) They did not have any relatives to look after them in their old age
(B) They were misers and did not spend even on keeping their house in good condition
(C) They were not allowed to eat sweets as they were old
(D) They were fond of animals and sometimes shared their food with stray animals
(E) They were afraid of the villagers and hid from them
Ans : (B)

8. Which of the following is/are TRUE in the context of the passage ?


(1) The old woman had prepared more laddoos than she had intended
(2) The family who had recently moved into the village were fond of the old couple
(3) The villagers were saddened by the death of the old couple and gave them a grand funeral
(A) Only (1)
(B) Only (2) & (3)
(C) Only (1) & (2)
(D) All (1), (2) & (3)
(E) None of the above
Ans : (A)

9. What puzzled the villagers?


(A) The old couple did not die in the fire
(B) The sight of the old couple whom they had presumed dead, alive
(C) The fact that the old couple had wasted good food
(D) The lack of concern the old woman had for her husband
(E) The old couple fed the laddoos to the dog.
Ans : (B)

10. Why did the old man scream when surrounded by the villagers ?
(A) He was afraid that his wife would get burned
(B) To scare away the villagers who wanted to kill him for his money
(C) His wife refused to allow him to eat more than one laddoo
(D) To indicate to the villagers that he was alive
(E) The fear of being hurt in the fire
Ans : (E)

Directions—(Q. 11 to 13) Which of the following is most similar in meaning to the word given
in bold as used in the passage.

11. breaks
(A) interrupts
(B) separates
(C) solves
(D) defeats
(E) escapes
Ans : (A)

12. passed
(A) overtook
(B) occupied
(C) unnoticed
(D) proceeded
(E) succeeded
Ans : (E)

13. share
(A) half
(B) part
(C) participate
(D) divide
(E) common
Ans : (B)

Directions–(Q. 14 and 15) Which of the following is most opposite in meaning to the word
given in bold as used in the passage.

14. last
(A) new
(B) begin
(C) constant
(D) discontinue
(E) first
Ans : (E)

15. still
(A) restless
(B) noisily
(C) conscious
(D) calmly
(E) lifeless
Ans : (A)

Directions–(16 to 25) Read each sentence to find out whether there is any grammatical mistake/
error in it. The error if any, will be in one part of the sentence. Mark the letter of the part with
error as your answer. If there is ‘No error’, mark (E)

16. If his aim is to (A) / buy a house in the (B) / next three years he (C) / should start immediate
saving. (D) No error (E)
Ans : (D)

17. Can you explain why (A) / you were late this morning (B) / when we had to make (C) / such
an important presentation ? (D) No error (E)
Ans : (B)

18. Their company is going to incur (A) / heavy losses this year because (B) / of the big
discounts they (C) / offers to customers. (D) No error (E)
Ans : (D)

19. The main advantage of (A) / investing such schemes (B) / is that you will not (C) / have to
pay any taxes. (D) No error (E)
Ans : (B)

20. The final decision can (A) / be taken only after (B) / we consider all (C) / the panel
suggestions. (D) No error (E)
Ans : (C)

21. The Foreign Investment Promotion Board is (A) / the government body who (B) / regulates
investment received (C) / by Indian factories from foreign countries. (D) No error (E)
Ans : (B)

22. Our company was recently (A) / award the contract (B) / to construct two major (C) / bridges
in the state. (D) No error (E)
Ans : (B)

23. That customer has (A) / written a letter (B) / thanks the manager (C) / for her timely help. (D)
No error (E)
Ans : (C)

24. Accordingly the circular (A) / that was issued yesterday RBI (B) / has raised the rate at (C) /
which it lends to banks. (D) No error (E)
Ans : (B)

25. A large number of policy (A) / have lapsed because many (B) / people have not paid (C) /
their instalments on time. (D) No error (E)
Ans : (A)

Directions—(Q. 26 to 30) In each question below a sentence with four words printed in bold
type is given. These are lettered as (A), (B), (C) and (D). One of these four words printed in bold
may be either wrongly spelt or inappropriate in the context of the sentence. Find out the word
which is wrongly spelt or inappropriate if any. The letter of that word is your answer. If all the
words printed in bold are correctly spelt and also appropriate in the context of the sentence, mark
(E) i.e., ‘All correct’ as your answer.

26. The Chairman has promissed (A) to look (B) into the matter (C) and take necessary (D)
action. All correct (E)
Ans : (A)

27. Most (A) projects are delayed (B) because of the absents (C) of proper planning. (D) All
correct (E)
Ans : (C)

28. He found (A) the company in 1980 to provide (B) electricity to people living (C) in rural
areas. (D) All correct (E)
Ans : (A)

29. Salaries (A) of bank employees (B) are likely (C) to go above (D) by ten percent. All
correct (E)
Ans : (D)

30. Since (A) the interest rate on bank deposits is low (B) at present, people prefer (C) to deposit
their savings (D) in post office schemes. All correct (E)
Ans : (B)

Directions—(Q. 31 to 40) In the following passage there are blanks, each of which has been
numbered. These numbers are printed below the passage and against each, five words are
suggested, one of which fits the blank appropriately. Find out the appropriate word in each case.

A rich merchant owned a beautiful horse. Now, one of his neighbours was keen to (31) this
horse. He offered the merchant gold, (32) the merchant refused. This angered the neighbour. He
(33) to trick the merchant. He disguised himself (34) a beggar and lay down at the (35) of the
road, that the merchant used everyday. (36) at the sad plight of the beggar, he (37) from his horse
and gently lifted the beggar (38) his horse. No sooner did the beggar get into the (39) than the
galloped away. But hearing the merchant (40) out to him, he stopped when he was a safe
distance away. “Please don’t tell anyone how you came to possess this horse. Some day an ill
man may be lying by the roadside and people who have heard about your trick, may not want to
help,’’ said the merchant. The merchant’s neighbour was deeply ashamed, rode back and
returned the horse to the merchant.

31. (A) buy


(B) steal
(C) free
(D) train
(E) sell
Ans : (A)
32. (A) so
(B) unless
(C) otherwise
(D) but
(E) since
Ans : (D)

33. (A) desires


(B) decided
(C) declined
(D) deliberately
(E) deceived
Ans : (B)

34. (A) being


(B) alike
(C) as
(D) identical
(E) similar
Ans : (C)

35. (A) middle


(B) gutter
(C) ending
(D) distance
(E) side
Ans : (E)

36. (A) Carefully


(B) Distressed
(C) Misery
(D) Upsetting
(E) Unhappily
Ans : (B)

37. (A) stepped


(B) jump
(C) fell
(D) dismounted
(E) climbed
Ans : (D)

38. (A) with


(B) above
(C) from
(D) off
(E) onto
Ans : (E)

39. (A) saddle


(B) arms
(C) horse
(D) danger
(E) way
Ans : (A)

40. (A) appeal


(B) shouted
(C) call
(D) cried
(E) beg
Ans : (C)

Allahabad Bank Clerk Exam., 2010 [Held on 14-3-2010 (Second Shift)]


English Language : Solved Paper

Directions—(Q. 1–15) Read the following passage carefully and answer the questions given below it.
Certain words have been printed in bold to help you locate them while answering some of the questions.

Keshav and Yash lived in neighbouring villages. Once, a fair was held nearby and they set off from their
homes hoping to do some business there. Keshav filled his sack with cheap cotton, overlaid it with a layer
of fine wool, and set off for the fair. Meanwhile Yash, collected some old clothes, put a layer of sheer
cotton on top and made his way to the fair. Each stopped to rest under a tree and got to talking. “I have
the finest wool in my sack,” boasted Keshav Not to be outdone, Yash said, “I have cotton of the most
superior quality.”

The two struck a deal. They would exchange their goods and since wool was more expensive, Yash
would pay Keshav an extra rupee. But Yash had no money on him. So after agreeing to pay Keshav later,
they went home, laughing at each other’s folly. It did not take them long to discover that they had been
duped. The next day, Keshav landed at Yash’s house. “You cheat! Give me my rupee at least.” Yash was
untroubled by Keshav’s words, “Of course. But first help me find the treasure at the bottom of this well.
We can divide it.” So Keshav went into the well. But each time Yash, hauled up the bucket Keshav had
filled, he said, “Oh ! No treasure here. Try again.” Keshav soon realized he was being used as free labour
to clean the well. So he gave a loud shout, “Here is the treasure! Watch out it’s heavy !” Yash was
amazed that there actually was treasure in the well. As soon as he pulled up the bucket he threw away the
rope so that Keshav could not come up. But, what did he find in the bucket ? Keshav covered in mud !
They began fighting again. Soon it got too dark to continue and they left for their homes.

But Keshav did not give up easily. He arrived at Yash’s house a few days later. Yash spotted him and told
his wife, “I will pretend to be dead. Keshav will have to give up.” But when Keshav heard Yash’s wife
wailing, he rushed out to gather the villagers. “My friend has died. Let’s take his body for cremation.”
Yash’s wife got scared, “Go away. I shall arrange for cremation myself !” But the villagers thought she
was too grief stricken. Once they reached the cremation ground, Keshav told the villagers, “It is getting
dark. I shall watch over him during the night.” When the villagers had left, Keshav said to Yash, “Stop
pretending. Give me my money !” Now, a gang of thieves came upon them and seeing one figure seated
on the funeral pyre and another standing next to him talking, they assumed them to be ghosts and dropped
their bag of stolen goods and left at top speed. The two, saw the bag full of gold and silver ornaments and
divided it between themselves. Keshav made sure he got an extra gold coin and the account was settled at
last!

1. What made the two young men decide to visit the fair ?
(A) They thought they could get jobs and support themselves
(B) They wanted to trade their goods at the fair and earn some money
(C) To sell the produce they had grown for a handsome profit
(D) Being unemployed they thought they could entertain people at the fair
(E) Not clearly mentioned in the passage
Ans : (B)

2. What were Keshav and Yash quarrelling about ?


(A) The cost of wool
(B) The interest on the loan Yash had taken
(C) Dividing the thieves’ treasure equally
(D) The money Yash owed Keshav
(E) The wages for digging the well
Ans : (D)

3. Why were the two men happy despite not going to the fair ?
(A) They had managed to buy whatever they needed on the way
(B) They had got a good deal in exchange for a rupee
(C) They had become fast friends and had made a lot of money in the bargain
(D) Each thought he had got the better deal and had swindled the other
(E) None of the above
Ans : (D)

4. Why did the villagers gather at Yash’s house ?


(A) Yash’s wife was very upset seeing Keshav and her husband arguing
(B) To help Yash’s wife carry Yash’s body home as night had fallen
(C) To give Yash’s wife the news of her husband’s death
(D) They wanted to keep watch over Yash’s wife during the night
(E) None of the above
Ans : (E)

5. Why did Yash request Keshav to go down into the well ?


(A) He knew Keshav was smart and would clean it thoroughly
(B) He was in search of treasure that he had buried
(C) To trick Keshav into cleaning it
(D) His wife had refused to help him
(E) To make Keshav earn the extra gold coin
Ans : (C)

6. Which of the following cannot be said about the two young men ?
(a) They were lazy and made their wives work.
(b) They were a match for each other in shrewdness.
(c) They were dishonest and tried to swindle each other.
(A) Only (a)
(B) Only (a) and (c)
(C) Only (b) and (c)
(D) All (a), (b) and (c)
(E) None of the above
Ans : (A)

7. Why did Keshav get an extra gold coin from the thieves’ treasure ?
(A) As payment for digging the well
(B) It was the amount Yash owed Keshav for protecting him from the thieves
(C) As repayment of the rupee that Yash owed him
(D) It was his idea to spend the night at the cremation ground
(E) He had proved to be cleverer than Yash
Ans : (C)

8. What was Keshav’s reaction to Yash’s presumed ‘death’ ?


(A) He immediately sent for the villagers to comfort Yash’s wife
(B) He was very upset because he realized that he would never recover his money
(C) He paid for all Yash’s funeral arrangements
(D) He could not be consoled and kept a watch over Yash’s body all night
(E) He knew Yash was pretending and decided to trick him too
Ans : (E)

9. What frightened the thieves ?


(A) The sight of what they thought were the spirits of the dead
(B) The realization that the villagers would soon return
(C) Someone had discovered their hiding place
(D) Their loot had been stolen by other thieves
(E) The villagers had lit a funeral pyre for them
Ans : (A)

10. Which of the following is not true in the context of the passage ?
(A) Keshav was persevering by nature
(B) There was no treasure buried in Yash’s well
(C) Keshav guarded Yash’s body because he knew thieves lived there
(D) Yash’s wife supported her husband in his crooked dealings
(E) Neither Keshav nor Yash could be trusted to keep his word
Ans : (C)

Directions—(Q. 11–13) Which of the following is most similar in meaning to the word given in bold as
used in the passage ?

11. Discover
(A) Invent
(B) Explore
(C) Locate
(D) Realize
(E) Research
Ans : (D)

12. Struck
(A) Reached
(B) Impacted
(C) Slapped
(D) Attacked
(E) Occurred
Ans : (A)

13. Top
(A) Height
(B) Lid
(C) Finest
(D) Upper
(E) Maximum
Ans : (B)

Directions—(Q. 14–15) Which of the following is most opposite in meaning to the word given in bold
as used in the passage ?

14. Dark
(A) Bright
(B) Cheerful
(C) Shadow
(D) Fair
(E) Shining
Ans : (A)

15. Free
(A) Occupied
(B) Paid
(C) Captured
(D) Busy
(E) Dependent
Ans : (B)

Directions—(Q. 16–25) Read each sentence to find out whether there is any grammatical mistake/error in
it. The error if any, will be in one part of the sentence. Mark the letter of the part with error as your
answer. If there is no error, mark (E).

16. In order to (A) / attract tourists many (B) / hotels have been offered (C) / attractive deals and
discounts. (D) No error (E)
Ans : (C)

17. The Board is likely (A) / to take its time (B) / to examine the facts (C) / before giving their decision.
(D) No error (E)
Ans : (D)

18. How can we open (A) / these branches on time (B) / if we have not (C) / yet obtained the licences ?
(D) No error (E)
Ans : (C)

19. One of the disadvantage (A) / of fixed deposit schemes (B) / is that banks offer (C) / low rates of
interest. (D) No error (E)
Ans : (A)

20. According to experts, it is (A) / necessarily for you to save (B) / and invest at least twenty-five (C) /
percent of your monthly income. (D) No error (E)
Ans : (B)

21. The manager has not (A) / yet replied to the (B) / customer letter asking (C) / about her loan
application. (D) No error (E)
Ans : (C)

22. While the formation of RBI (A) / was approved in March, 1934 (B) / it was inaugurated (C) / only in
April 1, 1935. (D) No error (E)
Ans : (D)

23. The Insurance Regulatory Development Authority has (A) / asked all insurance (B) / companies
submitting their (C) / balance sheets by June. (D) No error (E)
Ans : (C)

24. Arun’s father has been ill (A) / since last year and Arun (B) / has spend his entire (C) / savings on his
treatment. (D) No error (E)
Ans : (C)

25. There are a large number (A) / of Chinese workers which (B) / are employed by (C) / software
companies in India. (D) No error (E)
Ans : (B)

Directions—(Q. 26–30) In each question below a sentence with four words printed in bold type is given.
These are lettered as (A), (B), (C) and (D). One of these four words printed in bold may be either wrongly
spelt or inappropriate in the context of the sentence. Find out the word which is wrongly spelt or
inappropriate if any. The letter of that word is your answer. If all the words printed in bold are correctly
spelt and also appropriate in the context of the sentence, mark (E) i.e., ‘All correct’ as your answer.

26. There are few (A) differents (B) between (C) these two marketing proposals. (D) All correct (E)
Ans : (B)

27. Most (A) banks have been hiring (B) staff for their corporate (C) banking divisions. (D) All correct
(E)
Ans : (B)

28. The interest rate for postal deposits is fixed (A) by the government and usually (B) remains (C)
stable. (D) All correct (E)
Ans : (D)

29. Companies (A) will benefit (B) if the Finance Ministry decides to lesser (C) taxes in this budget.
(D) All correct (E)
Ans : (C)

30. Compare (A) to private (B) banks, we have a larger (C) network (D) of branches in North India.
All correct (E)
Ans : (A)
Directions—(Q. 31–40) In the following passage there are blanks, each of which has been numbered.
These numbers are printed below the passage and against each, five words are suggested, one of which
fits the blank appropriately. Find out the appropriate word in each case.

A poor man was once travelling through the desert. Feeling thirsty and tired, he began to …(31)… for
water. At last finding a stream, he …(32)… to his knees and drank the water which …(33)… very sweet.
He filled up his bottle …(34)… continued on. After several days journey, he …(35)… the palace and got
an opportunity to …(36)… the King. As a mark of …(37)… he presented the King with the bottle …(38)
… water. The King tasted the water, and …(39)… rewarded the man. All the courtiers …(40)… to taste
the water but the King refused. When the man had left overjoyed, the king explained why he had acted so
strangely. “After such a long journey, the water was foul tasting. I could not risk anyone showing, his
disgust for a gift which was so sincerely given! For this reason alone I reserved the tasting of the water
for myself.”

31. (A) desire


(B) pursue
(C) need
(D) obtain
(E) search
Ans : (E)

32. (A) fell


(B) drop
(C) jumped
(D) leaped
(E) sat
Ans : (D)

33. (A) looked


(B) tasted
(C) flavour
(D) became
(E) sample
Ans : (B)

34. (A) before


(B) soon
(C) and
(D) that
(E) however
Ans : (C)

35. (A) came


(B) reached
(C) built
(D) seen
(E) arrived
Ans : (B)

36. (A) meet


(B) receive
(C) homage
(D) acquaint
(E) welcome
Ans : (A)

37. (A) rebuke


(B) regards
(C) regret
(D) respect
(E) reverend
Ans : (D)

38. (A) had


(B) of
(C) filled
(D) contained
(E) full
Ans : (B)

39. (A) foolishly


(B) hasty
(C) similarly
(D) selfishly
(E) generously
Ans : (E)
40. (A) began
(B) hoping
(C) begged
(D) anxious
(E) wish
Ans : (C)

Bank of India Probationary Officers Exam., 2010


(Held on 24-1-2010)
General Awareness on Current Developments : Solved Paper
1. The President of the Palestine recently emphasized that his country will not resume peace talks
until Israel fully halts settlement building in the—
(A) West Bank
(B) Haifa
(C) Gaza
(D) Tel-Aviv-Yafo
(E) Jerusalem
Ans : (A)

2. All the major world leaders gathered in Berlin in Nov. 2009 to mark the 20th anniversary of—
(A) European Union
(B) NATO
(C) Fall of Berlin Wall
(D) G-20
(E) None of these
Ans : (C)

3. Now almost all major newspaper journals and magazines are printing research reports giving
the analysis and/or the causes of the sub-prime crisis which gripped America and the world a few
months back. Which of the following was/were amongst the common cause(s) of the same ?
(They were present in almost all the economies)
(1) The problem was that investors erroneously believed property prices were quite predictable
and built a whole edifice of financial planning on the back of the American housing market.
(2) Credit rating agencies all over the world were not equipped to forecast the effect of sub-prime
crisis on world economy. Agencies were over-confident and did not react in time.
(3) Neither USA nor other countries took a note of the crisis in time. In fact they ignored it for
quite some time.
(A) Only 1
(B) Only 2
(C) Only 3
(D) All 1, 2 and 3
(E) Only 1 and 2
Ans : (A)

4. If you see a big hoarding at a prominent public place, the punch line of which says ‘We All
Were Born Free’; ‘We All Have Equal Rights’, in all probability, the hoarding is put up by—
(A) National Commission for Farmers
(B) National Human Rights Commission
(C) Directorate of Income Tax
(D) Ministry of Foreign Affairs
(E) Union Public Service Commission
Ans : (B)

5. Nowadays we frequently read news items about ‘Derivatives’ as used in the world of finance
and money market. Which of the following statement(s) correctly describes what a derivative is
and how it affects money/finance markets ?
(1) Derivatives enable individuals and companies to insure themselves against financial risk.
(2) Derivatives are like fixed deposits in a bank and are the safest way to invest one’s idle money
lying in a bank.
(3) Derivatives are the financial instruments which were used in India even during the British
Raj.
(A) Only 3
(B) Only 2
(C) Only 1
(D) All 1, 2 and 3
(E) None of these
Ans : (B)
6. Many a time we read in the newspapers that RBI has changed or revised a particular ratio/rate
by a few basis points. What is basis point ?
(A) Ten per cent of one hundredth point
(B) One hundredth of 1%
(C) One hundredth of 10%
(D) Ten per cent of 1000
(E) None of these
Ans : (B)

7. Which of the following issues cannot come under the purview of the functioning of the
Human Rights Commission of a country ?
(A) Racial Discrimination
(B) Treatment to Prisoners of War
(C) Human Trafficking
(D) Child Abuse
(E) Climate Migration
Ans : (E)

8. A prominent international weekly sometime ago printed a caption on its cover page which
read ‘Brazil Takes Off’. Other major newspapers/magazines also printed similar stories/articles
in their publications at that time. Why have magazines/newspapers decided to talk about Brazil
these days ?
(1) All major economies of the world have been taking time to recover from the recession but
Brazil was one of those which was ‘Last in and First out’.
(2) Brazil is a member of BRIC but unlike China it is a democracy, unlike India, it has no hostile
neighbours, no insurgents and unlike Russia it exports more oil and arms and treats foreign
investors with more respect.
(3) Brazil is the world’s second largest booming economy.
(A) Only 1
(B) Only 2
(C) Only 3
(D) All 1, 2 and 3
(E) Only 1 and 2
Ans : (A)

9. Which of the following countries has conferred the honour of ‘Legion d’ honneur’ on Bharat
Ratna Lata Mangeshkar ?
(A) Germany
(B) Norway
(C) Japan
(D) U.K.
(E) France
Ans : (E)

10. Expand the term NREGA—


(A) National Rural Employment Guarantee Agency
(B) National Rural Electrification Governing Agency
(C) National Rural Employment Guarantee Act
(D) New Rural Employment Guarantee Agency
(E) None of these
Ans : (C)

11. The Rajya Sabha has recently cleared the new poll bill. Which one of the following
amendment(s) is/are made in this bill ?
It has proposed—
(A) Increasing the security deposits to more than double
(B) Restricting the publication of exit polls
(C) Ensuring speedy disposal of electoral disputes
(D) Only (A) and (B)
(E) All (A), (B) and (C)
Ans : (D)

12. The market in which long term securities such as stocks and bonds are bought and sold is
commonly known as—
(A) Commodities Exchange
(B) Capital Market
(C) Bull Market
(D) Bullion Market
(E) None of these
Ans : (B)

13. Which one of the following was India’s top destination for exports during 2009 ?
(A) UAE
(B) USA
(C) Russia
(D) China
(E) Bangladesh
Ans : (A)

14. Which one of the following will be the first High Court in India, to implement the concept of
‘ecourts’ ?
(A) Delhi
(B) A.P.
(C) Chennai
(D) Kolkata
(E) None of these
Ans : (A)

15. Amongst the following, which one of the following sectors provides the highest contribution
in Industrial Production Index ?
(A) Crude Oil
(B) Petro Refinery Products
(C) Electricity
(D) Coal
(E) None of these
Ans : (C)

16. Amongst the following sectors, which sector/segment has shown the highest per cent growth
in the current fiscal ?
(A) Mining
(B) Manufacturing
(C) Electricity, gas and water supply
(D) Banking and Finance
(E) None of these
Ans : (B)

17. As per the reports published in newspapers, India purchased around 200 tonnes of gold
(almost half the quantity of gold put up for sale) in Sept. 2009. India purchased this gold from
which of the following organizations ?
(A) World Bank
(B) Asian Development Bank
(C) International Monetary Fund
(D) International Gold Council
(E) None of these
Ans : (C)

18. World Trade Organisation (WTO)’s ministerial meeting was organized in which of the
following cities recently ?
(A) Geneva
(B) Washington
(C) Paris
(D) Port of Spain
(E) None of these
Ans : (A)

19. In which one of the following countries, was the recent Commonwealth Meet (CHOGAM)
held ?
(A) Trinidad
(B) Canada
(C) Australia
(D) Jamaica
(E) U.K.
Ans : (A)

20. Which of the following decisions taken by the RBI will promote the concept of financial
inclusion in the country ?
(A) To appoint some additional entities as business correspondents
(B) To collect reasonable service charges from the customer in a transparent manner for
providing the services.
(C) To ask the banks to open at least 50 new accounts daily in non serviced areas.
(D) Only (A) and (B)
(E) None of these
Ans : (E)

21. For recapitalization of Public Sector Banks, the World Bank has decided to provide funds to
India. These funds will be made available in the form of—
(A) Soft Loan
(B) Term Loan
(C) Emergency aid
(D) Grants
(E) None of these
Ans : (A)

22. Citizens of which one of he following age-groups (in years) are covered under the New
Pension System (NPS) ?
(A) 18–50
(B) 21–55
(C) 25–55
(D) 18–60
(E) None of these
Ans : (D)

23. Which one of the following is the objective of the flagship scheme ‘Rashtriya Swasthya
Bima Yojana’ (RSBY) ?
(A) To provide life insurance cover to rural households
(B) To provide health insurance cover to rural households
(C) To provide both life and health insurance cover to rural household
(D) To provide life and health insurance covers only to people living below poverty line
(E) None of these
Ans : (D)

24. Which one of the following had set up the N. R. Narayana Murthy Committee on issues
relating to Corporate Governance ?
(A) SEBI
(B) RBI
(C) CII
(D) Ministry of Finance, GOI
(E) None of these
Ans : (A)

25. The proceeds of the disinvestment of profitable public sector units will be used for which of
the following purposes ?
(A) Expansion of existing capacity of PSEs
(B) Modernisation of PSEs
(C) Opening of new PSEs
(D) Meeting the expenditure for various social sector schemes
(E) None of these
Ans : (D)

26. Which one of the following is the target fixed for fiscal deficit in the year 2010-2011 ?
(A) 3•5%
(B) 4•0%
(C) 5•5%
(D) 5•0%
(E) None of these
Ans : (C)

27. Which one of the following companies is the largest producer of natural gas in the country ?
(A) ONGC
(B) OIL
(C) Cairn India
(D) RIL
(E) None of these
Ans : (A)

28. Which one of the following states has sought a package of Rs. 500 crores ($ 100 million) for
its Rural Poverty Reduction Programme ?
(A) M.P.
(B) Tamil Nadu
(C) A.P.
(D) Karnataka
(E) None of these
Ans : (C)

29. Chinese objections have stalled the road work at the village of Demchok, on the Indian side
of the line of control. In which one of the following states is this village located ?
(A) Arunachal Pradesh
(B) Assam
(C) Himachal Pradesh
(D) Rajasthan
(E) None of these
Ans : (E)

30. Constitutionally, which one of the following can levy Service Tax ?
(A) Union Govt. only
(B) State Govt. only
(C) Union Territory Govt. only
(D) All of these
(E) None of these
Ans : (D)

31. In the proposed low cost pension scheme, which one of the following has been made
responsible for maintaining of the records of pension account of an individual ?
(A) SIDBI
(B) National Securities Depositories Ltd. (NSDL)
(C) Stock Holding Corporation of India
(D) RBI
(E) None of these
Ans : (B)

32. Which one of the following is/are implication(s) of large inflow of foreign exchange into the
country ?
1. It makes monetary management difficult for RBI.
2. It creates money supply, asset bubbles and inflation.
3. It weakens the competitiveness of Indian exports.
(A) Only 1
(B) Only 2
(C) Only 3
(D) Only 1 and 3
(E) All 1, 2 and 3
Ans : (B)

33. The Central Banks of which one of the following countries has offered $ 115 billion
emergency credit to support its economy hit by falling prices and also to strengthen its currency ?
(A) South Korea
(B) Japan
(C) U.K.
(D) USA
(E) None of these
Ans : (B)

34. Which of the following books is written by Chetan Bhagat ?


(A) The Golden Gate
(B) A House for Mr. Biswas
(C) 2 States
(D) White Tiger
(E) None of these
Ans : (C)

35. NTPC Ltd. is in the process of exploring opportunities to source coal from overseas. In
which one of the following countries has the NTPC identified two new coal mines ?
(A) Bhutan
(B) Australia
(C) South Korea
(D) Indonesia
(E) None of these
Ans : (D)

36. Which of the following organizations has raised its loan amount to India to make it US $ 7
billion in 2009-10 fiscal ?
(A) Asian Development Bank
(B) International Monetary Fund
(C) World Bank
(D) European Union
(E) None of these
Ans : (C)

37. Who amongst the following cricket players became the first to score 17000 runs in One Day
Internationals ?
(A) Rahul Dravid
(B) Stephen Fleming
(C) Ricky Ponting
(D) Sachin Tendulkar
(E) None of these
Ans : (D)

38. The Govt. of India has decided to use the Core Banking System of banks to ensure proper
usage of funds provided for which of its following programmes ?
(A) Literacy Mission
(B) Pradhanmantri Gram Sadak Yojana
(C) National Health Mission
(D) All of these
(E) None of these
Ans : (A)

39. Famous Dada Saheb Phalke Award is given for exemplary work in the field of—
(A) Sports
(B) Literature
(C) Science & Technology
(D) Social Service
(E) None of these
Ans : (E)

40. The Reverse Mortgage Scheme was launched by some organizations a few years back. This
was done to help which of the following sections of society ?
(A) Beneficiaries of the NREGA
(B) People living Below Poverty Line
(C) Youngsters
(D) Senior Citizens
(E) None of these
Ans : (D)
41. The organization of the South East Asian countries is known as—
(A) OPEC
(B) ASEAN
(C) NATO
(D) UNCTAD
(E) SAARC
Ans : (B)

42. ‘Miss Earth 2009’ title has been conferred upon—


(A) Alejandra Pedrajas
(B) Sandra Saifert
(C) Jessica Barboza
(D) Larissa Ramos
(E) None of these
Ans : (D)

43. Which of the following countries became the third largest producer of Steel in the world in
2009 ?
(A) China
(B) India
(C) USA
(D) Japan
(E) None of these
Ans : (B)

44. The Foreign Trade policy is announced recently by the Govt. of India. Which of the
following ministries/agencies announce the same ?
(A) Reserve Bank of India
(B) Export Import Bank of India (EXIM Bank)
(C) Ministry of Foreign Affairs
(D) Ministry of Commerce & Industry
(E) None of these
Ans : (D)

45. Irina Bokova has taken over as the Chief of the—


(A) ILO
(B) UNICEF
(C) OPEC
(D) UNO
(E) UNESCO
Ans : (E)

46. Dorjee Khandu has taken over as the Chief Minister of—
(A) Meghalaya
(B) Uttarakhand
(C) Arunachal Pradesh
(D) Assam
(E) None of these
Ans : (C)

47. Which of the following terms is used in the game of Football ?


(A) Caddy
(B) Cutback
(C) Mid-on
(D) Love
(E) None of these
Ans : (B)

48. Which of the following is considered a Non Food Crop ?


(A) Wheat
(B) Maize
(C) Bajra
(D) Rice
(E) Jute
Ans : (E)

49. Who amongst the following was the Prime Minister of Nepal immediately prior to Madhav
Kumar Nepal ?
(A) Ram Baran Yadav
(B) Pushpakamal Dahal
(C) Girija Prasad Koirala
(D) Mahendra Kumar Suryavanshi
(E) None of these
Ans : (B)

50. The 10th India-European Summit was held in New Delhi in November 2009. Who amongst
the following represented India and also chaired the same ?
(A) S. M. Krishna
(B) Pratibha Patil
(C) Pranab Mukherjee
(D) Manmohan Singh
(E) None of these
Ans : (D)

Indian Bank Probationary Officer Exam., 2010


(Held on 17-1-2010)
English Language : Solved Paper

Directions—(Q. 1–5) Which of the phrases (A), (B), (C) and (D) given below should replace the
phrase given in bold in the following sentence to make it meaningful and grammatically correct.
If the sentence is correct as it is and ‘No correction is required’, mark (E) as the answer.

1. We have accepted over two billion dollars from them yet it is been used to build hospital in
the area—
(A) that is yet to use
(B) although it has been used
(C) not yet being used
(D) which will be used
(E) No correction required
Ans : (C)

2. People have respond in favour the government’s efforts to resolve the budget crisis—
(A) favourably responded
(B) response in favour of
(C) responded favourably to
(D) been responding favourably
(E) No correction required
Ans : (C)

3. Striking this deal will enable the company to expand its operations in Europe—
(A) strike this deal that
(B) to strike off this deal
(C) by striking this deal to
(D) this deal was struck which
(E) No correction required
Ans : (E)

4. We admire they are taking this step despite the numerous risks involved—
(A) them to take
(B) them for taking
(C) that they have taken over
(D) how their taking of
(E) No correction required
Ans : (B)

5. The management has been left with no option else to change the branch timings to avoid
losing business.
(A) but to change
(B) except the change of
(C) unless it changes with
(D) other than the changing
(E) No correction required
Ans : (A)
Directions—(Q. 6–20) Read the following passage carefully and answer the questions given
below it. Certain words have been printed in bold to help you locate them while answering some
of the questions—

The great fear in Asia a short while ago was that the region would suffer through the wealth
destruction already taking place in the U.S. as a result of the financial crisis. Stock markets
tumbled as exports plunged and economic growth deteriorated. Lofty property prices in China
and elsewhere looked set to bust as credit tightened and buyers evaporated. But with surprising
speed, fear in Asia swung back to greed as the region shows signs of recovery and property and
stock prices are soaring in many parts of Asia.

Why should the sharp Asian turnaround be greeted with scepticism ? Higher asset prices mean
households feel wealthier and better able to spend, which could further fuel the region’s nascent
rebound. But just as easily, Asia could soon find itself saddled with overheated markets similar
to the U.S. housing market. In short the world has not changed, it has just moved places.

The incipient bubble is being created by government policy. In response to the global credit
crunch of 2008, policy makers in Asia slashed interest rates and flooded financial sectors with
cash in frantic attempts to keep loans flowing and economies growing. These steps were logical
for central bankers striving to reverse a deepening economic crisis. But there’s evidence that
there is too much easy money around. It’s winding up in stocks and real estate, pushing prices up
too far and too fast for the underlying economic fundamentals. Much of the concern is focused
on China, where government stimulus efforts have been large and effective. Money in China has
been especially easy to find. Aggregate new bank lending surged 201% in the first half of 2009
from the same period a year earlier, to nearly $ 1.1 trillion. Exuberance over a quick recovery–
which was given a boost by China’s surprisingly strong 7.9% GDP growth in the second quarter–
has buoyed investor sentiment not just for stocks but also for real estate.

Former U.S. Federal Reserve Chairman Alan Greenspan argued that bubbles could only be
recognised in hindsight. But investors–who have been well schooled in the dangers of bubbles
over the past decade are increasingly wary that prices have risen too far, and that the slightest bit
of negative economic news could knock markets for a loop. These fears are compounded by the
possibility that Asia’s central bankers will begin taking steps to shut off the money. rumours that
Beijing was on the verge of tightening credit led to Shanghai stocks plunging 5%. Yet many
economists believe that, “there is close to a zero possibility that the Chinese government will do
anything this year that constitutes tightening.” And without a major shift in thinking, the easy-
money conditions will stay in place. In a global economy that has produced more dramatic ups
and downs than anyone thought possible over the past two years, Asia may be heading for
another disheartening plunge.

6. To which of the following has the author attributed the 2008 Asian financial crisis ?
(1) Reluctance of Asian governments to taper off the economic stimulus
(2) Greed of Asian investors causing them to trade stocks of American companies at high prices
(3) Inflated real estate prices in Asian countries
(A) None
(B) Only (1)
(C) Only (3)
(D) Only (1) and (2)
(E) Only (2)
Ans : (D)

7. What does the author want to convey through the phrase “The world has not changed it has
just moved places” ?
(A) At present countries are more dependent on Asian economics than on the US economy
(B) Economies has become interlinked on account of globalisation
(C) Asian governments are implementing the same economic reforms as developed countries
(D) All economies are susceptible to recession because of the state of the US economy
(E) None of these
Ans : (A)

8. Which of the following can be said about the Chinese government’s efforts to revive the
economy ?
(A) These were largely unsuccessful as only the housing market improved
(B) The government’s only concern was to boost investor confidence in stocks
(C) These efforts were ineffectual as the economy recovered owing to the US market stabilising
(D) These were appropriate and accomplished the goal of economic revival
(E) They blindly imitated the economic reforms adopted by the US
Ans : (B)

9. Why do experts predict that Asian policy makers will not withdraw fiscal stimulus ?
(1) The US economy is not likely to recover for a long time
(2) Stock markets are yet to regain their former levels
(3) Fear of revolt by greedy citizens
(A) None
(B) Only (3)
(C) Only (1) and (3)
(D) Only (2)
(E) Only (2) and (3)
Ans : (D)

10. What do the statistics about loans given by Chinese banks in 2009 indicate ?
(A) There was hardly any demand for loans in 2008
(B) The Chinese government has borrowed funds from the U.S.
(C) China will take longer than the US to recover from the economic crisis
(D) The GDP of China was below expectations
(E) None of these
Ans : (D)

11. Why has investor confidence in the Chinese stock market been restored ?
(1) Existing property prices which are stable and affordable
(2) The government has decided to tighten credit
(3) Healthy growth of the economy indicated by GDP figures
(A) Only (3)
(B) Only (1) and (2)
(C) All (1), (2) and (3)
(D) Only (2)
(E) None of these
Ans : (A)

12. What is the author’s main objective in writing the passage ?


(A) Illustrating that Asian economies are financially more sound than those of developed
countries
(B) Disputing financial theories about how recessions can be predicted and avoided
(C) Warning Asian countries about the dangers of favouring fast growth and profits over sound
economic principles
(D) Extolling China’s incredible growth and urging other countries to emulate it
(E) Advising governments about the changes in policy to strengthen economic fundamentals
Ans : (C)

13. Why does the author doubt the current resurgence of Asian economics ?
(A) Their economies are too heavily reliant on the American economy which is yet to recover
(B) Central banks have slashed interest rates too abruptly which is likely to cause stock markets
to crash
(C) With their prevailing economic conditions they are at risk for a financial crisis
(D) Their GDP has not grown significantly during the last financial year
(E) None of these
Ans : (B)

14. Which of the following can be inferred from the passage ?


(1) All Asian economies are recovering at the same pace
(2) Experts are apprehensive about the state of Asian economies despite their recovery
(3) Developed countries should implement the same economies reforms as Asian ones
(A) Only (1)
(B) Only (2) and (3)
(C) Only (1) and (2)
(D) Only (2)
(E) None of these
Ans : (D)

15. According to the passage, which of the following factor(s) has/ have had a negative impact
on the Asian stock markets ?
(1) Abrupt drop in exports by Asian countries
(2) Extravagant disbursement of housing loans in 2009
(3) Raising of interest rates by the Central Bank
(A) None
(B) Only (1) and (2)
(C) Only (1)
(D) Only (1) and (3)
(E) All (1), (2) and (3)
Ans : (B)

Directions—(Q. 16–18) Choose the word or group of words which is MOST SIMILAR in
MEANING to the word printed in bold as used in the passage.

16. Fuel
(A) Petrol
(B) Stimulate
(C) Sustain
(D) Heat
(E) Charge
Ans : (B)

17. Flooded
(A) Surged
(B) Saturated
(C) Overflowed
(D) Deluge
(E) Overcome
Ans : (C)

18. Evaporated
(A) Dehydrated
(B) Melted
(C) Vaporised
(D) Vanished
(E) Dodged
Ans : (D)

Directions—(Q. 19-20) Choose the word or group of words which is MOST OPPOSITE in
MEANING to the word printed in bold as used in the passage.

19. Buoyed
(A) Heavy
(B) Stifled
(C) Numbed
(D) Dull
(E) Abated
Ans : (E)

20. Sharp
(A) Blunt
(B) Incomplete
(C) Naive
(D) Indistinct
(E) Gradual
Ans : (E)

Directions—(Q. 21–30) Read each sentence to find out whether there is any grammatical
mistake/error in it. The error if any, will be in one part of the sentence. Mark the letter of that
part with error as your answer. If there is ‘No error’, mark (E).

21. Arranging such a large amount (A) /of funds now will be a problem why (B) / banks are
usually not open (C) /so early in the morning. (D) No error (E)
Ans : (C)

22. He had telephoned yesterday to (A) /ask how much of the youth (B) /who attend our classes
would be (C) /interested in working for a textile company. (D) No error (E)
Ans : (B)

23. Though he has promoted to (A) / the bank’s board as a director (B) / he continues to carry out
(C) / all his current responsibilities. (D) No error (E)
Ans : (A)

24. The Board’s decision has provided (A) / employees with the opportunity (B) / to acquire upto
100 (C) / shares by the company. (D) No error (E)
Ans : (D)

25. If the manufacturing sector continues (A) / to grow at the same rate for (B) / the next few
months, I think it (C) / has a high growth rate this year. (D) No error (E)
Ans : (D)

26. The government is working (A) / out a new system to compensate (B) / those companies to
sell (C) / products below the market price. (D) No error (E)
Ans : (C)

27. The success of the (A) / government sponsor job guarantee programme (B) / has resulted in a
(C) / drastic drop in poverty. (D) No error (E)
Ans : (B)

28. We were forced into react (A) / as no organisation can (B) / afford to adhere to (C) / these
outdated regulations. (D) No error (E)
Ans : (A)

29. We had extensively discussions (A) / with the participants and (B) / obtained their feedback
(C) / regarding our new services. (D) No error (E)
Ans : (A)

30. Their failure to inspect (A) / our factories is a (B) / clear indications that our (C) / license will
not be renewed. (D) No error (E)
Ans : (C)
Directions—(Q. 31–35) In each question below a sentence with four words printed in bold type
is given. These are lettered as (A), (B), (C) and (D). One of these four words printed in bold
may be either wrongly spelt or inappropriate in the context of the sentence. Find out the word
which is wrongly spelt or inappropriate if any. The letter of that word is your answer. If all the
words printed in bold are correctly spelt and also appropriate in the context of the sentence, mark
(E) i.e. ‘All correct’ as your answer.

31. Though these programmes have proved (A) to be extremely (B) effective (C) they do have
certain drawbacks. (D) All correct (E)
Ans : (E)

32. According to these estimates (A) our profitable (B) margin (C) will be higher if we adopt
(D) this approach. All correct (E)
Ans : (B)

33. In order to confront (A) the threat (B) of global warming it is imperative (C) that we work
altogether. (D) All correct (E)
Ans : (D)

34. Any failure (A) to complicit (B) with these fundamental (C) regulations will result in a
fine. (D) All correct (E)
Ans : (B)

35. Every organization needs to be proactive (A) in devising (B) stratergies (C) to ensure the
retention (D) in staff. All correct (E)
Ans : (C)

Directions—(Q. 36–40) Rearrange the following six sentences 1, 2, 3, 4, 5 and 6 in the proper
sequence to form a meaningful paragraph; then answer the questions given below them—
1. However if this happens it will cause problems for the elderly who mainly use cheques.
2. The use of cheques has fallen dramatically in the past few years.
3. Thus cheques may be phased out gradually making sure that the needs of all consumers
including the elderly are met.
4. This is because more and more consumers are transferring money electronically by direct
debit or credit cards.
5. Without cheques they are likely to keep large amounts of cash in their homes making them
vulnerable to theft.
6. British banks have thus voted to phase cheques out in favour of these more modern payment
methods.

36. Which of the following should be the FIFTH sentence after rearrangement ?
(A) 1
(B) 2
(C) 3
(D) 4
(E) 5
Ans : (C)

37. Which of the following should be the FIRST sentence after rearrangement ?
(A) 2
(B) 3
(C) 4
(D) 5
(E) 6
Ans : (A)

38. Which of the following should be the SECOND sentence after rearrangement ?
(A) 1
(B) 2
(C) 3
(D) 4
(E) 6
Ans : (D)

39. Which of the following should be the LAST (SIXTH) sentence after rearrangement ?
(A) 2
(B) 3
(C) 4
(D) 5
(E) 6
Ans : (E)

40. Which of the following should be the THIRD sentence after rearrangement ?
(A) 2
(B) 3
(C) 4
(D) 5
(E) 1
Ans : (E)

Directions—(Q. 41–50) In the following passage there are blanks, each of which has been
numbered. These numbers are printed below the passage and against each, five words are
suggested, one of which fits the blank appropriately. Find out the appropriate word in each case.

The World Diabetes Congress has determined that India has the largest number of diabetics in
the world. Apart from the loss of productivity, the …(41)… burden is alarming – $ 2.8 billion
annually. Sedentary jobs, …(42)… of electronic entertainment, changing diet patterns and …(43)
… dependence on automobiles have driven the activity …(44)… of Indians’ lives especially in
cities.

The …(45)… is, therefore, to make people physically …(46)… and requires interventions which
imapct a large …(47)… of the population. Admittedly physical activity is a …(48)… of choice
and is strongly driven by …(49)… preferences. But policymaking needs to shift to …(50)…
moderate levels of physical activity in the daily lives of people. One way to accomplish this is to
create walkable communities that give residents a variety of destinations within walking
distance.

41. (A) economic


(B) finance
(C) subsidy
(D) physical
(E) health
Ans : (B)

42. (A) broadcast


(B) spread
(C) prevalent
(D) expand
(E) widespread
Ans : (E)

43. (A) totally


(B) entirely
(C) grown
(D) mutual
(E) increasing
Ans : (E)

44. (A) most


(B) out
(C) from
(D) through
(E) outside
Ans : (B)

45. (A) ultimatum


(B) hazard
(C) sensitivity
(D) challenge
(E) dispute
Ans : (A)

46. (A) equip


(B) built
(C) active
(D) trained
(E) qualified
Ans : (C)

47. (A) section


(B) scale
(C) degree
(D) per cent
(E) piece
Ans : (A)

48. (A) want


(B) matter
(C) scarcity
(D) right
(E) lack
Ans : (B)

49. (A) showing


(B) given
(C) special
(D) personal
(E) individually
Ans : (D)

50. (A) pursuit


(B) indulge
(C) introduce
(D) insist
(E) attract
Ans : (C)

Baroda Rajasthan Gramin Bank Officers Exam., 2010


Held on 21-02-2010
English Language : Solved Paper
Directions—(Q. 1–15) Read the following passage carefully and answer the questions given
below it. Certain words have been printed in bold to help you locate them while answering some
of the questions.

Evidence is growing that relatively cheap policies like climate engineering and non-carbon
energy research could effectively prevent suffering from global warming, both in the short and
long term. Unfortunately, political leaders gathering at a special meeting of the United Nations in
New York will focus on a very different response. They will make many of the most important
decisions on how to respond to climate change over the next decade. They are expected to thrash
out political disputes like how much carbon rich and poor nations should agree to cut.

We have failed to rein in emission rises despite sincere and well-meaning promises made in
Kyoto in 1997 and earlier, because carbon cuts are expensive to enact. Research by climate
economists show that significant carbon cuts could cost a staggering 12•9% of global GDP in
2100. Available estimates show that for each dollar spent on global carbon cuts, we buy two
cents worth of avoided climate damage. The solution is far more costly than the problem. Thus a
global deal based around carbon cuts is expected to include a lot of spending from rich countries
to help poor nations to prepare for global warming. Developed countries too apparently seem to
have no problems in spending much money to save few lives in the distant future, instead of
combating malnutrition, malaria, or communicable diseases today. It is amoral to build a dam to
avoid flooding in 100 years, when the people living beside that dam are starving today.

Imagine if we could fix climate for the next hundred years for less than what a single country
spends on climate research in a year. Climate engineering has the potential to do just that. One
can explore the costs and benefits of so-called marine cloud whitening, a well-established
techproposal in which seawater droplets would be sprayed into clouds above the sea to make
them reflect more sunlight back into space thus augmenting the natural process where sea salt
helps to provide tiny particles for clouds to form around. About $9 billion spent developing this
technology might be able to cancel out this century’s global warming. The benefits from
preventing the temperature increase would add up to about $20 trillion. We should research this
technology today to identify its limitations, risks and potential so that it could buy us a century’s
delay in warming.

But this will not be sufficient because we need better non-carbonbased technology options. Non-
fossil sources like nuclear, wind, solar and geothermal energy will get us quite some way
towards the path of stable carbon emissions.

Policy makers should abandon carbon-reduction negotiations and make agreements to seriously
invest in research and development. As research spending would be much cheaper than carbon-
emission cuts, there would be a much higher chance of political agreement, and a much higher
probability of the promises being enacted. We have within our grasp alternative policy options
that would truly leave the planet in a better state.

1. Why have past efforts to reduce carbon emissions failed ?


(A) Scientists were not interested in this field of research
(B) Global warming was not considered as a problem by the U.N.
(C) The cost of accomplishing this was too high
(D) Poor nations did not have the necessary knowledge to reduce carbon emissions
(E) None of these

2. Which of the following is a characteristic of climate engineering ?


(A) It is beneficial in the short term and also in the long term
(B) It can only be adopted by poor countries
(C) It has worsened problems like poverty and hunger in developing countries
(D) It is seen as less cost effective than other ways of reducing pollution
(E) It uses technology which pollutes the environment

3. According to the author, which of the following is/are the outcome/s of the meeting of world
leaders in New York ?
(1) A resolution that carbon emissions will be reduced through climate engineering.
(2) Successful settlement of many conflicts regarding reduction of emissions.
(3) Developed countries have volunteered to spend over 12 per cent of their GDP on preventing
global warming.
(A) Only (1)
(B) Only (2)
(C) Only (1) and (3)
(D) All (1), (2) and (3)
(E) None of these

4. What does the author want to convey through the phrase ‘Nonfossil sources like nuclear,
wind, solar and geothermal energy will get us quite some way towards the path of stable
carbon emissions’ as given in the passage ?
(A) All countries should use non fossil sources of energy to be able to achieve high carbon
emissions
(B) The use of non fossil sources of energy will be able to check the carbon emissions
(C) Non fossil sources of fuel may actually increase carbon emissions
(D) If we use such non fossil fuels, it will take a long time to reduce carbon emissions
(E) None of these

5. Which of the following is true in the context of the passage ?


(A) Carbon emissions of poor countries are higher than those of rich ones
(B) Construction of dams is directly responsible for the starvation of millions
(C) The earth is in danger of extinction in a century
(D) There is a lack of consensus between rich and poor nations on the issue of reducing carbon
emissions
(E) Countries have to spend a substantial amount of their GDP on climate change

6. What is the author’s opinion about agreement among countries in Kyoto ?


(A) Countries which signed this agreement had no intention of reducing carbon emissions
(B) The percentage of carbon emissions to be reduced was highest for developing countries
(C) Climate economists should not have supported this agreement
(D) It was a failure because developed countries backed out from the agreement
(E) None of these

7. What is the author’s solution to handling climate change ?


(A) Countries should use more expensive and long lasting ways of reducing carbon emissions
(B) Negotiations on climate change should be between scientists rather than between political
leaders
(C) Countries should increase expenditure on research into ways of handling climate change
(D) The U.N. should finance climate research in developing countries
(E) Introduce economic sanctions against countries which do not reduce carbon emissions

8. Which of the following will be a suitable title for the given passage ?
(A) Risks of global warming
(B) The depletion of fossil fuel reserves
(C) Technology can fight global warming
(D) Global warming and decreasing GDPs
(E) The inefficient non carbon sources of energy

9. What is the author’s aim in promoting research into ‘marine cloud whitening’ ?
(A) To better understand how such technology can be utilized as an efficient solution to the
problem of global warming
(B) To ensure that the climate scientists benefit financially
(C) To give developing nations an advantage over developed nations in fighting global warming
(D) To highlight the risks of such techniques
(E) To delay agreements on global warming for as long as possible

10. According to the author, what role will the rich nations play in reducing carbon emissions ?
(A) Pressurising poor nations to sign agreements on carbon cuts
(B) Funding carbon emission cuts in developing countries
(C) Diverting research funding from disease prevention to environment protection
(D) Spending trillions of dollars annually on non carbon energy research
(E) None of these

Directions—(Q. 11–13) Choose the word which is most similar in meaning to the word printed
in bold as used in the passage.

11. BUY
(A) Accept
(B) Pay
(C) Provide
(D) Bargain
(E) Cost

12. STAGGERING
(A) Swaying
(B) Huge
(C) Shaking
(D) Unsteady
(E) Collapsing

13. STATE
(A) Express
(B) Mess
(C) Official
(D) Say
(E) Condition

Directions—(Q. 14–15) Choose the word/phrase which is most opposite in meaning to the word
printed in bold as used in the passage.

14. AUGMENTING
(A) Reducing
(B) Growing
(C) Channelising
(D) Criticising
(E) Supplementing

15. COMBATING
(A) Holding
(B) Forgiving
(C) Caring
(D) Supporting
(E) Fighting

Directions—(Q. 16–24) Read each sentence to find out whether there is any grammatical error
in it. The error if any will be in one part of the sentence, the letter of that part will be the answer.
If there is no error, mark (E) as the answer. (Ignore errors of punctuation, if any.)

16. India demonstrates its supremacy (A) / in space when it successfully (B) / launched its third
satellite (C) / into orbit yesterday. (D) No error (E)

17. India needs a value education system (A) /who will inculcate values (B) / among the students
and (C) / enrich their personalities. (D) No error (E)

18. Driven by the desire to save trees, (A) / residents of a locality (B) / has started using solar
appliances (C) / for their everyday needs. (D) No error (E)

19. A large number of unmanned aircrafts (A) / being used by the military (B) / are suspected of
having (C) / unsafe radio links. (D) No error (E)

20. Recent survey shows that (A) / 35 million children in the age group of (B) / 6 to 10 years
have never (C) / attended no primary school. (D) No error (E)

21. Coal mines constitute (A) / a major percentage of the (B) / sources which cause damage (C) /
on the environment. (D) No error (E)

22. A man who has been (A) / accused of fraud in (B) / an earlier job he will never be (C) /
welcome in any other organization. (D) No error (E)

23. Worried about the continuing violence in the city, (A) / much students are set (B) (C) / to
migrate to other cities / for higher education. (D) No error (E)

24. Many organizations have been offering (A) / attractive incentives to (B) / their employees in
an attempt (C) / to boosting employee retention. (D) No error (E)
Directions—(Q. 25–29) Which of the phrases (A), (B), (C) and (D) given below each statement
should replace the phrase printed in bold in the sentence to make it grammatically correct ? If the
sentence is correct as it is given and ‘No correction is required’, mark (E) as the answer.

25. In an attempt to grow economically, India plans to march towards an open economy by
opened its doors to global markets.
(A) Open doors towards its
(B) Opening its doors to
(C) Open its doors in
(D) Opening their doors for
(E) No correction required

26. Though poverty is still rampant in India, economic growth and commercial development has
served to reduce them substantially over the years.
(A) Has served to reduce it
(B) Is serving to reduce it
(C) Had served to reduce them
(D) Have served to reduce it
(E) No correction required

27. All witnesses of yesterday’s accident has been questioned by the police but none could
identify the culprits.
(A) Every witness of yesterday’s
(B) All witnesses for yesterday’s
(C) Most witnesses of yesterday
(D) Many witnesses of yesterday
(E) No correction required

28. In spite of the rapid development of medical science, production of artificial blood has
remained a distant dream for many scientists.
(A) Despite of the rapid
(B) As a result of the rapid
(C) In spite of some rapidly
(D) Because of the rapid
(E) No correction required

29. India has millions of job opportunities but there is a serious shortage of educated
professionals whom are actually employable.
(A) Those are actually employed
(B) That is actually employed
(C) Who are actually employable
(D) Which is actually employed
(E) No correction required

Directions—(Q. 30–34) Rearrange the following sentences (1), (2), (3), (4), (5) and (6) to make
a meaningful paragraph and then answer the questions which follow :
(1) The government too has not left any stone unturned in promoting the ‘go green’ concept
among the construction industrialists.
(2) It has succeeded in luring the builders into ‘going green’ by these incentives, thus mutually
benefiting the builders as well as the environment.
(3) It means that the construction of their buildings ensures energy efficiency, water conservation
and use of recycled and renewable energy sources.
(4) It has offered many attractive packages, subsidies and tax benefits to the builders who follow
this concept.
(5) As the construction industry revives from recession and begins to bloom once again, the
latest trend to catch the eye of the developers is to ‘go green’.
(6) All these not only have a positive impact on the environment but also prove to be more
economical for the builders as well as the residents in the long run.

30. Which of the following sentence should be the FIRST after rearrangement ?
(A) 1
(B) 2
(C) 3
(D) 4
(E) 5

31. Which of the following sentence should be the SECOND after rearrangement ?
(A) 1
(B) 2
(C) 3
(D) 5
(E) 6

32. Which of the following sentence should be the THIRD after rearrangement ?
(A) 1
(B) 5
(C) 4
(D) 6
(E) 3

33. Which of the following sentence should be the FIFTH after rearrangement ?
(A) 4
(B) 2
(C) 3
(D) 5
(E) 6

34. Which of the following sentence should be the SIXTH (LAST) after rearrangement ?
(A) 3
(B) 2
(C) 4
(D) 5
(E) 6

Directions—(Q. 35–44) In the following passage there are blanks, each of which has been
numbered. These numbers are printed below the passage and against each, five words/phrases
are suggested, one of which best fits the blank appropriately. Find out the appropriate
word/phrase in each case.

When we sit down for a meal of market-bought products, we like to think we’re getting a
reasonable …(35)… of the body’s nutrient requirements, but studies show that because of our
chemical intensive farming, the food that we have, does not …(36)… the vital nutrients that our
ancestors enjoyed.

Plants …(37)… with the help of soluble chemical fertilisers get lazy and do not develop the
deep, healthy roots systems that pull additional elements out of the …(38)… In addition, the
micro-organisms that break down organic matter and minerals to be taken up by plant root are …
(39)… by chemical bombardment and violent mechanised manipulation of their environment.
Essentially, we’re getting robbed, and having to pay for it in …(40)… health, energy, longevity,
and advancing medical bills.

Unfortunately, agriculture’s single-minded focus on increasing …(41)… at any cost over the last
half-century created a blind spot where rapid …(42)… of the nutritional quality of our food has
occurred. This decline has been observed to be quite …(43)… in some crops but has sadly gone
largely …(44)… by scientists, farmers, government and consumers.

35. (A) yield


(B) number
(C) amount
(D) deficiency
(E) strength

36. (A) grow


(B) specify
(C) inhabit
(D) cultivate
(E) contain

37. (A) growing


(B) eaten
(C) suffering
(D) arising
(E) rising

38. (A) pesticides


(B) plants
(C) food
(D) soil
(E) crop

39. (A) increased


(B) absorbed
(C) added
(D) wounded
(E) killed

40. (A) improving


(B) declining
(C) better
(D) simplifying
(E) defective

41. (A) yields


(B) fertilisers
(C) chemicals
(D) quality
(E) soil

42. (A) immigration


(B) improvement
(C) return
(D) progression
(E) deterioration

43. (A) more


(B) inadequate
(C) significant
(D) inferior
(E) resistant

44. (A) improved


(B) unnoticed
(C) eradicated
(D) alarming
(E) unaware

Directions—(Q. 45–50) Each question below has two blanks, each blank indicating that
something has been omitted. Choose the set of words for each blank that best fits the meaning of
the sentence as a whole.

45. The ……… of his crime was a heavy blow as not only did his reputation …… but the
business also declined.
(A) punishment, finished
(B) revelation, improve
(C) disclosure, suffer
(D) committing, lost
(E) realisation, hurt

46. The Secretaryship has been …… for a long time as nobody is ……… to undertake duties of
the post.
(A) open, reluctant
(B) taken, interested
(C) empty, capable
(D) occupied, volunteered
(E) vacant, willing

47. The innocent man could have easily defendend himself but he ……… to speak as he was
……… of offending his friend.
(A) refused, afraid
(B) decided, unwilling
(C) intended, concerned
(D) declined, inclined
(E) denied, forced

48. ………… use of pesticides, especially DDT has been held responsible for the ………
population of vultures in various states across India.
(A) careless, uncontrolled
(B) multiple, increasing
(C) alarming, distinct
(D) indiscriminate, declining
(E) much, growing

49. Many rebels were mercilessly slain to ………… the rebellion but a few managed to
………… by hiding in woods and marshes.
(A) surpass, evade
(B) end, fight
(C) suppress, escape
(D) incite, run
(E) promote, revolt

50. Agriculture in India ……… over all other sectors because it plays a ………… role in the
sociocultural life of its people.
(A) dominates, minor
(B) prevails, vital
(C) important, significant
(D) survives, minimal
(E) beats, critical

Answers :
1. (C) 2. (A) 3. (E) 4. (B) 5. (D) 6. (D) 7. (B) 8. (C) 9. (A) 10. (A)
11. (E) 12. (B) 13. (E) 14. (A) 15. (D) 16. (A) 17. (B) 18. (C) 19. (E) 20. (D)
21. (D) 22. (C) 23. (B) 24. (D) 25. (B) 26. (D) 27. (A) 28. (E) 29. (C) 30. (E)
31. (C) 32. (A) 33. (B) 34. (E) 35. (C) 36. (E) 37. (A) 38. (D) 39. (E) 40. (B)
41. (A) 42. (E) 43. (C) 44. (B) 45. (C) 46. (E) 47. (A) 48. (D) 49. (C) 50. (B)

Union Bank of India Clerk Exam., 2010


(Held on 10-1-2010)
General English : Solved Paper
Directions—(Q. 1–5) In each question below a sentence with four words printed in bold type is
given. These are lettered as (A), (B), (C) and (D). One of these four boldly printed words may
be either wrongly spelt or inappropriate in the context of the sentence. Find out the word
which is wrongly spelt or inappropriate, if any. The letter of that word is your answer. If all the
boldly printed words are correctly spelt and also appropriate in the context of the sentence, mark
(E) i.e., ‘All Correct’ as your answer.

1. RBI is unwilling (A) to enforce (B) these reuglations as these will discourage (C)
investment. (D) All Correct (E)
Ans : (E)

2. Many Indian firms have entry (A) into partnership (B) with foreign (C) ones of late. (D) All
Correct (E)
Ans : (A)

3. This is a prime (A) example (B) of what the government can achieve (C) if it is determine.
(D) All Correct (E)
Ans : (D)

4. They managed (A) to accomplice (B) this by coming (C) up with unique (D) schemes. All
Correct (E)
Ans : (B)

5. In such circumstanses (A) do not take unnecessary (B) risks (C) with your savings. (D) All
Correct (E)
Ans : (A)

Directions—(Q. 6–20) Read the following passage carefully and answer the questions given
below it. Certain words have been printed in bold to help you locate them while answering some
of the questions.

Rajendra was a landlord. He had made a lot of money by cheating and ill-treating his laboures
who worked in the fields. One day, a young man named Mani came to Rajendra, asking for
work. Rajendra was pleasantly surprised. No one had ever wanted to work for him because of
his reputation, and here was someone walking right into his house ! Mani’s next few words
surprised him even more. Mani said, “I will work for you for free. Only give me a place to sleep,
two sets of clothes and two square meals a day.” Rajendra was beside himself with joy when he
heard this and was about to agree, when Mani added, “I have only one condition : I will tell you
the truth always, but one day in the year, I will lie.”

Rajendra, who lied happily everyday of the year, agreed to this odd condition. So Mani began
working for him. He was a wonderful worker–hard-working and trustworthy. He was very
honest and soon became Rajendra’s right-hand man. Because of Mani’s hard work, Rajendra had
an excellent harvest. He and his wife, Manda, decided to have a big feast to celebrate. They
invited all their relatives and friends, from the village and outside as well. Everyone was looking
forward to the delicious feast being planned. On the morning of the feast, Rajendra decided he
would also give away some gifts to his relatives, just to show off. So he set off for the market in
his cart.

As soon as he was out of sight, Mani went running to Manda. He wept loudly and beat his chest.
Then he fell on the floor, sobbing and announced, “The master is dead ! The cart overturned on
the road. Our master has been flattened like a chapatti !” As soon as Rajendra’s wife and
relatives heard this, they started wailing. Mani rushed out, saying he would bring back the body,
while everyone started preparing for the last rites. Mani now went running to his master and said,
“Master ! Your wife is dead. A cobra bit her and she fell to the ground, as blue as the spring
sky.” Rajendra was stunned. What ! His beloved Manda, his partner in all his schemes, was
dead ! He hurried home shouting her name.

Manda too was weeping loudly, sitting in the courtyard. When she saw her husband run in, she
stopped mid-wail, and Rajendra too stoodopen mouthed. Then they fell into each other’s arms,
unable to believe their eyes.

As one, they turned to Mani. “What is the meaning of this, Mani ?” His master angrily
demanded. Mani smiled. “Remember my condition, that I would lie once in the year ? Well, I
chose today. You see what lies can do ? Now think what happens to the people whom you lie to
everyday of the year !” Saying this, he walked out, leaving behind a stunned and ashamed
landlord.

6. Why did Mani want to work for Rajendra ?


(A) Rajendra offered him food, clothing and shelter.
(B) To learn from Rajendra who was a successful businessman.
(C) To earn his trust so he could take over the business one day.
(D) He wanted to show Rajendra the error of his ways.
(E) Rajendra did not mind bad habit of telling lies.
Ans : (D)

7. Why did Rajendra shout at Mani one day ?


(1) Mani had lied to Rajendra and his wife causing them to fight.
(2) The feast had to be cancelled because of Mani and Rajendra suffered huge losses.
(3) He had embarrassed his wife and him in front of their friends and relatives.
(A) None
(B) Only (1)
(C) Only (1) and (2)
(D) Only (3)
(E) All (1), (2) and (3)
Ans : (D)

8. What was Rajendra’s immediate reaction on hearing the news of his wife’s ‘death’ ?
(A) He panicked because his wife was responsible for them being rich.
(B) He ran home at once because he doubted Mani’s honesty.
(C) He was very upset and rushed home at once.
(D) He wanted to make funeral arrangements.
(E) He went home to inform everyone that the feast was cancelled.
Ans : (C)

9. Why did Rajendra go out on the day of the feast ?


(A) He wanted to share his good fortune with his relatives.
(B) He took gifts for his relatives who were not as well off as he was.
(C) He wanted to boast about his prosperity to his relatives.
(D) To purchase groceries required for the feast from the market.
(E) He wanted to personally invite his relatives who lived outside the village.
Ans : (C)

10. Why did Mani work very hard for Rajendra ?


(A) He took pride in his work and wanted to ensure a good harvest.
(B) He was grateful to Rajendra for giving him a job.
(C) He knew Rajendra would share the profit with his right hand man.
(D) He considered himself as part to Rejendra’s family.
(E) None of these
Ans : (E)

11. Why did Rajendra accept Mani’s condition ?


(A) He did not value honesty.
(B) He had no intention of letting Mani fulfill it.
(C) He was an expert at detecting when people lied.
(D) He knew how difficult it was to stop lying.
(E) He predicted that Mani would be an excellent worker.
Ans : (D)

12. Why did Mani lie to Manda ?


(A) Out of revenge for not being invited to the feast.
(B) To find out whether Rajendra’s relatives cared about him or his wealth.
(C) To help her realise how much Rajendra meant to her.
(D) To delay the start of the feast as his master had not yet arrived.
(E) None of these
Ans : (B)
13. Which of the following is TRUE in the context of the passage ?
(A) Rajendra was ashamed only because Mani had managed to trick him.
(B) Mani had many friends who were cheated by Rajendra.
(C) Rajendra was a miser because despite being wealthy he travelled by cart.
(D) Mani was clever and a good actor.
(E) Manda was an excellent cook and had prepared a delicious feast.
Ans : (C)

14. Which of the following can be said about Manda ?


(1) She trusted Mani more than Rajendra.
(2) She had a good relationship with her husband’s relatives.
(3) She was dishonest.
(A) None
(B) Only (3)
(C) Only (1) and (3)
(D) Only (2) and (3)
(E) All (1), (2) and (3)
Ans : (A)

15. What opinion did Rajendra’s workers have of him ?


(A) He was generous for paying them and providing for their basic needs.
(B) He was a cheat and he exploited them.
(C) He was a weak man who listened only to his wife.
(D) He was unfair to his workers and paid each a different salary.
(E) None of these
Ans : (B)

Directions—(Q. 16–18) Choose the word or group of words which is MOST SIMILAR in
MEANING to the word printed in bold as used in the passage.

16. Pleasantly
(A) Extremely
(B) Delightfully
(C) Charming
(D) Friendly
(E) Coolly
Ans : (A)

17. Right
(A) Suitable
(B) Legally
(C) Accurately
(D) Straight
(E) Immediately
Ans : (D)
18. Wailing
(A) Crying
(B) Complaining
(C) Shouting
(D) Tears
(E) Grumbling
Ans : (A)

Directions—(Q. 19–20) Choose the word or group of words which is MOST OPPOSITE in
MEANING to the word printed in bold as used in the passage.

19. Celebrate
(A) Insult
(B) Reject
(C) Dishonour
(D) Mourn
(E) Infamous
Ans : (C)

20. Planned
(A) Organised
(B) Deliberate
(C) Cancelled
(D) Informal
(E) Automatic
Ans : (C)

Directions—(Q. 21–25) Rearrange the following six sentences 1, 2, 3, 4, 5 and 6 in the proper
sequence to form a meaningful paragraph; then answer the questions given below them.
1. Being generous by nature, the king offered them a meal and treated them kindly.
2. In appreciation the king ordered a stew to be prepared for the feast in honour of the farmer.
3. The king welcomed them too and placed before each a bowl of water saying. “This is the stew
of the stew of the hare !”
4. A few days later, a few people came to the court claiming to be neighbours of the farmer.
5. A poor farmer presented a fine hare to the king.
6. Not long afterwards a large number of people appeared at the palace and introduced
themselves as neighbours of the neighbours of the farmer.

21. Which of the following should be the LAST (SIXTH) sentence after rearrangement ?
(A) 2
(B) 3
(C) 4
(D) 5
(E) 6
Ans : (C)
22. Which of the following should be the THIRD sentence after rearrangement ?
(A) 2
(B) 3
(C) 4
(D) 5
(E) 6
Ans : (D)

23. Which of the following should be the FIRST sentence after rearrangement ?
(A) 1
(B) 2
(C) 3
(D) 4
(E) 5
Ans : (E)

24. Which of the following should be the SECOND sentence after rearrangement ?
(A) 1
(B) 2
(C) 3
(D) 4
(E) 6
Ans : (B)

25. Which of the following should be the FOURTH sentence after rearrangement ?
(A) 1
(B) 2
(C) 3
(D) 4
(E) 5
Ans : (A)

Directions—(Q. 26–30) Which of the phrases (A), (B), (C) and (D) given below should replace
the phrase given in bold in the following sentence to make it meaningful and grammatically
correct. If the sentence is correct as it is and No correction is required, mark (E) as the answer.

26. It is too early to say how the impact the new tax will have on investors.
(A) what impact
(B) that the impact
(C) how much impacts
(D) what are the impacts of
(E) No correction required
Ans : (A)

27. Unfortunately many of our towns and cities do have more good transportation systems.
(A) have a good
(B) not have good
(C) not have much good
(D) not having better
(E) No correction required
Ans : (B)

28. Banks charge differently rate of interest depending on the size of the loan.
(A) difference in rate of interests
(B) differently what rate of interest
(C) different rates of interest
(D) the different rate of interest
(E) No correction required
Ans : (C)

29. It is necessary that we take any steps to reduce pollution soon.


(A) we should take every
(B) we have taken no
(C) us to take any
(D) we take some
(E) No correction required
Ans : (D)

30. Kindly ask his advice regarding the various health insurance policies presently available.
(A) him to advise that
(B) regarding his advice
(C) that he should advise
(D) about his advice
(E) No correction required
Ans : (E)

Directions—(Q. 31–40) Read the sentence to find out whether there is any grammatical error or
idiomatic error in it. The error, if any, will be in one part of the sentence. The letter of that part
is the answer. If there is no error, the answer is (E). (Ignore errors of punctuation, if any.)

31. The scheme has been (A) / implemented only in these (B) / part of the country and the (C) /
Prime Minister will visit it shortly. (D) No error (E)
Ans : (B)

32. The company has (A) / decided to sell half (B) / of it shares (C) / to a Chinese firm. (D) No
error (E)
Ans : (C)

33. Rajiv has won a prize of (A) / two million dollars which (B) / has to be shared (C) / with all
his team members. (D) No error (E)
Ans : (D)
34. We have an account (A) / with this bank and (B) / find the staff much (C) / helpful and
knowledgeable. (D) No error (E)
Ans : (C)

35. By opening so many (A) / branches in such a (B) / short time, they have (C) / used the wrong
strategy. (D) No error (E)
Ans : (B)

36. I do not understand (A) / about how the payment (B) / was made without (C) / the manager’s
permission. (D) No error (E)
Ans : (B)

37. We have been (A) / telling them to (B) / drive carefully but (C) / they are never listening. (D)
No error (E)
Ans : (D)

38. During the interview (A) / with the General Manager (B) / I asked her what challenges (C) /
she was faced. (D) No error (E)
Ans : (D)

39. There is all sorts (A) / of regulations that have (B) / to be met before (C) / we get a licence.
(D) No error (E)
Ans : (A)

40. Today we are closer (A) / to reaching an agreement (B) / than we were (C) / a few months
ago. (D) No error (E)
Ans : (B)

Directions—(Q. 41–50) In the following passage there are blanks, each of which has been
numbered. These numbers are printed below the passage and against each, five words are
suggested, one of which fits the blank appropriately. Find out the appropriate word in each case.

For as long as I can remember I have thought of myself as a teacher and I still do even today. But
in 1965 I decided to …(41)… my hand at business. So I …(42)… my father to allow me to …
(43)… up a printing and packaging plant. I …(44)… the project proposal and applied for a …
(45)… Once it was approved I went …(46)… the challenges of employing the right …(47)…
My project was successful and I …(48)… an attractive profit. Becoming a rich businessman …
(49)… never my dream. It was just a …(50)… of proving to myself and my family that I could
be a success in buisness. This experience also gave me a lot of selfconfidence.

41. (A) attempt


(B) try
(C) give
(D) show
(E) use
Ans : (B)

42. (A) permitted


(B) appealed
(C) offered
(D) persuaded
(E) prevailed
Ans : (D)

43. (A) built


(B) raise
(C) stand
(D) begin
(E) set
Ans : (E)

44. (A) extended


(B) prepared
(C) thought
(D) formulate
(E) draft
Ans : (B)

45. (A) loan


(B) post
(C) leave
(D) job
(E) relief
Ans : (A)

46. (A) through


(B) ahead
(C) for
(D) away
(E) under
Ans : (A)

47. (A) equipment


(B) knowledge
(C) behaviour
(D) role
(E) people
Ans : (E)

48. (A) gain


(B) benefited
(C) obtain
(D) reap
(E) made
Ans : (E)

49. (A) fulfilled


(B) pursued
(C) was
(D) had
(E) stayed
Ans : (C)

50. (A) state


(B) certificate
(C) way
(D) proof
(E) direction
Ans : (B)

Punjab National Bank Clerk Exam., 2010


English Language : Solved Paper
(Held on 4-4-2010)
Directions—(Q. 1–15) Read the following passage carefully and answer the questions given
below it. Certain words have been printed in bold to help you locate them while answering some
of the questions.

Once upon a time, there lived an old lion. The lion, the king of the forest had grown old. He
became frail and due to this, he could not hunt for food. With each passing day he became more
and more weak. He realized that he would not live for long if it continued like that. He thought
how could he arrange for his food ? After pondering over it for quite some time, he decided that
he should have an assistant.

The lion thought that a fox would be the best person to handle this position as he was intelligent
and clever. He summoned the fox and said, “Dear friend, I have always liked you because you
are smart. I want to appoint you as my minister and advise me on all the affairs of the forest”.
The old lion also asked the fox, that since he was the king of the forest, he should not have to
hunt for his food. With respect to this, the fox’s first duty as minister was to bring him an animal
to eat every day. The fox could not refuse the king and accepted the offer.

After the conversation, the fox went out to find an animal for the lion. On the way, he met a fat
donkey. He said “My friend, I have got good news for you. You are very lucky. Our king, the
lion has chosen you to be his chief minister. He asked me to meet you and inform you about his
decision.” The donkey was scared of the lion and said, “I am afraid of the lion. He might kill me
and eat me up. Why has he chosen me as his chief minsiter ? I am not even fit enough to be a
minster as I am not as intelligent as other animals.” The clever fox laughed and, said, “Dear, you
don’t know your great qualities. Our king is dying to meet you. He has chosen you because you
are wise, gentle, and hard working. By serving the king, you will be the second most powerful
animal of our forest. Imagine, all the other animals will respect you and seek favours from you.”
“You must not lose your greatest chance in life.” So, the poor donkey was convinced and got
ready to go along with the fox.

In this way, the fox managed to attract the donkey to the lion’s den. When the fox and the
donkey approached, the lion was hungrier than ever. But he kept a smiling face and said,
“Welcome, my dear friend. Come near me. You are my chief minister.” As the donkey came
closer, the lion pounced on him and killed him instantly. The lion thanked the clever fox and
was happy to get the food. As the lion sat down to take his meal, the fox said, “Your Majesty, I
know you are very hungry but a king must take a bath before his meal”. The lion thought it was a
good idea and told the fox to keep a watch on the carcass of the donkey”. The fox silently sat
down to keep a watch of the donkey and thought to himslef, “I took all the trouble of getting the
donkey here. It is I who deserve the best portion of the meal”. Thus, the fox cut open the head of
the donkey and ate up the whole brain. When the lion returned he shouted, “What happened to
the donkey’s brain ? I wanted to eat the brain first.” The fox smilingly replied, “Your majesty,
donkeys have no brains. If he had any, he would not have come near a lion at all”.

1. Why did the lion decide to have an assistant for him ?


(A) He was too lazy to hunt for himself
(B) He was old and weak and could not hunt any more
(C) He wanted someone to help him kill the fat donkey
(D) He could not handle the affairs of the forest alone
(E) None of these

2. Why did the lion select the fox as his assistant ?


(A) He had heard that the fox had good hunting skills
(B) He wanted the fox to take over as the king of the forest
(C) The fox had offered to let the lion have his leftovers
(D) He had planned to eat the fox after luring him to become his assistant
(E) None of these

3. Why did the fox say ‘our king is dying to meet you’ to the donkey ?
(A) The king would have died of hunger if the donkey did not meet him
(B) The king desperately wanted the donkey to be his chief minister as he was gentle and hard
working
(C) The fox wanted to convince the donkey to come with him to the lion so that the lion could
eat him
(D) The king wanted to meet the doneky since all other animals respected the donkey more than
the king
(E) None of these

4. Which of the following best describes the donkey ?


(A) Honest
(B) Wicked
(C) Clever
(D) Opportunist
(E) Foolish

5. Which one of the phrases given below the following statement should be placed in the blank
space provided so as to make a meaningfully correct sentence in the context of the passage ?
When the lion did not see any brain in the donkey’s head ……….
(A) he spared his life and let him go
(B) he got upset with the fox for having selected such a donkey
(C) he took his decision to make him the chief minister back
(D) the fox explained to him that donekys do not have any brains
(E) None of these

6. What, according to the lion, was the fox’s primary duty as a minister ?
(A) To force all animals to respect their king and seek favours from him
(B) To convince the donkey to become his chief minister
(C) To take over as the king of the forest since the lion had grown too old
(D) To bring him an animal to eat every day
(E) None of these

7. What did the fox do when the lion went to take a bath before having his meal ?
(A) He secretly told the donkey to run away as the lion had planned to kill him
(B) He ate up the donkey’s brain as he had done all the hard work of bringing him to the lion’s
den
(C) He held himself responsible for the death of the poor donkey and did not let the lion eat the
donkey
(D) He killed the lion with the help of the donkey and became the king of the forest
(E) None of these

8. Which of the following is TRUE in context of the passage ?


(A) Finally, the fox got the best part of the meal
(B) The donkey was appointed as chief minister to the king
(C) The donkey which the fox had brought for the king did not have any brain in his head
(D) The donkey was very intelligent and clever
(E) None is true

9. What did the lion do when he saw the donkey in his den ?
(A) He was impressed by the donkey and made him his chief minister
(B) He ordered the fox to kill him and eat his brain
(C) He went to take a bath before meeting the donkey
(D) He immediately noticed that the donkey did not have any brain
(E) None of these

10. Which of the following is the moral of the story ?


(A) An idle brain is the devil’s workshop
(B) Fools are deaf to wise words
(C) Never believe an enemy’s sweet talks
(D) Morality can be best tested while one has power
(E) One can only lead a horse to water, but not make him drink it

Directions—(Q. 11–13) Choose the word which is most similar in meaning to the word printed
in bold as used in the passage.

11. POUNCED
(A) Climbed
(B) Grew
(C) Attacked
(D) Plunged
(E) Roared

12. FIT
(A) Healthy
(B) Deserving
(C) Strong
(D) Valuable
(E) Important

13. LOSE
(A) Misplace
(B) Suffer
(C) Dispose
(D) Defeat
(E) Miss

Directions—(Q. 14–15) Choose the word which is most opposite in meaning to the word
printed in bold as used in the passage.

14. HANDLE
(A) Mismanage
(B) Drop
(C) Confront
(D) Decline
(E) Uncover

15. FRAIL
(A) Unhealthy
(B) Massive
(C) Rich
(D) Robust
(E) Civilised
Directions—(Q. 16–25) Read each sentence to find out whether there is any grammatical error
or idiomatic error in it. The error, if any, will be in one part of the sentence. The letter of that part
is the answer. If there is ‘No error’, the answer is (E). (Ignore errors of punctuation, if any.)

16. When I called him yesterday, (A) / he offered to donate (B) / a handsome sum to (C) / the
flood relief fund. (D) No error (E)

17. The student which (A) / you had thought (B) / so highly of has (C) / failed to pass the
examination. (D) No error (E)

18. Government took strict action (A) / against the doctors on strike (B) / but they refused to
(C) / resume to work. (D) No error (E)

19. If a person has been given (A) / diplomatic immunity then he (B) / cannot be arrested on a
(C) / foreign land under any circumstance. (D) No error (E)

20. A group of birds (A) / migrate from southern part (B) / of the country to the (C) / Northern
part during summer. (D) No error (E)

21. The constable said that (A) / the prisoner seize a (B) / fully loaded gun from a policeman
(C) / and shot the prosecutor. (D) No error (E)

22. It will not be possible for you (A) / to catch the train on time (B) / because the nearest
railway station (C) / is at ten kilometres away. (D) No error (E)

23. Each of the survivors of the Tsunami (A) (B) / have been offered free (C) / psychological
consultation to ease their trauma, / by some of the top consultants. (D) No error (E)

24. Considering about her good credentials, (A) / the manager offered her a job (B) / in his
organization (C) / despite the lack of experience. (D) No error (E)

25. Although he has been (A) / winning the elections (B) / all years, this year his popularity (C) /
has substantially reduced. (D) No error (E)

Directions—(Q. 26–30) Rearrange the following six sentences (a), (b), (c), (d), (e) and (f) in the
proper sequence to form a meaningful paragraph; then answer the questions given below them.
(a) To their surprise, however, the reward went to a beggar who had contributed only a Rupee
instead of a wealthy donor.
(b) He received funds from many people as rich and poor donated generously to his trust.
(c) The man explained that the one Rupee given by the beggar was worth millions of Rupees as
that was all the money he possessed and that he had made a much greater sacrifice than others.
(d) During the function everyone waited with bated breath to hear who had made the maximum
contribution.
(e) A man went from town to town to collect money for his charitable trust.
(f) On returning he decided to hold a function and reward the person whose contribution had
been maximum.

26. Which of the following should be the SECOND sentence after rearrangement ?
(A) b
(B) c
(C) d
(D) e
(E) f

27. Which of the following should be the LAST (SIXTH) sentence after rearrangement ?
(A) a
(B) c
(C) d
(D) e
(E) f

28. Which of the following should be the FOURTH sentence after rearrangement ?
(A) b
(B) c
(C) d
(D) e
(E) f

29. Which of the following should be the THIRD sentence after rearrangement ?
(A) a
(B) b
(C) c
(D) e
(E) f

30. Which of the following should be the FIRST sentence after rearrangement ?
(A) a
(B) b
(C) d
(D) e
(E) f

Directions—(Q. 31–35) In each of the following questions six words are given which are
denoted by (a), (b), (c), (d), (e) and (f). By using all the six words, each only once, you have to
frame a meaningful and grammatically correct sentence. The correct order of words is the
answer. Choose from the five alternatives, the one having the correct order of words and mark it
as your answer on the answer-sheet.

31. (a) OTHERS


(b) HER
(c) SHE
(d) MISTAKES
(e) FOR
(f) BLAMES
(A) cfaebd
(B) acfebd
(C) bdacfe
(D) cfdabe
(E) bdafce

32. (a) FINALISE


(b) WE
(c) DETAILS
(d) LATER
(e) THE
(f) WILL
(A) BFEACD
(B) BDCFEA
(C) BDCAEF
(D) ECFDBA
(E) BFAECD

33. (a) UV-RAYS


(b) ABSORBS
(c) THE
(d) LAYER
(e) HARMFUL
(f) OZONE
(A) ACBEFD
(B) CFDBEA
(C) ACBFED
(D) CFBDAE
(E) CBEAFD

34. (a) PRACTISING


(b) SPEECH
(c) SPENT
(d) HIS
(e) HE
(f) HOURS
(A) DBCFEA
(B) DBFCAE
(C) EBCDAF
(D) FACBED
(E) ECFADB

35. (a) TO
(b) AROUND
(c) SEE
(d) THEY
(e) HIM
(f) HATED
(A) DFAECB
(B) DBFAEC
(C) DFCAEB
(D) DFACEB
(E) CEBFAD

Directions—(Q. 36–40) In each question below a sentence with four words printed in bold type
is given. These are lettered as (A), (B), (C) and (D). One of these four words printed in bold may
be either wrongly spelt or inappropriate in the context of the sentence. Find out the word, if
any, which is wrongly spelt or inappropriate. The letter of that word is your answer. If all the
words printed in bold are correctly spelt and also appropriate in the context of the sentence, mark
(E), i.e., ‘All correct’ as your answer.

36. A large number of celebrities (A) have joined an NGO involved (B) in the movement for
protection (C) of animal writes. (D) All correct (E)

37. The teacher liked the poem (A) so much that she requested (B) Saba to read it allowed (C)
to the whole (D) class. All correct (E)

38. Breathe (A) deeply and inhale the sents (B) of Roses and Daisies in the landscaped gardens
(C) surrounding (D) my house. All correct (E)

39. As the wind blue (A) harder every minute, (B) people got a fairly (C) good idea that a storm
was approaching (D) the town. All correct (E)

40. The family had to bear (A) a leaky sealing (B) throughout the rainy season (C) as they could
not afford (D) to get it repaired. All correct (E)

Directions—(Q. 41–50) In the following passage there are blanks, each of which has been
numbered. These numbers are printed below the passage and against each, five words are
suggested, one of which fits the blank appropriately. Find out the appropriate word in each case.

Once upon a time, there lived a sparrow on a banyan tree. She laid her eggs in the nest. One
afternoon, a wild elephant came under the tree and in a fit of rage, broke a branch of the tree on
which the nest was …(41)…. Unfortunately, all the eggs of the sparrow …(42)… after falling
down though the sparrow was saved. The sparrow was full of grief and began weeping for her
eggs.

A woodpecker, a close friend of the sparrow, heard her crying and asked her, “Why are you
crying, my friend ?” The sparrow said, “The …(43)… elephant has killed my offspring. If you
are a true friend of mine, suggest a way-to kill him”. The woodpecker …(44)… her and told her
that he knew a fly and she would definitely help them kill the elephant.

Both of them went to seek the help of the fly. The woodpecker said, “A wild elephant has
crushed my friend’s eggs. We need your help in killing him.” the fly replied, “One of my friends
is a frog. Let us go to him and take his help too”. They went to the frog and …(45)… the whole
incident. The frog said, “What can an elephant do before a united crowd like us ? Do what I tell
you. Dear Fly, you go to the elephant and hum a sweet tune into his ears. When he closes his
eyes in delight, the woodpecker will poke his eyes. This way, he will become …(46)…. When he
gets thirsty, he will …(47)… for water. I will go to a marshy land and begin croaking there. …
(48)… that there is water, the elephant will come there. He will sink into the marshy area and …
(49)….

The next day in the noon, all of them played out the plan and the elephant was killed, as he
drowned into a marshy area after being blinded by the woodpecker, when he closed his eyes in
response to the music. Thus, the smartness of all the animals enabled the sparrow in taking her
…(50)… on the elephant.

41. (A) Born


(B) Built
(C) Grown
(D) Broken
(E) Found

42. (A) Fell


(B) Escaped
(C) Damaged
(D) Survived
(E) Broke

43. (A) Defective


(B) Attacking
(C) Wicked
(D) Poor
(E) Harmless

44. (A) Ignored


(B) Protected
(C) Scolded
(D) Consoled
(E) Defended

45. (A) Said


(B) Informed
(C) Revived
(D) Mention
(E) Narrated
46. (A) Blind
(B) Unseen
(C) Regretful
(D) Sorry
(E) Dead

47. (A) Need


(B) Want
(C) Search
(D) Drink
(E) Demands

48. (A) Recalling


(B) Informed
(C) Sure
(D) Assuming
(E) Accepting

49. (A) Hurt


(B) Die
(C) Fall
(D) Kill
(E) Realise

50. (A) Help


(B) Revenge
(C) Anger
(D) Insult
(E) Fight

Answers :
1. (B) 2. (E) 3. (A) 4. (E) 5. (D) 6. (D) 7. (B) 8. (A) 9. (E) 10. (C)
11. (C) 12. (B) 13. (E) 14. (A) 15. (D) 16. (C) 17. (A) 18. (D) 19. (D) 20. (B)
21. (B) 22. (D) 23. (B) 24. (D) 25. (B) 26. (A) 27. (B) 28. (C) 29. (E) 30. (D)
31. (A) 32. (E) 33. (B) 34. (E) 35. (D) 36. (D) 37. (C) 38. (B) 39. (A) 40. (B)
41. (B) 42. (E) 43. (C) 44. (D) 45. (E) 46. (A) 47. (C) 48. (D) 49. (B) 50. (B)

Allahabad Bank Probationary Officers Exam., 2010


(Held on 21-2-2010)
Computer Knowledge : Solved Paper
1. A………typically connects personal computers within a very limited geographical area,
usually within a single building.
(A) LAN
(B) BAN
(C) TAN
(D) NAN
(E) None of these
Ans : (A)

2. Computers manipulate data in many ways, and this manipulation is called—


(A) utilizing
(B) batching
(C) upgrading
(D) processing
(E) None of these
Ans : (D)

3. An e-mail address typically consists of a user ID followed by the ……… sign and the name of
the e-mail server that manages the user’s electronic post office box.
(A) @
(B) #
(C) &
(D) *
(E) None of these
Ans : (A)

4. Software applies……, also called algorithms, to process data.


(A) arithmetic
(B) procedures
(C) objects
(D) rules
(E) None of these
Ans : (A)

5. A file extension is separated from the main file name with a(n) ……, but no spaces.
(A) question mark
(B) exclamation mark
(C) underscore
(D) period
(E) None of these
Ans : (E)

6. An ad hoc query is a—
(A) pre-planned question
(B) pre-scheduled question
(C) spur-of-the-moment question
(D) question that will not return any results
(E) None of these
Ans : (B)

7. A Web ……… consists of one or more Web pages located on a Web server.
(A) hub
(B) site
(C) story
(D) template
(E) None of these
Ans : (B)

8. A computer ……… is a set of program instructions that can attach itself to a file, reproduce
itself, and spread to other files.
(A) worm
(B) virus
(C) trojan horse
(D) phishing scam
(E) None of these
Ans : (B)

9. The desktop contains small graphics called—


(A) windows
(B) logos
(C) icons
(D) pictures
(E) None of these
Ans : (C)

10. C, BASIC, COBOL, and Java are examples of ……… languages.


(A) low-level
(B) computer
(C) system programming
(D) high-level
(E) None of these
Ans : (D)

11. A(n)………camera is a peripheral device used to capture still images in a digital format that
can be easily transferred into a computer and manipulated using graphics software.
(A) digital
(B) analog
(C) classic
(D) film
(E) None of these
Ans : (A)

12. ……makes it possible for shoppers to make purchases using their computers.
(A) E-world
(B) E-commerce
(C) E-spend
(D) E-business
(E) None of these
Ans : (D)

13. Networks are monitored by security personnel and supervised by ………who set(s) up
accounts and passwords for authorized network users.
(A) IT managers
(B) the government
(C) network administrators
(D) password administrators
(E) None of these
Ans : (C)

14. Application software is designed to accomplish—


(A) real-world tasks
(B) computer-centric tasks
(C) gaming tasks
(D) operating system tasks
(E) None of these
Ans : (D)

15. A telephone number, a birth date, and a customer name are all examples of—
(A) a record
(B) data
(C) a file
(D) a database
(E) None of these
Ans : (D)

16. The human-readable version of a program is called—


(A) source code
(B) program code
(C) human code
(D) system code
(E) None of these
Ans : (D)

17. A ……… computer (also referred to as a laptop), is a small, lightweight personal computer
that incorporates the screen, the keyboard, storage, and processing components into a single
portable unit.
(A) notebook
(B) journal
(C) diary
(D) briefcase
(E) None of these
Ans : (A)

18. ……… is the result produced by a computer.


(A) Data
(B) Memory
(C) Output
(D) Input
(E) None of these
Ans : (C)

19. Programs such as Internet Explorer that serve as navigable windows into the Web are called

(A) Hypertext
(B) Networks
(C) Internet
(D) Web browsers
(E) None of these
Ans : (D)

20. A ……… is a device that not only provides surge protection, but also furnishes your
computer with battery backup power during a power outage.
(A) surge strip
(B) USB
(C) UPS
(D) battery strip
(E) None of these
Ans : (C)

21. When you save to ………, your data will remain intact even when the computer is turned off.
(A) RAM
(B) motherboard
(C) secondary storage device
(D) primary storage device
(E) None of these
Ans : (C)

22. The motherboard is the—


(A) circuit board that houses peripheral devices
(B) same as the CPU chip
(C) the first chip that is accessed when the computer is turned on
(D) circuit board that contains a CPU and other chips
(E) None of these
Ans : (D)

23. A computer-intensive problemruns on a—


(A) server
(B) main frame
(C) supercomputer
(D) super PC
(E) None of these
Ans : (C)

24. A(n)……converts and executesone statement at a time.


(A) compiler
(B) interpreter
(C) converter
(D) instructions
(E) None of these
Ans : (B)

25. The term ……… designates equipment that might be added to a computer system to enhance
its functionality.
(A) digital device
(B) system add-on
(C) disk pack
(D) peripheral device
(E) None of these
Ans : (D)

26. Approximately how many bytes make one megabyte ?


(A) One thousand
(B) Ten thousand
(C) One hundred
(D) One million
(E) None of these
Ans : (D)

27. What is MP3 ?


(A) A mouse
(B) A Printer
(C) A Sound format
(D) A Scanner
(E) None of these
Ans : (C)

28. All the deleted files go to—


(A) Recycle Bin
(B) Task Bar
(C) Tool Bar
(D) My Computer
(E) None of these
Ans : (A)

29. ……… this is the act of copying or downloading a program from a network and making
multiple copies of it.
(A) Network piracy
(B) Plagiarism
(C) Software piracy
(D) Site-license piracy
(E) None of these
Ans : (C)

30. A directory within a directory is called—


(A) Mini Directory
(B) Junior Directory
(C) Part Directory
(D) Sub Directory
(E) None of these
Ans : (D)

31. Which is the best definition of a software package ?


(A) An add-on for your computer such as additional memory
(B) A set of computer programs used for a certain function such as word processing
(C) A protection you can buy for a computer
(D) The box, manual and license agreement that accompany commercial software
(E) None of these
Ans : (B)

32. In MICR, C stands for ……….


(A) Code
(B) Colour
(C) Computer
(D) Character
(E) None of these
Ans : (D)

33. Fax machines and imaging systems are examples of—


(A) bar-code readers
(B) imaging systems
(C) scanning devices
(D) pen-based systems
(E) None of these
Ans : (B)

34. When writing a document, you can use the ……… feature to find an appropriate word or an
alternative word if you find yourself stuck for the right word.
(A) dictionary
(B) word finder
(C) encyclopedia
(D) thesaurus
(E) None of these
Ans : (A)

35. Which key is used in combination with another key to perform a specific task ?
(A) Function
(B) Control
(C) Arrow
(D) Space bar
(E) None of these
Ans : (B)

36. A disk’s content that is recorded at the time of manufacture and that cannot be changed or
erased by the user is—
(A) read-only
(B) memory-only
(C) run-only
(D) write-only
(E) None of these
Ans : (A)

37. In an information system, alphanumeric data normally takes the form of—
(A) Sentences and paragraphs
(B) Numbers and alphabetical characters
(C) Graphic shapes and figures
(D) Human voice and other sounds
(E) None of these
Ans : (B)

38. A Website’s main page is called its—


(A) home page
(B) browser page
(C) search place
(D) bookmark
(E) None of these
Ans : (A)

39. When installing ………, the user must copy and usually decompress program files from a
CDROM or other medium to the hard disk.
(A) programming software
(B) system hardware
(C) applications hardware
(D) applications software
(E) None of these
Ans : (A)

40. A collection of interrelated files in a computer is a—


(A) file manager
(B) field
(C) record
(D) database
(E) None of these
Ans : (D)

41. A ……… computer is a large and expensive computer capable of simultaneously processing
data for hundreds or thousands of users.
(A) server
(B) mainframe
(C) desktop
(D) tablet
(E) None of these
Ans : (B)

42. The trend in computer systems is toward the use of graphical user interfaces (GUIs). In these
operating systems, a trackball is described as—
(A) a roller ball which moves the cursor
(B) a pen-shaped device which allows data to be entered through the CRT screen
(C) a figure which resembles a familiar office device
(D) an outdated input device
(E) None of these
Ans : (A)

43. Various applications and documents are represented on the Windows desktop by—
(A) Symbols
(B) Labels
(C) Graphs
(D) Icons
(E) None of these
Ans : (D)

44. What is usually used for displaying information at public places ?


(A) Monitors
(B) Overhead Projections
(C) Monitors and Overhead Projections
(D) Touch Screen Kiosks
(E) None of these
Ans : (D)

45. The real business and competitive value of information technology lies in—
(A) The software applications that are used by many companies
(B) The capabilities of the software and value of the information a business acquires and uses
(C) The infrastructure of hardware, networks, and other IT facilities that are commonly used by
many companies
(D) The capabilities of the hardware and the speed at which it processes information
(E) None of these
Ans : (B)

46. Companies use which of the following vendors to provide access to software and services
rather than purchasing the applications and maintaining the applications themselves ?
(A) Open source vendors
(B) Alliances
(C) Application service providers
(D) All of the above
(E) None of these
Ans : (C)

47. Which one of the following would be considered as a way that a computer virus can enter a
computer system ?
(A) Opening an application previously installed on the computer
(B) Borrowed copies of software
(C) Viewing a website without causing any additional transactions
(D) Running antivirus programs
(E) None of these
Ans : (B)

48. Collecting personal information and effectively posing as another individual is known as the
crime of—
(A) spooling
(B) dentity theft
(C) spoofing
(D) hacking
(E) None of these
Ans : (B)

49. The first step in the transaction processing cycle is—


(A) database operations
(B) audit
(C) data entry
(D) user inquiry
(E) None of these
Ans : (C)

50. In the information systems concept, the output function involves—


(A) Capturing and assembling elements that enter the system to be processed
(B) Transformation processes that convert input into output
(C) Transferring elements that have been produced by a transformation process to their ultimate
destination
(D) Monitoring and evaluating feedback to determine whether a system is moving toward the
achievement of its goal
(E) None of these
Ans : (A)

IDBI Bank Executive Exam., 2009


Reasoning : Solved Paper
(Held on 13-12-2009)

1. If ‘M’ denotes ‘+’, ‘N’ denotes ‘÷’, ‘R’ denotes ‘–’ and Q denotes ‘x’ then—
15 M 12 Q 5 R 40 N 8 = ?
(A) 70
(B) 130
(C) 45
(D) 60
(E) None of these
Ans : (A)

2. How many meaningful English words can be formed using the third, fifth, ninth and eleventh
letters of the word ‘COMMANDMENT’ using each letter only once ?
(A) None
(B) One
(C) Two
(D) Three
(E) More than three
Ans : (E)

3. Seema correctly remembers that she took leave after 21st October and before 27th October.
Her colleague Rita took leave on 23rd October but Seema was present on that day. If 24th
October was a public holiday and 26th October was Sunday, on which day in October did Seema
take leave ?
(A) 22nd October
(B) 25th October
(C) 22nd or 25th October
(D) Data Inadequate
(E) None of these
Ans : (C)

Directions—(Q. 4 and 5) The following questions are based on the five three letter words given
below—
PUN SAD CRY FOE STY
If each of the words is rearranged in alphabetical order then—
4. If it is possible to make only one meaningful English word using each letter only once, from
the first letters of each of the five words after the letters of each word are rearranged in
alphabetical order, the last letter of that word is your answer. If more than one word can be
formed, ‘X’ is the answer and if no such word can be formed then ‘W’ is the answer.
(A) E
(B) N
(C) X
(D) W
(E) S
Ans : (C)

5. How many words will remain unchanged even after their rearrangement in alphabetical
order ?
(A) None
(B) One
(C) Two
(D) Three
(E) More than three
Ans : (C)

6. How many pairs of letters are there in the word ‘VERIFIED’ each of which has as many
letters between them in the word as in the English alphabet ?
(A) None
(B) One
(C) Two
(D) Three
(E) More than three
Ans : (C)

7. The fare paying capacity of people who travel on routes connecting to small towns is very
low. Most successful airlines which operate in such regions have a large number of seats.
Which of the following can be inferred from the above information ?
(A) Regional airlines are quite profitable.
(B) People from cities are increasingly travelling to small towns.
(C) Regional airlines have to charge low fares in order to be profitable.
(D) The number of people travelling from small towns to cities is massive.
(E) None of these
Ans : (C)

8. If ‘A ×D’ means ‘A is the sister of D’, ‘A + D’ means ‘D is the daughter of A’ and ‘A ÷ D’


means ‘A is the mother of D’, then how will N is the aunt of M be denoted ?
(A) M + L ×N
(B) M ÷ L + N
(C) L ×N ÷ M
(D) N ×L ÷ M
(E) None of these
Ans : (D)

9. M earns more than X and less than T. V earns more than M and T. R earns more than only X.
Who earns the least among the five of them ?
(A) X
(B) V
(C) M
(D) Cannot be determined
(E) None of these
Ans : (A)

10. The Court has stayed proceedings against the stockbroker. He can now return to the country.
Which of the following can be assumed from the given information ?
(A) The stockbroker is innocent.
(B) The police cannot arrest the stockbroker.
(C) The judge has been bribed.
(D) Complaints filed against the stockbroker have been withdrawn.
(E) None of these
Ans : (B)

Directions—(Q. 11–15) Read the following passage carefully and answer the questions given
below it.

A, B, C, D, E, F, G and H are sitting around a circular table, facing the centre. A sits third to the
left of C and second to the right of E. B sits second to the right of D who is not an immediate
neighbour of E. H sits second to the left of F. G is not an immediate neighbour of D.

11. Which of the following pairs has only one person sitting-between them, if the counting is
done in clockwise direction ?
(A) F, G
(B) H, G
(C) H, C
(D) H, B
(E) None of these
Ans : (B)

12. Who sits third to the right of E ?


(A) D
(B) G
(C) F
(D) B
(E) None of these
Ans : (C)

13. What is the position of G with respect to A’s position ?


(A) Immediately to the right
(B) Second to the left
(C) Third to the right
(D) Third to the left
(E) Fourth to the right
Ans : (D)

14. Who sits between E and A ?


(A) F
(B) D
(C) G
(D) B
(E) None of these
Ans : (E)

15. Starting from A’s position, if all the eight are arranged in alphabetical order in clockwise
direction, the seating position of how many members (excluding A) would remain unchanged ?
(A) None
(B) One
(C) Two
(D) Three
(E) Four
Ans : (A)

Directions—(Q. 16–23) In each of the questions below are given four statements followed by
four conclusions numbered I, II, III and IV. You have to take the given statements to be true
even if they seem to be at variance with commonly known facts. Read all the conclusions and
then decide which of the given conclusions logically follows from the given statements
disregarding commonly known facts.

16. Statements :
Some stoves are ovens.
All ovens are cylinders.
Some engines are stoves.
Some metals are cylinders.
Conclusions :
I. Some ovens are metals
II. Some cylinders are stoves
III. Some ovens are engines
IV. No engine is a cylinder
(A) None follows
(B) Only II and IV follow
(C) Only II and III follow
(D) Only III follows
(E) Only II follows
Ans : (E)
17. Statements :
Some cars are buses.
Some buses are trains.
All airplanes are trains.
All trucks are buses.
Conclusions :
I. Some airplanes are trucks.
II. Some cars are trains.
III. Some trucks are airplanes.
IV. No truck is train.
(A) None follows
(B) Only II and IV follow
(C) Only III and IV follow
(D) Only I and III follow
(E) None of these
Ans : (A)

18. Statements :
Some tools are hammers.
All tools are trees.
Some trees are flowers.
No hammer is flower.
Conclusions :
I. All hammers are tools.
II. No tool is flower.
III. Some hammers are trees.
IV. Some flowers are tools.
(A) None follows
(B) Only I and either II or IV follow
(C) Only II and IV follow
(D) Only III follows
(E) None of these
Ans : (E)

19. Statements :
Some bags are pockets.
Some pockets are trousers.
All skirts are pockets.
Some belts are bags.
Conclusions :
I. Some trousers are belts.
II. Some skirts are bags.
III. No trouser is belt.
IV. Some skirts are trousers.
(A) All follow
(B) Only II and IV follows
(C) Only III follows
(D) Only either I or III follows
(E) None of these
Ans : (D)

20. Statements :
Some cats are tigers.
All lions are cats.
Some horses are lions.
All horses are animals.
Conclusions :
I. Some lions are tigers.
II. No horse is tiger.
III. Some horses are cats.
IV. Some horses are tigers.
(A) None follows
(B) Only III follows
(C) Only II and IV follow
(D) Only III and either II or IV follow
(E) None of these
Ans : (D)

21. Statements :
Some clocks are radios.
No radio is laptop.
Some fridges are clocks.
Some clocks are laptops.
Conclusions :
I. Some fridges are laptops.
II. No radio is fridge.
III. Laptops are either fridges or clocks.
IV. No laptop is fridge.
(A) Only II follows
(B) Only IV follows
(C) Only either I or IV follows
(D) Only III follows
(E) None of these
Ans : (C)

22. Statements :
Some notes are coins.
Some papers are plastics.
All coins are papers.
No note is cardboard.
Conclusions :
I. Some notes are papers.
II. Some coins are plastics.
III. No paper is cardboard.
IV. No note is plastic.
(A) Only I follows
(B) Only I and II follow
(C) Only I and III follow
(D) All follow
(E) None of these
Ans : (A)

23. Statements :
All stools are desks.
No desk is shelf.
All shelves are cupboards.
Some cupboards are mirrors.
Conclusions :
I. No mirror is shelf.
II. No stool is shelf.
III. No cupboard is desk.
IV. No mirror is desk.
(A) None follows
(B) Only II and III follow
(C) Only II follows
(D) Only II, III and IV follow
(E) None of these
Ans : (C)

Directions—(Q. 24–28) Study the following arrangement carefully and answer the questions
given below—
M?2DB7A4*96$T+N5@VEW#U8F©3

24. How many such prime numbers are there in the given arrangement each of which is
immediately followed by a symbol and preceded by a consonant ?
(A) None
(B) One
(C) Two
(D) Three
(E) More than three
Ans : (B)

25. If the places of the symbols are interchanged with those of the numbers immediately
following them in the given arrangement then which of the following will be the eleventh from
the right end ?
(A) V
(B) @
(C) 5
(D) N
(E) None of these
Ans : (C)

26. Which of the following is the seventh letter to the left of # ?


(A) A
(B) N
(C) T
(D) B
(E) None of these
Ans : (D)

27. What should come in place of the question mark (?) in the following series based on the
above arrangement ?
27B469T5N?
(A) @ W E
(B) 5 E V
(C) V U #
(D) V # W
(E) None of these
Ans : (D)

28. If the symbols and numbers are dropped from the given arrangement and then the letters are
rearranged in alphabetical order which of the following will be the sixth from the left end ?
(A) M
(B) F
(C) N
(D) E
(E) None of these
Ans : (A)

Directions—(Q. 29–33) Below in each question are given two statements (I) and (II). These
statements may be either independent causes or may be effects of independent causes or a
common cause. One of these statements may be the effect of the other statement. Read both the
statements and then decide which of the
following answer choice correctly depicts the relationship between these two statements. Mark
answer—
(A) If statement I is the cause and statement II is its effect.
(B) If statement II is the cause and statement I is its effect.
(C) if both statements I and II are independent causes.
(D) if both statements I and II are effects of independent causes.
(E) if both statements I and II are effects of some common causes.

29. I. There has been mass recruitment of IT professionals by Indian IT companies


II. Many developed countries are increasingly outsourcing IT related functions to India and
China.
Ans : (B)

30. I. Many farmers have given up jute cultivation as it is no longer economically viable.
II. The textile ministry has proposed a hike in the Minimum Support Price of jute.
Ans : (A)

31. I. The government is considering changes in the Land Acquisition Act.


II. Several large infrastructure development projects have been stalled due to unavailability of
land.
Ans : (E)

32. I. The Government is considering the possibility of involving private sector companies in
highway construction projects.
II. The implementation of many highway projects undertaken by government agencies is behind
schedule in various states.
Ans : (B)

33. I. The price of aircraft fuel has risen during the past few months.
II. Many passenger airlines in India have been forced to cut their air fares by about 10 per cent.
Ans : (D)

Directions—(Q. 34–40) In each question below is given a group of letters followed by four
combinations of digits/symbols lettered (A), (B), (C) and (D). You have to find out which of the
combinations correctly represents the group of letters based on the coding system and mark the
letter of that combination as your answer. If none of the combinations correctly represents the
group of letters, mark (E) i.e., ‘None of these’ as your answer.
Letter :
WRAPGBMUSEFTND
Digit/Symbol Codes :
$8!27#9@?5b4*6
Conditions :
I. If the middle letter is a vowel, the codes for the first and the fourth letters are to be
interchanged.
II. If the first two letters are consonants, the first letter is to be coded, no code may be given to
the second letter and the remaining three letters are to be coded.
III. If the first letter is a vowel and the last letter is a consonant both are to be coded as the code
for the consonant.

34. NEAST—
(A) ?5!*4
(B) ?5!4*
(C) 4*!5?
(D) 45!?
(E) None of these
Ans : (A)
35. DAEWP—
(A) 6!5$2
(B) $!562
(C) 6!52$
(D) $265!
(E) None of these
Ans : (B)

36. PEFTG—
(A) 25b42
(B) 2b4G
(C) 25b47
(D) 47b25
(E) None of these
Ans : (C)

37. SMGBE—
(A) ?975#
(B) 97#5?
(C) ?97#5
(D) ?#79?
(E) None of these
Ans : (E)

38. UGREN—
(A) *785@
(B) *785*
(C) @785@
(D) @85*
(E) None of these
Ans : (B)

39. RBMFN—
(A) 8#9b4
(B) b*98#
(C) 89b*
(D) 8#9b*
(E) None of these
Ans : (C)

40. MUTWA—
(A) 9@$4!
(B) 94$!
(C) 94@!$
(D) 9@4$!
(E) None of these
Ans : (D)

Directions-(Q. 41-50) Study the following information carefully and answer the questions which
follow—

A bank is disbursing educational loans to meritorious students. Loans will be made available to
applicants who possess the following criteria.
The candidate must—
I. Be a citizen of India
II. Have secured admission to a post graduate course (Masters or P. G. Diploma) offered by an
Indian or foreign university in India.
III. Have secured 60 per cent marks in the entrance examination for the course.
IV. Have security (property/LIC policy) equivalent to the loan amount.
However if the applicant fulfils the above mentioned criteria except—
(1) At (II) above the case may be referred to the Loan committee.
(2) At (IV) above guarantee of a third party who has an account with the bank may be obtained.
Mark answer :
(A) If the loan is to be sanctioned.
(B) If the loan is to be rejected.
(C) If the loan is to be referred to the loan committee.
(D) If guarantee from an account holder is needed.
(E) If the data provided is inadequate to take a decision.

41. Shruti has secured admission to MNV University in Chennai for a post-graduate degree in
management. She obtained 65 per cent in the entrance test for the course. She has an Indian
passport.
Ans : (E)

42. Milind has applied and secured admission for a post-graduate degree in advertising at MIC
University Ahmedabad. He has an LIC policy equivalent to the loan amount. He secured 70 per
cent in his graduation and is an Indian citizen.
Ans : (A)

43. Prakash has applied for a postgraduate course in IT and Animation which will commence
from January in Kolkata. He has secured admission to the course having obtained 60 per cent in
the written entrance test.
His uncle who has an account with the bank is willing to stand guarantor.
Ans : (E)

44. Anil is an Indian science graduate who has secured admission for a Masters degree in
Computer Science from a prestigious Indian University. He stood first in the entrance test with
82 per cent marks. His family is willing to use their house, which is equivalent in value to the
loan amount, as security. He will stay with his uncle in Dubai during the course.
Ans : (A)

45. Nikhil’s part time P.G. Diploma in Management will begin in December. He secured 65 per
cent in the All India entrance exam and has secured admission to a college in Mumbai. He has
requested a transfer for the
duration of the course. He has an LIC policy equivalent to the loan amount. Nikhil holds an
Indian passport.
Ans : (A)

46. Maria secured 75% in the entrance exam to NVT College Pune and has obtained admission.
An Indian national she has the necessary property to offer as security for the loan amount. The
duration of her undergraduate degree course in Computer Applications is 3 years.
Ans : (E)

47. Deepa obtained 70 per cent in her engineering entrance exam for her Masters and has secured
admission to the college of her choice. She has an LIC policy equivalent in value to the loan
amount. Her interview to obtain a visa for her course in the U.S. is on November 25. She is an
Indian citizen.
Ans : (A)

48. Sudhir has applied for a postgraduate degree in Pharmacy at All India Institute at New Delhi.
He is an Indian citizen and having secured 60 per cent in his entrance exam, he has been granted
admission. He does not have any property to use as security. His father who has an account with
the bank is willing to guarantee the loan.
Ans : (D)

49. Anandi has secured admission for a P.G. Diploma in Management at a University in
Allahabad with 72 per cent in the entrance examination. She has an LIC Policy amounting to the
loan amount. She is an Indian citizen.
Ans : (A)

50. Ravi has obtained admission to the Indian Institute of Research at Chandigarh for a Masters
in Mathematics offered only to Indian citizens. He secured 68 per cent in the entrance test. He
does not have any security against the loan. However his employer who has an account with the
bank is willing to guarantee the loan.
Ans : (D)

Oriental Bank of Commerce Probationary Officers Exam., 2009


(Held on 13-12-2009)
Reasoning : Solved Paper
1. In a certain code ‘PLANT’ is written as ‘$@2*©’ and ‘YIELD’ is written as ‘β64@%’. How
is ‘DELAY’ written in that code ?
(A) β4*2%
(B) β4@2%
(C) %42@β
(D) %4@2β
(E) None of these
Ans : (D)

2. How many meaningful English words can be formed with the letters ARILT using each letter
only once in that word ?
(A) None
(B) One
(C) Two
(D) Three
(E) More than three
Ans : (C)

3. D said, “A’s father is the only brother of my sister’s son.” How is A’s father related to D ?
(A) Cousin
(B) Nephew
(C) Aunt
(D) Data Inadequate
(E) None of these
Ans : (B)

Directions—(Q. 4–6) The following questions are based on the five three digit numbers given
below—
394 632 783 576 895

4. If the positions of the first and the second digit within each number are interchanged, which of
the following will be the second highest number ?
(A) 632
(B) 783
(C) 576
(D) 895
(E) 394
Ans : (E)

5. If the first and second digits of each of the numbers are added the resulting sum of which of
the following numbers will not be exactly divisible by 3 ?
(A) 895
(B) 394
(C) 576
(D) 632
(E) 783
Ans : (A)

6. If 2 is added to the last digit of each number and then the positions of the first and the third
digits are interchanged, which of the following will be the highest number ?
(A) 576
(B) 895
(C) 783
(D) 394
(E) 632
Ans : (A)

7. If ‘A’ is coded as 1, ‘B’ as 3, ‘C’ as 5 and so on, which of the following is the numerical value
of the word ‘FAZED’ ?
(A) 81
(B) 79
(C) 77
(D) 80
(E) None of these
Ans : (B)

8. Which of the following pairs of words have the same relationship as FAN : HEAT ?
(A) Water : Drink
(B) Light : Night
(C) Teach : Student
(D) Air : Breathe
(E) Food : Hunger
Ans : (B)

9. Q types faster than R but not as fast as V.T types faster than R S types faster than V. Who
amongst the five of them types the fastest ?
(A) V
(B) T
(C) S
(D) Data Inadequate
(E) None of these
Ans : (D)

10. If ‘B ×C’ means ‘B, is the daughter of C’, ‘B + C’ means ‘B is the husband of C’ and ‘B – C’
means ‘B is the sister of C’, then what does ‘M + N – P ×Q’ mean ?
(A) M is the brother-in-law of Q
(B) M is the uncle of Q
(C) M is the son-in-law of Q
(D) M is the mother-in-law of Q
(E) None of these
Ans : (C)

Directions—(Q. 11–17) Read the following passage carefully and answers the questions given
below it—
A group of seven friends, A, B, C, D, E, F and G work as Economist, Agriculture Officer, IT
Officer, Terminal Operator, Clerk, Forex Officer and Research Analyst, for Banks L, M, N; P, Q,
R and S but not necessarily in the same order. C works for Bank N and is neither a Research
Analyst nor a Clerk. E is an IT Officer and works for Bank R. A works as Forex Officer and does
not work for Bank L or Q. The one who is an Agriculture Officer works for Bank M. The one
who works for Bank L works as a Terminal Operator. F works for Bank Q. G works for Bank P
as a Research Analyst. D is not an Agriculture Officer.

11. Who amongst the following works as an Agriculture Officer ?


(A) C
(B) B
(C) F
(D) D
(E) None of these
Ans : (B)

12. What is the profession of C ?


(A) Terminal operator
(B) Agriculture Officer
(C) Economist
(D) Cannot be determined
(E) None of these
Ans : (C)

13. For which Bank does B work ?


(A) M
(B) S
(C) L
(D) Either M or S
(E) None of these
Ans : (A)

14. What is the profession of the person who works for Bank S ?
(A) Clerk
(B) Agriculture Officer
(C) Terminal Operator
(D) Forex Officer
(E) None of these
Ans : (D)

15. For which Bank does D work ?


(A) Q
(B) L
(C) N
(D) S
(E) None of these
Ans : (B)

16. Who amongst the following works as a Clerk ?


(A) C
(B) B
(C) F
(D) D
(E) None of these
Ans : (C)

17. Which of the following combinations of person, profession and bank is correct ?
(A) A-Forex Officer-M
(B) D-Clerk-L
(C) F-Agriculture Officer-Q
(D) B-Agriculture Officer-S
(E) None of these
Ans : (E)

Directions—(Q. 18–23) In each of the questions below are given four statements followed by
four conclusions numbered I, II, III and IV. You have to take the given statements to be true
even if they seem to be at variance with commonly known facts. Read all the conclusions and
then decide which of the given conclusions logically follows from the given statements
disregarding commonly known facts.

18. Statements :
Some stones are rocks.
All rocks are boulders.
Some boulders are mountains.
All hills are mountains.
Conclusions :
I. Some stones are mountains.
II. Some hills are boulders.
III. Some boulders are stones.
IV. All mountains are hills.
(A) None follows
(B) Only II follows
(C) Only II and III follow
(D) Only III follows
(E) None of these
Ans : (D)

19. Statements :
All arrows are bows.
All bows are swords.
Some swords are daggers.
All daggers are knives.
Conclusions :
I. All knives are bows.
II. Some swords are knives.
III. All bows are arrows.
IV. All arrows are swords.
(A) Only II follows
(B) Only II and IV follow
(C) Only III and IV follow
(D) Only I and III follow
(E) None of these
Ans : (B)

20. Statements :
Some pianos are violins.
Some violins are drums.
All drums are guitars.
No guitar is a flute.
Conclusions :
I. Some guitar are pianos.
II. Some drums are flutes.
III. Some pianos are drums.
IV. No flute is a drum.
(A) None follows
(B) Only I follows
(C) Only either II or IV follows
(D) Only IV follows
(E) None of these
Ans : (D)

21. Statements :
Some airplane are helicopters.
All helicopters are gliders.
All gliders are kites.
All kites are balloons.
Conclusions :
I. Some helicopters are balloons.
II. All kites are airplanes.
III. All balloons are gliders.
IV. All helicopters are kites.
(A) Only IV follows
(B) Only either II or III follow
(C) Only III follows
(D) Only I follows
(E) None of these
Ans : (A)

22. Statements :
All kings are warriors.
All soldiers are warriors.
All sentries are warriors.
Some sentries are soldiers.
Conclusions :
I. Some sentries are kings.
II. All warriors are soldiers.
III. Some warriors are sentries.
IV. Some soldiers are kings.
(A) None follows
(B) Only I follows
(C) Only II follows
(D) Only II and III follow
(E) None of these
Ans : (E)

23. Statements :
All plastics are glasses.
Some glasses are wood.
Some woods are metals.
All metals are cloths.
Conclusions :
I. Some cloths are wood.
II. Some woods are both cloths and glasses.
III. Some glasses are plastics.
IV. Some cloths are metals.
(A) None follows
(B) All follow
(C) Only II and IV follow
(D) Only I and II follow
(E) None of these
Ans : (B)

Directions—(Q. 24–25) Read the information given below carefully and then answer the
questions that follow. In a certain code language :
I. ‘she likes apples’ is written as ‘pic sip dip’.
II. ‘parrot likes apples lots’ is written as ‘dip pic tif nit’.
III. ‘she likes parrots’ is written as ‘tif sip dip’.

24. How is ‘parrot’ written in that code language ?


(A) pic
(B) dip
(C) tif
(D) Cannot be determined
(E) None of these
Ans : (C)

25. Which of the following statements is not necessary in order to answer the above question ?
(A) I
(B) II
(C) III
(D) All are required
(E) Either II or III
Ans : (B)

Directions—(Q. 26–30) Study the following arrangement carefully and answer the questions
given below :
SMP63*$A4LN@Z5#VU7E8B%TYXG2I&9W

26. Which of the following is the third to the left of the fifteenth from the left end ?
(A) @
(B) 5
(C) 7
(D) U
(E) None of these
Ans : (A)

27. How many such consonants are there in the above arrangement each of which is immediately
followed by an even number and preceded by a symbol ?
(A) None
(B) One
(C) Two
(D) Three
(E) More than three
Ans : (A)

28. What should come in place of the question-mark (?) in the following series based on the
above arrangement ?
PM3A$LZ@#?
(A) U V E
(B) V # E
(C) 7 U 8
(D) V 7 #
(E) None of these
Ans : (C)

29. Four of the following five are alike in a certain way based on their position in the above
arrangement and hence form a group.
Which is the one that does not belong to that group ?
(A) A 4 *
(B) T B Y
(C) 9 I W
(D) V 5 U
(E) P S 6
Ans : (A)
30. How many such numbers are there in the above arrangement each of which is immediately
followed by a vowel and immediately preceded by a consonant ?
(A) None
(B) One
(C) Two
(D) Three
(E) More than three
Ans : (B)

Directions—(Q. 31–35) Below in each question are given two statements I and II. These
statements may be either independent causes or may be effects of independent causes or a
common cause. One of these statements may be the effect of the other statement. Read both the
statements and then decide which of the following answer choice correctly depicts the
relationship between these two statements. Mark answer—
(A) if statement I is the cause and statement II is its effect.
(B) if statement II is the cause and statement I is its effect.
(C) if both statement I and II are independent causes.
(D) if both statements I and II are effects of independent causes.
(E) if both statement I and II are effects of some common cause.

31. I. The government has amended tax laws to boost exports.


II. The export sector has been passing through difficult times due to heavy tax burdens.
Ans : (B)

32. I. Budgetary allocation for building a better railway network e.g. constructing new railway
lines has increased.
II. There has been a substantial drop in the number of passenger opting for air travel.
Ans : (D)

33. I. The production of pulses has dropped for the third consecutive year.
II. India has decided to import pulses this year.
Ans : (A)

34. I. The Bank has provided a link on its website to obtain feed-back from customers.
II. Customers have been complaining about poor services in the Bank’s branches.
Ans : (B)

35. I. Indian citizens are willing to incur the cost of using environment friendly technology.
II. Many countries are taking steps to cut their carbon emissions.
Ans : (E)

Directions—(Q. 36–40) Below is given a passange followed by several possible inferences


which can be drawn from the facts stated in the passage. You have to examine each inference
separately in the context of
the passage and decide upon the degree of its truth or falsity. Mark answer—
(A) if the inference is ‘definitely true’ i.e. it properly follows from the statements of facts given.
(B) if the inference is ‘probably true’ though not ‘definitely true’ in the light of the facts given.
(C) if the ‘data are inadequate’ i.e. from the facts given you cannot say whether the data is likely
to be true or false.
(D) if the inference is ‘probably false’ though not ‘definitely false’ in the light of the facts given.
(E) if the inference is ‘definitely false’ i.e. it cannot be drawn from the given facts or contradicts
the given facts.
The pace of recruitment has unmistakably picked up in the last few months. After two abysmal
quarters the business sentiment has turned positive. A flurry of hiring activity has been observed
across the manufacturing sector and the IT and ITES sector catering to telecom and legal process
outsourcing. The advent of new players and an aggressive expansion of telecom giants has seen a
bullish outlook for the industry with a promise of exponential growth. But is the worst over for
businesses across the spectrum ? Firms are going back to the drawing board and dusting off their
plans to see if there is scope for atleast incremental hiring in specific domains where they want to
build expertise. Besides that there are few signs to show that there is a broad based revival in the
market.

36. Companies in various sectors have projected higher business growth in the near future.
Ans : (B)

37. The telecom sector has experienced negative growth in the past few years.
Ans : (D)

38. Many companies in the manufacturing sector have decided to reduce their fresh intake of
employees.
Ans : (C)

39. Persons with legal qualifications and expertise will earn more in the future.
Ans : (C)

40. Most companies across sectors have decided to increase their manpower requirement
manifold.
Ans : (A)

Directions—(Q. 41–45) In each question below is given a group of letters followed by four
combinations of digits/symbols lettered (A), (B), (C) and (D). You have to find out which of the
combinations correctly represents the group of letters based on the coding system and mark the
letter of that combination as your answer. If none of the combinations correctly represents the
group of letters, mark (E) i.e. ‘None of these’ as
your answer.
Letter :
TAGRSDNEVUXP
Digit/Symbol Code :
@ 4 9 # % 3 ©5 1 β7 6
Conditions—
(i) if the first letter is a consonant and the last letter is a vowel only the first letter may be coded
and no code
may be given to the last letter.
(ii) if both the first and last letters are vowels then both are to be coded as $.
(iii) if the first letter is a vowel and the last letter is a consonant then the codes are to be
interchanged.

41. DESGRT
(A) 35%9#
(B) 35%9#@
(C) @5%9#3
(D) 359%#@
(E) None of these
Ans : (B)

42. NERSPU
(A) @5#%6@
(B) @5#56©
(C) ©5#%6
(D) ©5#%6β
(E) None of these
Ans : (C)

43. XNTPGA
(A) 7©@694
(B) 7©@964
(C) 4©@69
(D) 7©@69
(E) None of these
Ans : (D)

44. ESVNTU
(A) $%1©@$
(B) 5%1©@β
(C) $%1©@β
(D) $%1©β 5
(E) None of these
Ans : (A)

45. UXPTGN
(A) ©76@9β
(B) $76@9©
(C) β 76@9©
(D) β 76@9B
(E) None of these
Ans : (A)
Directions—(Q. 46–50) In each question below is given a statement followed by two courses of
action numbered I and II. A course of action is a step or administrative decision to be taken for
improvement, follow-up or further action in regard to the problem, policy etc. On the basis of the
information given in the statement, you have to assume everything in the statement to be true and
then decide which of the suggested courses of action logically follows for pursuing. Given
answer—
(A) if only Course of action I follows.
(B) if only Course of action II follows.
(C) if either Course of action I or Course of action II follows.
(D) if neither Course of action I nor Course of action II follows.
(E) if both courses of action I and II follow.

46. Statement : An increasing number of graduates produced by Indian universities are


unemployable.
Courses of action :
I. Colleges and Institutes of higher learning should be given greater autonomy to decide course
content.
II. World class foreign universities should be encouraged to set up campuses in India.
Ans : (A)

47. Statement : Heavy rains hit the state during October, just before the State Assembly
elections and caused heavy damage to standing crops in most parts of the state.
Courses of action :
I. Elections should be postponed to give candidates the opportunity to campaign.
II. The Government should announce a relief package for those who are affected.
Ans : (B)

48. Statement : Cell phone users have found that tariff plans are not as attractive as promoted by
telecome companies and complained to regulatory authority about the same.
Courses of action :
I. The regulatory authority should direct telecome companies to be transparent on the tariff
structure of all plans.
II. The government should restrict the number of telecome companies operating in the country.
Ans : (A)

49. Statement : A number of school children in the local schools have fallen ill after the
consumption of their subsidised tiffin provided by the school authority.
Courses of action :
I. The tiffin facility of all schools should be discontinued with immediate effect.
II. The government should implement a system to certify the quality of tiffin provided by the
school.
Ans : (B)

50. Statement : The dolphin population in India has been decreasing sharply over the past few
years.
Courses of action :
I. Dolphins should be declared an endangered species and be bred in aquariums or protected
areas.
II. Locals should be enlisted to protect dolphins.
Ans : (E)

Directions—(Q. 51–60) Study the following information carefully and answer the questions
which follow :
A Research Institute is recruiting a libration to digitise its information resources among other
duties. Candidates must possess the following criteria. The candidate must :
I. be not less than 35 years and not exceed 40 years as on 01- 11-2009.
II. have a Bachelor’s Degree in Library and Information Science with 65 per cent marks.
III. have a Ph.D. in Library Science.
IV. have post qualification experience of at least 4 years in a University Library.
However if the candidate fulfils the above mentioned criteria except—
(a) at (II) above but the candidates has a UGC NET certification with all the other above criteria
fulfilled he/she may be referred to the Dean.
(b) at (IV) above but all the eligibility criteria are met and the candidate has at least one year’s
experience in a research institute he/ she may be offered contractual appointment for a year.
Based on the above criteria study carefully whether the following candidates are eligible for the
recruitment process and mark your answer as follows. You are not to assume anything other than
the information provided in each question. All cases are given to you as on 1-11-2009.
Mark answer :
(A) if he/she is to be shortlisted.
(B) if he/she is not to be shortlisted.
(C) if he/she should be referred to the Dean.
(D) if he/she may be offered contractual appointment if required.
(E) if the data provided is inadequate to take a decision.

51. Anil Rath has a doctorate in Library Science from Karnataka University in 2003. Born on
July 21, 1969, he graduated in Library and Information Science from Karnataka University
where he was Assistant Librarian for four years since 2005.
Ans : (B)

52. Dr. Samir Bali has a Ph.D. in Library Science and has been with the Institute of Fundamental
Research as Assistant Librarian since October 2008. He graduated with a degree in Library and
Information Science in 1994 at the age of 22. He obtained 70 per cent in his graduation.
Ans : (D)

53. Vaishali Shetty has a librarian at STS University since 2007 when she qualified in the UGC
NET examination. She has a degree in Library and Information Science with 60 per cent. Her
first job was as junior librarian at TRP Institute of Development Research from October 2000 to
December 2001.
Ans : (B)

54. Vivek Jha has a Ph.D. in Library and Information Science. He graduated in Library and
Information Science in 1992 with 65 per cent. He was born on 1-10- 1974. Since July 2005, he
has been working as Deputy Librarian at a deemed University.
Ans : (A)

55. A graduate in Library Science with 69 per cent Dr. M. Puri has been working at Ranchi
University for 4 years as Deputy Librarian. She earned her doctorate while working for the
Labour Research Institute for 5 years as Assistant Librarian. She is 38 years old as on the
required date.
Ans : (A)

56. Megha Vaidya has a graduate degree in Library Science from Punjab University where she
has been a librarian for the past 5 years. In 2002 she obtained her UGC NET qualification at the
age of 29. She obtained 72 per cent graduation.
Ans : (E)

57. Anup Gupta is obtaining his Ph.D. from YCM University where he has been Junior Librarian
since 2004. He qualified in the UGC NET exam. in June, 2000. He has a degree in Library and
Information Science with 62 per cent. His date of birth is 17-10-1973.
Ans : (B)

58. Kirti Shukla obtained her doctorate and Bachelor’s degree from Patna University. She
obtained 63 per cent at graduation. She obtained her UGC NET qualification in 1998 when she
was 26.
Ans : (E)

59. Prakash Sinha has been a librarian for a government institution for three years. Prior to this
he was a University Librarian for 7 years after completing his Ph.D. in Library Science. He
graduated in 1991 with 68 per cent in Library Science. He is exactly 40 years of age on the
specified date.
Ans : (A)

60. Rohan Sachdev obtained his UGC NET qualification after his graduation in 1998 when he
began working. He has been Assistant Librarian for the past two years with a research instiute.
He obtained 65 per cent in his graduation in Library Science. He earned his Ph.D. in Library
Science in 2007. His date of birth is 22-10-1974.
Ans : (D)

Corporation Bank Probationary Officers Exam., 2009


(Held on 22-11-2009)
General Awareness on Socio-Economic Developments : Solved Paper

1. Which of the following is/are the highlights of the Union Budget 2009-10 ? The Finance
Minister promised
1. To bring fiscal deficit under control, institutional reforms will be initiated during the year.
2. Banks and insurance companies will remain in public sector and government will provide full
support to these institutions to remain competitive by capital infusion, wherever necessary.
3. Banks will be allowed to set up offsite ATMs without prior approval from the RBI.
(A) Only 1
(B) Only 2
(C) Only 3
(D) Only 1 and 3
(E) All 1, 2 and 3
Ans : (E)

2. As per the reports published in various newspapers/magazines and journals, the year 2008 was
an important milestone in the evaluation of human settlements. For the first time in the human
history, the population living in urban areas exceeded that in rural areas. What is India’s position
in this regard ?
1. India is a relatively less urbanized country with only 30% of its total population living in
cities.
2. The size of its urban population is the second largest in the world.
3. India has more than 100 biggest cities in the world.
(A) Only 1
(B) Only 2
(C) Only 1 and 2
(D) All 1, 2 and 3
(E) None of these
Ans : (E)

3. According to Dr. Mohammad Yunus, the Nobel prize winner Economist, a particular type of
financing undertaken by the banks, has created a miraculous situation where every year about 5
per cent clients of the Grameen Banks come out of their poverty and join the main stream of the
socio economic development in poor/developing countries. This financing activity of the banks
in known as—
(A) Trade Finance
(B) Micro Credit
(C) Export finance
(D) All of these
(E) None of these
Ans : (B)

4. As per the reports published by the International Monetary Fund (IMF), the developing
economies across the world are holding a massive stockpile of foreign exchange reserves. Which
of the following is/are the resultant consequences of this type of stockpilings by various nations ?
1. A trade imbalance (vast difference in value of imports/exports) is generated between the
developing and emerging economies and the rich countries. It is not a healthy sign.
2. Many economists are of the opinion that these imbalances created the underpinning of the
current global economic crisis.
3. This stockpiling is responsible not only for problems in trade balances but has also created a
problem of money laundering.
(A) Only 1
(B) Only 2
(C) Only 3
(D) Only 1 and 2
(E) All 1, 2 and 3
Ans : (E)

5. Which of the following is/are true about the Railway Budget 2009-10 ?
1. Railway tickets to be made available through post offices and mobile vans.
2. Long distance trains will have on-board doctor and infotainment services.
3. Tatkal scheme of booking is modified. Instead of booking tickets under the scheme five days
in advance, it can now be done only two days in advance.
(A) Only 1
(B) Only 2
(C) Only 3
(D) All 1, 2 and 3
(E) None of these
Ans : (D)

6. The Govt. of India has set up a special group to devise a long term pricing formula, so that it
can cope up with the prevailing global volatility in the prices of which of the following
commodities ?
(A) Sugar
(B) Paddy
(C) Crude oil and Petroleum
(D) Gold
(E) Pulses
Ans : (C)

7. Which of the following is not included in the core sectors of Indian economy ?
(A) Transport
(B) Coal
(C) Steel
(D) Cement
(E) Electricity
Ans : (A)

8. As we all know India has a threetier structure of tax administration. There are taxes levied by
Central Govt., State Govts. and Local Govts. Which of the following taxes is levied by the Local
Government Bodies ?
(A) Value Added Tax
(B) Stamp Duty
(C) Service Tax
(D) Land Revenue
(E) Tax on water supply and drainage etc.
Ans : (E)

9. What was the amount sanctioned by the Govt. of India during 2008-09 for providing drinking
water to rural parts of India ?
(A) Rs. 30,000 crore
(B) Rs. 40,000 crore
(C) Rs. 53,000 crore
(D) Rs. 60,000 crore
(E) Rs. 73,000 crore
Ans : (E)

10. The Reserve Bank of India has reconstituted the Technical Advisory Committee (TAC) on
monetary policy with effect from 1st July 2009. Who is the Chairperson of the same ?
(A) D. Subbarao
(B) K. C. Chakrabarty
(C) Shyamala Gopinath
(D) Usha Thorat
(E) None of these
Ans : (E)

11. Which of the following statements about the ‘State of Indian Economy, during 2008-09 is/are
true ?
1. The growth of real GDP during 2008-09 was recorded at 6•7%, compared to 9% during 2007-
08.
2. The fiscal deficit was the greatest cause of concern as it exceeded the budget estimates and
stood at 13%.
3. Forex reserves were at a very comfortable position as India got a lot of FDIs and FIIs during
the year. However, 60% of the reserves were deployed to manage oil and petroleum imports and
subsidies on all such products.
(A) Only 1
(B) Only 2
(C) Only 3
(D) All 1, 2 and 3
(E) None of these
Ans : (A)

12. Many times we see a term ‘PPP in financial newspapers. One full form of it is ‘Private Public
Partnership’. What is another version of the same also used in the financial world ?
(A) Perfect, Preferred and Protected
(B) Point to Point Protocol
(C) Power Point Presentation
(D) Purchasing Power Parity
(E) None of these
Ans : (B)

13. As per the announcement made recently, the Govt. of India is going to launch a National
Mission for which of the following in near future ?
(A) National Mission for Female Literacy
(B) National Mission for Abolishing Child Labour
(C) National Mission for Computer Literacy
(D) National Mission for protecting the Rights of Physically Challenged
(E) None of these
Ans : (A)

14. Which of the following is one of the most successful programmes under Bharat Nirman ?
(A) Pradhan Mantri Gram Sadak Yojana
(B) Watershed Development Scheme
(C) Rajiv Gandhi Grameen Vidyutikaran Yojana
(D) Indira Awas Yojana
(E) None of these
Ans : (D)

15. As we all know ‘Mobile Banking’ is comparatively a new mode of banking all over the
world, what is its status in India ?
1. The willingness to conduct banking through mobile devices is higher in India.
2. About 4% of total m -commerce transactions have taken place in the first few months of the
current fiscal year in India.
3. Since RBI’s permission is needed for offering mobile banking, only two private banks are
providing this service at present.
(A) Only 1
(B) Only 2
(C) Only 3
(D) All 1, 2 and 3
(E) None of these
Ans : (C)

16. Which of the following is/are true about the Pradhan Mantri Adarsh Gram Yojana (PMAGY)
to be launched soon ?
1. The scheme will be launched in one village of each district in U.P., Bihar and Rajasthan as a
pilot project.
2. The scheme is only for villages where the 50 per cent population is of Scheduled Castes.
3. An amount of Rs. 1,00,000 crore is provided for the scheme out of which 50% will come as
grant-in-aid from the UNO.
(A) Only 1
(B) Only 2
(C) Only 3
(D) All 1, 2 and 3
(E) None of these
Ans : (B)

17. As per the recent announcement made by the Govt. of India, a special One Time Grant of Rs.
100 crore is given to which of the following organizations ?
(A) Administrative Staff College of India (Hyderabad)
(B) Tata Institute of Social Sciences (Mumbai)
(C) International Institute for Population Sciences (Mumbai)
(D) Indian Institute of Technology (Delhi)
(E) Indian Council of Forestry Research and Education (Dehradun)
Ans : (E)

18. Many times we hear about the ‘Banking Ombudsman’. What is/are the major roles of same ?
1. Banking Ombudsman is a quasi judicial authority having powers to summon, the banks and
the customers both in case of dispute.
2. The Banking Ombudsman are appointed by the President of India and are retired judges of the
High or Supreme Courts.
3. The decisions/judgements given by the Banking Ombudsman cannot be taken to any court of
justice as their decisions are last in case of bank related problems/disputes.
(A) Only 1
(B) Only 2
(C) Only 3
(D) All 1, 2 and 3
(E) None of these
Ans : (A)

19. Who amongst the following is the Union Home Minister at present ?
(A) Kapil Sibbal
(B) Sharad Pawar
(C) P. Chidambaram
(D) Murli Deora
(E) None of these
Ans : (C)

20. Which of the following is NOT prescribed/recommended by the Economic Survey 2008-09
presented to the Lok Sabha recently ?
(A) Allow 49% FDI in defence and insurance sectors
(B) Decontrol diesel and petroleum prices
(C) Increase exemption limit on personal income tax upto Rs. 5,00,000
(D) List all unlisted PSUs and sell a minimum 10% equity to public
(E) Decontrol sugar and fertilizer
Ans : (C)

21. The Reliance Power (R-Power) is working on four Hydropower projects in Arunachal
Pradesh which are ‘Run-of-River’ (R-O-R) type of projects. What is the main characteristic of
ROR projects ?
(A) These projects are developed only in hilly areas. They can’t be developed on plains
(B) In these projects there is no need of storing the water in dams which is very costly
(C) In ROR projects the only requirement is to make a big reservoir and turn the flow of river in
the direction in which we need it
(D) ROR projects are three to four times costlier as compared to old patterned dam type of
projects
(E) None of these
Ans : (B)
22. Which of the following countries is planning to launch its first Mars probe in near future ?
(A) China
(B) Japan
(C) India
(D) Russia
(E) USA
Ans : (A)

23. India recently inducted Airborne Warning And Control System (AWACS) platforms in its
Airforce. These sophisticated machines are supplied by which of the following countries ?
(A) Australia
(B) Germany
(C) Israel
(D) France
(E) Russia
Ans : (E)

24. Dr. Manmohan Singh is the member of the Parliament from which of the following
constituencies ?
(A) New Delhi (South)
(B) Amritsar
(C) Lucknow
(D) Medak
(E) None of these
Ans : (E)

25. Which of the following pairs of countries are the signatories of a Gas pipeline Deal
christened as Peace Pipeline ?
(A) Iran – Iraq
(B) Iraq – UAE
(C) Kuwait – Iran
(D) Pakistan – Iran
(E) Pakistan – India
Ans : (D)

26. Which of the following countries is at the top of the ‘World Green Index’ compiled by the
America’s ‘National Geographic Society’ and also through an international polling from ‘Global
Scan’ ?
(A) South Africa
(B) USA
(C) China
(D) India
(E) None of these
Ans : (D)
27. As we all know G-20 has set up a working group to find out the reasons of the recent
economic crisis and global slowdown.
What according to the reports of the working group was/were the main reasons of the crisis and
slowdown ?
1. Allowing cheap iquidity and sub-prime lending
2. Poor credit appraisals and securitisation
3. Excessive dependence on credit ratings
(A) Only 1
(B) Only 2
(C) Only 3
(D) Only 1 and 3
(E) All 1, 2 and 3
Ans : (E)

28. Which of the oil rich gulf countries elected its first women parliamentarians when four
women were declared elected in its recently held elections ?
(A) Iran
(B) UAE
(C) Kuwait
(D) Iraq
(E) None of these
Ans : (C)

29. ‘Mardeka Cup’ is associated with game of—


(A) Hockey
(B) Golf
(C) Football
(D) Badminton
(E) Lawn Tennis
Ans : (C)

30. Which of the following state govts. decided to have a ‘Mahila Court’ in every district of the
state ?
(A) Tamil Nadu
(B) Andhra Pradesh
(C) Karnataka
(D) Delhi
(E) Uttar Pradesh
Ans : (B)

31. Who amongst the following is the author of the book ‘Faith and Compassion’ ?
(A) Arun Gandhi
(B) Kuldeep Nayyar
(C) Amit Chaudhury
(D) Navin Chawla
(E) None of these
Ans : (D)

32. Which of the following countries has developed ‘sajjil-2’ missiles, which were test fired
recently ?
(A) Pakistan
(B) Iran
(C) North Korea
(D) India
(E) None of these
Ans : (B)

33. The first Republic Day of the Federal Democratic Republic of Nepal was celebrated on
which of the following days ?
(A) 16th August 2009
(B) 29th June 2009
(C) 16th July 2009
(D) 29th May 2009
(E) 20th May 2009
Ans : (E)

34. Pankaj Advani has become the world’s professional champion player of—
(A) Badminton
(B) Tennis
(C) Golf
(D) Chess
(E) None of these
Ans : (E)

35. We hear a term ‘Carbon Foot Print’ these days very frequently. What does it indicate ?
1. The amount of carbondioxide released in the atmosphere ever day by an electronic
gadget/organization or even by an individual.
2. The quantity of CO2 released when one ton of air recycled by an Air conditioner, during every
hour of its functioning.
3. The quantity of all green house gases put together required to maintain the temperature of the
environment at a fixed level C at nodal points in a country.°of 30
(A) Only 1
(B) Only 2
(C) Only 3
(D) All 1, 2 and 3
(E) None of these
Ans : (D)

36. The letter ‘D’ stands for which of the following in the abbreviated name OECD ?
Organisation of Economic Cooperation and……
(A) Defence
(B) Direction
(C) Dominance
(D) Development
(E) Devaluation
Ans : (D)

37. The ‘8th Annual Asian Security Summit’ was organized in which of the following places in
May 2009 ?
(A) Seoul
(B) Yangon
(C) Singapore
(D) Kathmandu
(E) New Delhi
Ans : (C)

38. Which of the following countries is the host of FIFA World Cup 2010 ?
(A) Italy
(B) France
(C) Brazil
(D) Germany
(E) South Africa
Ans : (E)

39. Late Y. S. Rajasekhar Reddy was from which of the following political parties ?
(A) Telugu Desham
(B) Bharatiya Janta Party
(C) DMK
(D) Samajwadi Party
(E) None of these
Ans : (E)

40. Who amongst the following won Women’s singles Title of ‘Spanish Open Badminton-2009’
the final for which was played at Madrid in May 2009 ?
(A) Sayali Gokhale
(B) Lianne Tan
(C) Nathalie Descamps
(D) Jill Pittard
(E) None of these
Ans : (E)

41. Which of the following is the Bank Rate at present ?


(A) 3%
(B) 5%
(C) 6%
(D) 7%
(E) None of these
Ans : (C)
42. P. Mangesh Chandran whose name was in news recently is a famous —
(A) Chess player
(B) Playback singer
(C) Cine actor
(D) Author
(E) Journalist
Ans : (A)

43. Which of the following teams won the Champions League Football 2009, the final of which
was played in Rome in May 2009 ?
(A) Manchester United
(B) Barcelona
(C) Everton
(D) Chelsea
(E) None of these
Ans : (B)

44. Which of the following states is holding a unique summit on Tigers at global level ?
(A) West Bengal
(B) Andhra Pradesh
(C) Tamil Nadu
(D) Rajasthan
(E) Chhattisgarh
Ans : (D)

45. Which of the following books is NOT written by Daris Lessing ?


(A) The Golden Note book
(B) Memoirs of a survivor
(C) The city of joy
(D) The summer before the Dark
(E) The fifth child
Ans : (C)

46. Which of the following is Statutory Liquidity Ratio at present in India ?


(A) 16%
(B) 11%
(C) 6%
(D) 33%
(E) None of these
Ans : (E)

47. Mr. Deep Joshi’s name was in news as he has been awarded—
(A) Pulitzer Prize
(B) Raman Magsaysay Award
(C) Man Booker Prize
(D) Jamunalal Bajaj Award
(E) None of these
Ans : (B)

48. Bihar Sugarcane Amendment Bill (Regulation of Supply and purchase) 2007 will enable
State Govt. to—
(A) Purchase sugarcane from the farmers and supply it to sugar factories directly at a
predetermined price. This will save farmers from exploitation by the factories who do not pay
them adequately and in time.
(B) Produce ethanol directly from sugarcane
(C) Force farmers to sell their entire production of sugarcane to sugar mills in cooperative sector
only and not to private mills who are paying exorbitantly high price to the farmers. Cooperative
mills are not getting enough supply to keep them alive
(D) Compel farmers to produce sugarcane in at last 30% of their land as Bihar is losing its major
share in sugarcane production due to unwillingness of farmers to grow sugarcane. They wish to
switch over to other high cost crops which give them good returns
(E) None of these
Ans : (B)

49. Ms. Nirupama Rao has taken over as the new—


(A) Home Secretary of India
(B) Foreign Secretary of India
(C) Cabinet Secretary of India
(D) Attorney General of India
(E) None of these
Ans : (B)

50. Who amongst the following is adjudged as the CEO of the year 2009 by the ‘Global Supply
Chain Leaders’ ?
(A) Mr. Lalit Modi
(B) Mr. Yash Birla
(C) Mr. Kumar Mangalam Birla
(D) Ms. Indra Nooyi
(E) Mr. Ratan Tata
Ans : (D)

Malwa Gramin Bank Clerk Exam., 2010


(Held on 31-1-2010)
English Language : Solved Paper

Directions—(Q. 1–15) Read the following passage carefully and answer the questions given
below it. Certain words/phrases have been printed in bold to help you locate them while
answering some of the questions.
Once upon a time, there was a royal elephant which used to reside in the premises of the king’s
palace. The elephant was very dear to the king, so he was well-fed and well treated. There was
also a Dog who lived near the Elephant’s shed. He was very weak and skinny. He was always
fascinated by the smell of rich sweet rice being fed to the royal elephant. One day, the Dog could
no longer resist the aroma of the rice and somehow managed to sneak into the Elephant's shed.
He ate the grains of sweet rice that fell from the Elephant’s mouth. He liked the rice so much,
that he started going there daily to eat the rice. For days, the huge Elephant didn’t notice the
small dog as he was busy enjoying the delicious food. Gradually, the Dog grew bigger and
stronger eating such rich food. Finally the Elephant noticed him and allowed him access to the
food. The Elephant enjoyed the company of the Dog and started sharing his food with him. They
also started spending time with each other and soon became good friends. They ate together,
slept together and played together. While playing, the Elephant would hold the Dog in his trunk
and swing him back and forth. Soon neither of them was happy without the other. They became
great friends and didn’t want to be separated from each other.

Then one day, a man saw the Dog and asked the Elephant-keeper, “I want to buy this Dog. What
price do you want for it ?” The Elephantkeeper didn’t own the Dog but sold it and extracted a
sum of money from this deal. The man took the Dog to his home village, which was quite far
away. The King’s Elephant became very sad after this incident. He missed his friend a lot and
started neglecting everything. He didn’t want to do anything without his dear friend, so he
stopped eating, drinking and even bathing. Finally, the Elephant-keeper reported this to the King;
however he didn’t mention anything about the Dog. The King had a wise minister, who was
known for his keen understanding of animals. The King ordered the minister, “Go to the
Elephant shed and find out the reason for the Elephant’s condition”. The intelligent minister
went to the Elephant shed and found the Elephant very sad. He examined the Elephant and asked
the Elephantkeeper, “There is nothing wrong with this Elephant’s body, then why does he look
so sad ? I think this Elephant is grief stricken, possibly due to the loss of a dear friend.

Do you know if this Elephant shared a close friendship with anyone ? The Elephant-keeper said,
“There was a Dog who used to eat, sleep and play with the Elephant. He was taken by a stranger
three days ago”. The minister went back to the King and said, “Your majesty, in my opinion, the
royal Elephant is not sick, but he is lonesome without his dear friend, the Dog”. The King said,
“You’re right, friendship is one of the most wonderful things of life. Do you know where that
Dog is ?”

The Minister replied, “Elephantkeeper has informed me that a stranger took him away and he
doesn’t know his whereabouts”. The King asked, “How can we bring back my Elephant’s friend
and make him happy again ?” The Minister suggested, “Your Majesty, make a declaration that
whoever has the dog that used to live at the royal Elephant’s shed will be penalized”. The King
did the same and the man who had taken the dog, instantly turned him loose when he heard the
proclamation. As soon as he was freed, the Dog ran back as fast as he could to the Elephant’s
shed. The Elephant was so delighted to see the Dog that he picked his friend up with his trunk
and swung him back and forth. The Dog wagged his tail, while the Elephant’s eyes sparkled with
happiness. The King was content to see the Elephant happy once again and rewarded the
Minister for his wise judgment.
1. What was the Minister’s diagnosis of the Elephant’s condition ?
(A) The Elephant hated his keeper
(B) The Elephant was lonely
(C) The Elephant was starving
(D) The Elephant had hurt his leg and as in pain
(E) None of these
Ans : (B)

2. What method did the Minister suggest to the King to get back the Dog ?
(A) To declare that whoever had that particular Dog would be punished
(B) To keep a bowl of rice for the dog in the Elephant’s shed so that he could be lured

back to the palace


(C) To command the Elephantkeeper to look for the Dog in the village
(D) To persuade the Elephant to call out to the Dog
(E) None of these
Ans : (A)

3. Why had the Elephant become very sad ?


(A) He no longer got his daily bowl of rice
(B) He was unhappy with the King for having sold the Dog
(C) He missed his friend the Dog
(D) He was sold to an unknown man by his keeper
(E) None of these
Ans : (C)

4. What did the Elephant-keeper do to the Dog ?


(A) He sold the Dog to an unknown man for a price
(B) He hit the Dog as the Dog was eating the Elephant’s food
(C) He killed the Dog
(D) He complained to the King about the Dog
(E) None of these
Ans : (A)

5. Which of the following would be the most appropriate title for the passage ?
(A) Friends and Enemies
(B) The Playful Dog
(C) The King and the Minister
(D) The Elephant-keeper
(E) The Bond of Friendship
Ans : (E)

6. Why was the Elephant taken care of ?


(A) He was a very special Elephant as he could talk to Dogs
(B) He was a very loyal Elephant
(C) He was the strongest Elephant in the Kingdom
(D) He was weak and the King had a lot of sympathy for him
(E) None of these
Ans : (E)

7. Why did the Dog start going to the Elephant’s shed everyday ?
(A) He liked the Elephant a lot and wanted to become friends with him
(B) He was being fed by the King everyday
(C) He was fond of the Elephant’s shed
(D) He liked the taste of the rice being fed to the Elephant
(E) None of these
Ans : (D)

8. What did the Dog do once he was set free ?


(A) He ate rice to his heart’s content
(B) He thanked the King for his kindness
(C) He ran away from the Kingdom to a place faraway
(D) He ran back to his friend the Elephant
(E) None of these
Ans : (D)

9. What of the following can definitely be said about the Elephantkeeper ?


(1) He was greedy
(2) He was insensitive
(3) He was brave
(A) Only (1)
(B) Only (2)
(C) Only (1) and (2)
(D) Only (2) and (3)
(E) All (1), (2) and (3)
Ans : (C)

10. Which of the following can definitely be said about the King?
(1) He was compassionate
(2) He was deceitful
(3) He loved animals.
(A) Only (1)
(B) Only (1) and (3)
(C) Only (1) and (2)
(D) Only (2)
(E) All the three (1), (2) and (3)
Ans : (A)

Directions—(Q. 11–13) Choose the word/group of words which is most

similar in meaning to the word/group of words printed in bold as


used in the passage.

11. EXTRACTED
(A) Pulled
(B) Inserted
(C) Wing
(D) Dug out
(E) Received
Ans : (E)

12. DECLARATION
(A) Pact
(B) Praise
(C) Announcement
(D) Writ
(E) Resolve
Ans : (C)

13. KEEN
(A) Shallow
(B) Urgent
(C) Concentrated
(D) Deep
(E) Eager
Ans : (D)

Directions—(Q. 14–15) Choose the word/group of words which is most

opposite in meaning to the word/group of words printed in bold as

used in the passage.

14. RESIST
(A) Give in
(B) Please
(C) Struggle
(D) Try out
(E) Defy
Ans : (A)

15. SEPARATED
(A) Stuck
(B) United
(C) Estranged
(D) Bound
(E) Joined
Ans : (B)

Directions—(Q. 16–20) Which of the phrases (A), (B), (C) and (D) given

below each sentence should replace the phrase printed in bold in the

sentence to make it grammatically correct ? If the sentence is correct as it is given and

‘No correction is required”, mark (E) as the answer.

16. Trisha could not solve the problem at all and was at her wit’s ending.
(A) her wit’s end
(B) the wit ends
(C) her witty end
(D) the wit end
(E) No correction required
Ans : (A)

17. It’s a small theatre and the seats are uncomfortable, but the saving

grace is that the air conditioning is good.


(A) grace to save
(B) gracing save
(C) saver grace
(D) save to grace
(E) No correction required
Ans : (E)

18. Tarun had to prepare the document for his meeting urgently but he was hardly

pressed for time.


(A) hard pressed for timely
(B) hard pressed for time
(C) hardly press to time
(D) hard pressing to timely
(E) No correction required
Ans : (B)

19. Suraj lied from his teeth to get out of the tense situation with his

boss—
(A) lies for his teeth
(B) lie to his teeth
(C) lied through his teeth
(D) lied from his tooth
(E) No correction required
Ans : (C)

20. Satish lay in bed wide awaken as he was worried about his exams

starting the next day—


(A) widen awakening
(B) widely awake
(C) wide and awake
(D) wide awake
(E) No correction required
Ans : (D)

Directions—(Q. 21–25) In each question below, a sentence with four words

printed in bold types is given. These are lettered as (A), (B), (C) and

(D). One of these four words printed in bold may be either wrongly spelt

or inappropriate in the context of the sentence. Find out the word which is wrongly

spelt or inappropriate, if any. The letter of that word is your answer. If all the

words printed in bold are correctly spelt and also appropriate in the

context of the sentence. mark (E) i.e., ‘All correct’ as your answer.

21. People went crazy (A) when the musician along with his

friend, (B) took to the staging (C) and belted out

popular (D) tracks. All correct (E)


Ans : (C)

22. For a growing number of coupals, (A) adoption is not a helpless

compulsion (B) but also deliberate, (C) and often

noble, (D) choice. All correct (E)


Ans : (A)

23. On two consecutive (A) nights the two greatest icones

(B) of show business were honoured with unprecedented (C)

affection. (D) All correct (E)


Ans : (B)

24. We are so busy looking at mediocrity (A) that when truly outstanding

achievement (B) stares us in the face we resort (C) to

the same terms of praise. (D) All correct (E)


Ans : (E)

25. The unlikely migration (A) was officially (B)

explained as a move to meet expanding (C) business

oportunities. (D) All correct (E)


Ans : (D)

Directions—(Q. 26–30) Rearrange the following six sentences (1), (2), (3),

(4), (5) and (6) in the proper sequence to form a meaningful paragraph; then answer the

questions given below them.


(1) He immediately acknowledged Mohan’s good work and invited him to his home for dinner.
(2) One day a wealthy merchant sent his son’s bicycle to the shop for repair.
(3) The next day the merchant came to claim the bicycle and noticed that it was shiny.
(4) After repairing the bicycle, Mohan cleaned it up and made it look new.
(5) Once upon a time, there was a boy named Mohan who worked as an apprentice in a bicycle

shop.
(6) Other apprentices in the shop laughed at Mohan for doing unnecessary work.

26. Which of the following should be the SECOND sentence after

rearrangement ?
(A) 1
(B) 2
(C) 3
(D) 4
(E) 6
Ans : (B)

27. Which of the following should be the THIRD sentence after

rearrangement ?
(A) 1
(B) 2
(C) 3
(D) 4
(E) 5
Ans : (D)

28. Which of the following should be the FIRST sentence after

rearrangement ?
(A) 1
(B) 2
(C) 3
(D) 4
(E) 5
Ans : (E)

29. Which of the following should be the LAST (SIXTH) sentence after

rearrangement ?
(A) 1
(B) 2
(C) 4
(D) 5
(E) 6
Ans : (A)

30. Which of the following should be the FOURTH sentence after

rearrangement ?
(A) 2
(B) 3
(C) 4
(D) 5
(E) 6
Ans : (E)

Directions—(Q. 31– 40) Read each sentence to find out whether there is any

grammatical error or idiomatic error in it. The error, if any, will be in one part of the

sentence. The letter of that part is the answer. If there is ‘No error’, the answer is (E).

(Ignore errors of punctuation, if any.)

31. In about five minutes, they would (A) / cover the distance from the road to the point

(B) / where the waves would (C) / begin lick their feet. (D) No error (E)
Ans : (D)
32. The devastating experiences (A) / of many wars taught some countries (B) / the

necessary of pursuing peace (C) / at the expense of nationalist egos. (D) No error (E)
Ans : (E)

33. The former superstar recently (A) / visit an orphanage, (B) / fuelling speculation that

(C) / she is planning to adopt a child. (D) No error (E)


Ans : (B)

34. Accordance to a new study (A) / taking steps to remain healthy and young (B) / may help

delay (C) / the onset of Alzheimer’s disease. (D) No error (E)


Ans : (A)

35. A day after he was diagnosed with (A) / swine flu, preventive measures were put in

place (B) / to ensure that others were not (C) / affected by the dreaded virus. (D) No

error (E)
Ans : (B)

36. The administration has conclusive (A) / that it is retailers who are (B) / responsible

for upsetting (C) / the city’s household budget. (D) No error (E)
Ans : (A)

37. The assurances, unfortunately, (A) / remained on paper, as (B) / neither the Centre or

the state initiated steps (C) / for the development of the backward region. (D) No error

(E)
Ans : (C)

38. Sediment deposit along the coast (A) / may be the primary reason for (B) / the change

in conditions, (C) / but a lot more remains to be understand. (D) No error (E)
Ans : (D)

39. A committee will be set up (A) / to explore pros and cons by (B) / a common fee

structure, and will (C) / take a final decision on it within a week. (D) No error (E)
Ans : (E)

40. The infection which causes (A) / gums to bleed and teeth to fall out (B) / results from
the build-up of (C) / a particular bacteria that is common to most mouths. (D) No error (E)
Ans : (D)

Directions—(Q. 41–50) In the following passage there are blanks, each of

which has been numbered. These numbers are printed below the passage and against each, five

words are suggested, one of which fits the blank appropriately. Find out the appropriate

word in each case.

Once upon a time there was a prince who wanted to marry a princess; but she would have to

be a real princess. He …(41)… all over the world to find one, but nowhere

could he get what he wanted. One evening during a terrible storm; there was thunder and

lightning, and the rain poured down in torrents. Suddenly a knocking was

…(42)… at the palace door, and the old king went to open it. It was a

princess standing out there. But, good gracious ! What a sight the rain and the wind had

made her look. The water ran down her hair and clothes; into the toes of her shoes and out

again at the heels. And yet she insisted that she was a real princess. Well, we’ll soon

…(43)… out, thought the old queen. But she said nothing, went into the

bedroom, took all the bedding off the bedstead, and …(44)… a pea on the

bottom; then she took twenty mattresses and laid them on the pea, and then twenty quilts on

…(45)… of the mattresses. On this the princess had to lie all night. In

the …(46)… she was asked how she had slept. “Oh, very badly !” said she.

“I scarcely closed my eyes all night. Heaven only knows what was in the bed. But I was

lying on something hard, as a …(47)… I am black and blue all over my body.

It’s horrible !” Now they knew that she was a real princess because she had

…(48)… the pea right through the twenty mattresses and the twenty quilts.
Nobody but a real princess could be as …(49)… as that. So the prince took

her for his wife, for now he …(50)… that he had a real princess.

41. (A) called


(B) tour
(C) sent
(D) saw
(E) travelled
Ans : (E)

42. (A) made


(B) felt
(C) heard
(D) seen
(E) sounded
Ans : (C)

43. (A) assure


(B) find
(C) judge
(D) mark
(E) try
Ans : (B)

44. (A) drew


(B) flung
(C) placed
(D) cooked
(E) stitch
Ans : (C)

45. (A) top


(B) head
(C) bottom
(D) between
(E) middle
Ans : (A)

46. (A) morning


(B) dinner
(C) room
(D) fields
(E) dark
Ans : (A)
47. (A) vengeance
(B) price
(C) cause
(D) result
(E) time
Ans : (D)

48. (A) slept


(B) felt
(C) located
(D) carried
(E) found
Ans : (B)

49. (A) worried


(B) rough
(C) irritable
(D) sensitive
(E) pretty
Ans : (D)

50. (A) trust


(B) assured
(C) wanted
(D) think
(E) knew
Ans : (E)

Rajasthan Gramin Bank Clerk Exam., 2009


(Held on 29-11-2009)
English Language : Solved Paper
Directions—(Q. 1–10) In the following passage, there are blanks, each of which has been
numbered. These numbers are printed below the passage and against each, five words are
suggested, one of which fits the blank appropriately. Find out the appropriate word in each case.

India, it is …(1)… said, is not a …(2)… but a continent. From North to South and East to …(3)
…, the people are different, the languages are …(4)…, the customs are different. There are few
countries on earth with the enormous variety that India has to offer. India is far from the …(5)…
country in the world to travel …(6)…. It can be hard going. Yet it is …(7)… worth visiting.

Basically India …(8)… what you make of it and what you want it to be. If you want to see
temples, there are temples with enough styles and types. If it is history you want, India has …(9)
… of it; the forts, abandoned cities, ruins, battle fields and monuments all have their …(10)… to
tell.
1. (A) not
(B) often
(C) never
(D) twice
(E) read
Ans : (B)

2. (A) country
(B) village
(C) city
(D) town
(E) place
Ans : (A)

3. (A) North
(B) East
(C) West
(D) rest
(E) no where
Ans : (C)

4. (A) same
(B) some
(C) national
(D) different
(E) few
Ans : (D)

5. (A) difficult
(B) good
(C) matching
(D) rich
(E) easiest
Ans : (E)

6. (A) around
(B) inside
(C) train
(D) cities
(E) upon
Ans : (A)

7. (A) no
(B) never
(C) yes
(D) all
(E) thrice
Ans : (D)

8. (A) was
(B) in
(C) is
(D) are
(E) gives
Ans : (C)

9. (A) books
(B) plenty
(C) notes
(D) little
(E) some
Ans : (B)

10. (A) poverty


(B) plenty
(C) forts
(D) cities
(E) tales
Ans : (E)

Directions—(Q. 11–25) Read the following passage carefully and answer the questions given
below it. Certain words in the passage have been printed in bold to help you locate them when
answering some of the questions—

He was a funny looking man with a high, bald, dome shaped head, a face very small in
comparison and a long wavy beard. His unusual features were a standing joke among his friends.
He was a poor man—an idler. He didn’t work at his trade—a stonecutter, more than what was
necessary to keep his wife and three boys alive. He preferred to talk but since his wife was an
irate complaining woman, he loved to be away from home.

The whole city he lived in was seething with argumentation. The city was Athens and the man
was Socrates–the Greek philosopher. He had funny ways and notions. And to the astonishment
of all, the Oracle at Delphi, the priestess when asked, “Who is the wisest man in Athens ?”
mentioned Socrates, Socrates was the evangelist of clear thinking : he would present people with
questions pretending he didn’t know the answers and get them to make astounding admissions.
Socrates would go upto a prominent statesman coming to the end of a speech on ‘courage’,
about the glory of dying for one’s country and say, “Forgive my intrusion, but just what do you
mean by courage ?”
“Courage is sticking to your post in danger” would be the reply. “But supposing good strategy
demands that you retire ?”, Socrates would ask. “You wouldn’t stay in that case” the man would
be forced to admit. Socrates would persist, “Then is courage sticking to your post or retiring ?”.
‘I’m afraid I don’t know.” ‘I don’t either”, Socrates would say “but perhaps it is not different
from just doing the reasonable thing regardless of the danger.”

11. Socrates by profession was—


(A) Philosopher
(B) Idler
(C) Politician
(D) Evangelist
(E) None of these
Ans : (E)

12. The author’s main objective in writing the passage is to—


1. describe the situation prevalent in Athens in Socrates’ time.
2. describe Socrates’ physical characteristics.
3. criticize Socrates’ way of life.
(A) Only 1
(B) Only 3
(C) Both 2 and 3
(D) Both 1 and 3
(E) None of these
Ans : (A)

13. How did Socrates perceive courage ?


(A) Not abandoning one’s post
(B) Retiring from one’s post
(C) Dying for one’s country
(D) Rightful action in the face of danger
(E) Not changing one’s opinion
Ans : (A)

14. Which of the following best describes Socrates ?


(A) He loved to argue with his wife
(B) He was a handsome man
(C) He was not the wisest man
(D) He had no sense of humour
(E) None of these
Ans : (D)

15. Which of the following is true in the context of the passage ?


(A) The Oracle at Delphi did not know anything
(B) Socrates could not support his family
(C) Socrates’ ideas were the same as those of other citizens of Athens
(D) Socrates believed in reason and logic
(E) All of these
Ans : (D)

16. What was Socrates’ style of arguing ?


(A) Make fun of people by pointing out the flaws in their arguments
(B) Preach till his audience was convinced about his point of view
(C) Question a person’s assumptions to lead him to a different point of view
(D) Rudely interrupt speeches and question the orator
(E) Insult the speaker so that he would get angry and lose the argument
Ans : (C)

17. An appropriate title for the passage would be—


(A) Ancient Greece
(B) He Taught Us to Think
(C) True Courage
(D) The Oracle at Delphi
(E) Arguing
Ans : (D)

18. Which of the following is not true in the context of the passage ?
(A) Socrates valued logic
(B) Socrates had an unhappy personal life
(C) Socrates was considered the wisest man in Athens
(D) Socrates worked very hard so that his family could live in comfort
(E) None of these
Ans : (D)

19. Why did the Oracle name Socrates as the wisest man in Athens ?
(A) Socrates had the answer to all questions
(B) Socrates had a sense of humour
(C) Socrates believed in reasoning; not in blind faith
(D) Socrates was a good orator
(E) Socrates valued friendship
Ans : (C)

Directions—(Q. 20-22) Pick out the word which is closest in meaning to the word printed in
bold as used in the passage.

20. Notion
(A) Rules
(B) Clothes
(C) Thoughts
(D) Advice
(E) Gestures
Ans : (C)
21. Seething
(A) Assembling
(B) Suffering
(C) Receiving
(D) Searching
(E) None of these
Ans : (B)

22. Astonishment
(A) Disgust
(B) Offence
(C) Realization
(D) Interest
(E) Wonder
Ans : (E)

Directions—(Q. 23–25) Pick out the word which is MOST OPPOSITE in meaning to the word
printed in bold as used in the passage.

23. Irate
(A) Irritable
(B) Easygoing
(C) Sleepy
(D) Calm
(E) Weak
Ans : (D)

24. Prominent
(A) Ordinary
(B) Dull
(C) Unprofessional
(D) Intelligent
(E) Incapable
Ans : (A)

25. Forgive
(A) Exempt
(B) Abstain
(C) Lenient
(D) Pardon
(E) Condemn
Ans : (E)

Directions—(Q. 26–30) Read each sentence to find out whether there is any grammatical error
in it. The error, if any, will be in one part of the sentence, the letter of that part is the answer. If
there is no error, mark your answer as (E).
26. Look ! (A) /somebody (B) / had climbing up (C) / that there over there. (D) No Error (E)
Ans : (C)

27. According to (A) / at a party (B) / a few months ago (C) / and he did seems fine. (D) No
Error (E)
Ans : (D)

28. I saw Jack (A) / at a party (B) / a few month ago (C) / and he did seems fine. (D) No Error
(E)
Ans : (D)

29. Betty has a (A) / term paperto write (B) / she has a lab report (C) / in finish too. (D) No Error
(E)
Ans : (D)

30. If a hotel is (A) / very expensive, it does not (B) / mean that its (C) / services is best. (D) No
Error (E)
Ans : (D)

Directions—(Q. 31–35) In each of the following sentences there are blank spaces. Below each
sentence there are five words denoted by letters (A), (B), (C), (D) and (E). Find out which of the
words can be filled up in the blanks in the sentence to make the sentences meaningfully
complete.

31. Looking ……… I now see all the mistakes I made when I was young.
(A) back
(B) on
(C) forward
(D) into
(E) above
Ans : (A)

32. The ship of the merchants…… tossed to the island by the rough sea wave.
(A) had
(B) was
(C) were
(D) should
(E) will
Ans : (B)

33. They……… his house and found a number of stolen articles.


(A) broke
(B) launched
(C) searched
(D) smashed
(E) razed
Ans : (C)

34. You can either pay the ……… or go to prison for a month.
(A) fine
(B) home
(C) house
(D) privacy
(E) levy
Ans : (A)

35. The wolves ……… all night and kept us awake.


(A) cried
(B) howled
(C) scared
(D) played
(E) barked
Ans : (B)

Directions—(Q. 36–40) Which of the phrase (A), (B), (C) and (D) given below should replace
the phrase given in bold in the following sentence to make the sentence grammatically
meaningful and correct ? If the sentence is correct as it is and no correction is required mark (E)
as the answer.

36. The research reported here is based at a survey.


(A) based on a
(B) base at a
(C) based on an
(D) based into a
(E) No correction required
Ans : (A)

37. It is increasingly being realized that human resource development is necessary.


(A) been realized that
(B) being realized that
(C) being realized on
(D) realizing that
(E) No correction required
Ans : (E)

38. The north–eastern region of India presents a somehow unique situation.


(A) a somewhere unique
(B) a somewhat unique
(C) an somewhat unique
(D) a unique somehow
(E) No correction required
Ans : (B)

39. This approach has affectedly the methodology of research.


(A) have affectedly
(B) had affect
(C) has affected
(D) will affectedly
(E) No correction required
Ans : (C)

40. It is in this land where people by different corners live together.


(A) people different by
(B) peoples by differently
(C) different by people
(D) people from different
(E) No correction required
Ans : (D)

Directions—(Q. 41–50) Which of the phrase (A), (B), (C) and (D) given below should replace
the phrase given in bold in the following sentence to make the sentence grammatically
meaningful and correct ? If the sentence is correct as it is and no correction is required mark (E)
as the answer.

41. He must have behaved impolite and suffer because of that.


(A) impolitely and suffer
(B) impolitely and suffered
(C) impolite and suffered
(D) impolitely and have suffer
(E) No correction required
Ans : (B)

42. Because of his efficiency, the Director wanted him to work as assistant.
(A) wanted to him
(B) wants him to be
(C) wanted he to
(D) wanted of him
(E) No correction required
Ans : (E)

43. Even less intelligent students can be succeeded through hard work and perseverance.
(A) get succession
(B) be success
(C) get success
(D) be getting succeeded
(E) No correction required
Ans : (C)
44. The Neo Club has been extended financial and technical help to poor and deserving women.
(A) has been extending
(B) have been extending
(C) is being extended
(D) have been extended
(E) no correction required
Ans : (A)

45. Highly intelligent people seldom tolerate any ambiguity.


(A) tolerate hardly any ambiguity
(B) hardly tolerate ambiguous
(C) tolerate any clarity whatsoever
(D) hardly tolerate any lacuna
(E) No correction required
Ans : (E)

46. Being toiled for two hours, he sat down to take some rest.
(A) Have toiled for
(B) Had toiled for
(C) Having to toiled at
(D) Having toiled for
(E) No correction required
Ans : (D)

47. Some people by naturally talk more than others.


(A) by naturally talk
(B) naturally talk
(C) nature by talking
(D) naturally by talking
(E) No correction required
Ans : (B)

48. One of the basic choices we make in life is whether following the path of smart selfishness
or try to avoid all problems.
(A) while following
(B) is whether to follow
(C) is whether or not to follow
(D) has been whether follow
(E) No correction required
Ans : (B)

49. Some people experienced life only if they are totally at the mercy of their feelings.
(A) experience life as if they are
(B) experienced life as if they are
(C) experiencing if only when they are life
(D) experience life when they were
(E) No correction required
Ans : (A)

50. Good-natured people have been suffered because of their politicking companions.
(A) Have suffereing with
(B) Are suffered because of
(C) He does worry
(D) May suffer because of
(E) No correction required
Ans : (D)

Gurgaon Gramin Bank Clerk Exam., 2009


(Held on 1-11-2009)
English Language : Solved Paper
Directions—(Q. 1–15) Read the following passage carefully and answer the questions given
below it. Certain words/phrases have been printed in bold to help you locate them while
answering some of the questions.

Before my many years’ service in a restaurant, I attended a top science university. The year was
2023 and I was finishing the project that would win me my professorship.

My forty-second birthday had made a lonely visit the week before, and I was once again by
myself in the flat. Like countless other mornings, I ordered a bagel from the toaster. ‘Yes, sir!’ it
replied with robotic relish, and I began the day's work on the project. It was a magnificent
machine, the thing I was making– capable of transferring the minds of any two beings into each
other’s bodies.

As the toaster began serving my bagel on to a plate, I realized the project was in fact ready for
testing. I retrieved the duck and the cat–which I had bought for this purpose from their
containers, and set about calibrating the machine in their direction. Once ready, I leant against
the table, holding the bagel I was too excited to eat, and initiated the transfer sequence. As
expected, the machine whirred and hummed into action, my nerves tingling at its synthetic
sounds. The machine hushed, extraction and injection nozzles poised, scrutinizing its targets. The
cat, though, was suddenly gripped by terrible alarm. The brute leapt into the air, flinging itself
onto the machine. I watched in horror as the nozzles swung towards me; and, with a terrible,
psychedelic whirl of colours, felt my mind wrenched from its sockets.

When I awoke, moments later, I noticed first that I was two feet shorter. Then, I realized the lack
of my limbs, and finally it occurred to me that I was a toaster. I saw immediately the solution to
the situation–the machine could easily reverse the transfer–but was then struck by my utter
inability to carry this out. After some consideration, using what I supposed must be the toaster’s
onboard computer; I devised a strategy for rescue. I began to familiarize myself with my new
body: the grill, the bread bin, the speaker and the spring mechanism. Through the device’s
rudimentary eye–with which it served its creations–I could see the internal telephone on the
wall. Aiming carefully, I began propelling slices of bread at it. The toaster was fed by a large
stock of the stuff, yet as more and more bounced lamely off the phone, I began to fear its
exhaustion. Toasting the bread before launch proved a wiser tactic. A slice of crusty wheat
knocked the receiver off its cradle, and the immovable voice of the reception clerk answered.
Resisting the urge to exclaim my unlikely predicament, I called from the table : ‘‘I’m having a
bit of trouble up here, Room 91. Could you lend a hand ?’’ ‘‘Certainly, sir, there’s a burst water
pipe on the floor above, I suppose I’ll kill two birds with one stone and sort you out on the
way.”

The clerk arrived promptly, and after a detailed and horrifying explanation, finally agreed to
press the button on the machine and bring me back to my original state.

1. Why did the author believe that he would earn professorship ?


(A) The author was conducting experiments on his cat and duck
(B) The author was working on making a new toaster
(C) The author was working on a project where he was inventing a new machine
(D) The author thought that he was very intelligent
(E) None of these
Ans : (C)

2. Why was the author afraid that the bread would get exhausted ?
(A) Because he thought he would remain a toaster all his life
(B) Because although he was throwing a lot of bread at the phone, the cradle was not coming off
the hook
(C) Because there was limited bread in the room
(D) Because the author had forgotten to load bread in the toaster
(E) None of these
Ans : (A)

3. What does the phrase ‘kill two birds with one stone’ used in the passage mean ?
(A) Take care of two things at one time
(B) Kill two birds with one bullet
(C) Kill both the cat and the duck simultaneously
(D) Please the clerk and gain professorship at the same time
(E) None of these
Ans : (A)

4. Why did the author bring out the cat and the duck ?
(A) So that they could watch while he performed the experiment
(B) So that he could perform the experiment on them
(C) So that the cat could jump on the machine
(D) So that the author could call the clerk
(E) None of these
Ans : (B)
5. The term ‘immovable voice’ as used in the passage means……
(A) hoarse voice
(B) fixed voice
(C) steady voice
(D) loud voice
(E) shaky voice
Ans : (B)

6. Which of the following is NOT true in the context of the passage ?


(A) The clerk was part of the experiment right from the start
(B) The clerk managed to bring the author back to his original state
(C) The experiment did not go completely as planned
(D) The author’s mind was transferred into the toaster
(E) All of the above are true
Ans : (A)

7. What was the author making ?


(A) He was making a toaster which could talk
(B) He was making a machine which could control animal behaviour
(C) He was making a machine which could convert itself into a toaster
(D) He was making a machine which could exchange the minds of two entities
(E) None of these
Ans : (D)

8. Why was the author two feet shorter when he woke up ?


(A) Because he was transformed into a duck
(B) Because the cat had jumped on him
(C) Because he was transformed into a telephone
(D) Because he had not made the machine correctly
(E) None of these
Ans : (E)

9. Who does the term 'The brute' used in the passage refer to ?
(A) The author
(B) The toaster
(C) The machine
(D) The clerk
(E) None of these
Ans : (E)

10. How did the author manage to call the clerk ?


(A) By dragging the phone towards himself
(B) By hitting the button on the machine
(C) By shouting for help
(D) By flinging toast at the phone on the wall and then asking the clerk for help
(E) None of these
Ans : (D)

Directions–(Q. 11–13) Choose the word which is most similar in meaning to the word printed
in bold as used in the passage.

11. SYNTHETIC
(A) loud
(B) clothed
(C) real
(D) electronic
(E) soft
Ans : (B)

12. FLINGING
(A) touching
(B) holding
(C) bending
(D) feeling
(E) throwing
Ans : (E)

13. INITIATED
(A) withstand
(B) worked
(C) stopped
(D) started
(E) culminate
Ans : (D)

Directions–(Q. 14–15) Choose the word which is most opposite in meaning to the word printed
in bold as used in the passage.

14. TERRIBLE
(A) better
(B) wonderful
(C) extreme
(D) awful
(E) fearful
Ans : (B)

15. RUDIMENTARY
(A) advanced
(B) basic
(C) evil
(D) forward
(E) artificial
Ans : (A)

Directions–(Q. 16–20) Which of the phrases (A), (B), (C) and (D) given below each sentence
should replace the phrase printed in bold in the sentence to make it grammatically correct ? If the
sentence is correct as it is given and no correction is required, mark (E) as the answer.

16. One of the greatest actors pass away in his sleep at dawn today.
(A) passed away
(B) is pass away
(C) passing away
(D) was passed away
(E) No correction required
Ans : (A)

17. In light of the current scenario people should think of saving money rather spending it.
(A) more of
(B) because then
(C) rather than
(D) rather of
(E) No correction required
Ans : (C)

18. In every society there will always be a struggle between the rich and the poor.
(A) will always being
(B) is always being
(C) be always
(D) is always be
(E) No correction required
Ans : (E)

19. As he was keen in doing some social work, he established an NGO for the benefit of the
downtrodden.
(A) is eager
(B) was keen on
(C) was eager
(D) is keen on
(E) No correction required
Ans : (B)

20. I am very thanking to you for helping me get through this situation.
(A) greatly thanking for
(B) great thankful to
(C) very thanking for
(D) very thankful to
(E) No correction required
Ans : (D)

Directions–(Q. 21–25) In each question below, a sentence with four words printed in bold type
is given. These are lettered as (A), (B), (C) and (D). One of these four words printed in bold may
be either wrongly spelt or inappropriate in the context of the sentence. Find out the word
which is wrongly spelt or inappropriate, if any. The letter of that word is your answer. If all the
words printed in bold are correctly spelt and also appropriate in the context of the sentence, mark
(E) i.e., 'All correct' as your answer.

21. She opened (A) and closed the apartment (B) door carefuly (C) and finally went for a
walk. (D) All correct (E)
Ans : (C)

22. The information (A) contain (B) in this mail is privileged (C) and confidential. (D) All
correct (E)
Ans : (B)

23. In the end, we will remember (A) not the words of our enemies, (B) but the silence (C) of
our friends. (D) All correct (E)
Ans : (E)

24. He decided to quite (A) the company (B) as he was not being (C) paid well. (D) All correct
(E)
Ans : (A)

25. The first step towards (A) getting someway (B) is to decide (C) that you are not going to
stay (D) where you are. All correct (E)
Ans : (B)

Directions—(Q. 26–30) Rearrange the following six sentences (1), (2), (3), (4), (5) and (6) in the
proper sequence to form a meaningful paragraph; then answer the questions given below them.
(1) When I arrived, they all sat around the table while I was given a chair to sit.
(2) And to show that the English could do great things, that evening I ate more than usual.
(3) One day the Emperor and his young Princes stated that they would like to invite me for
dinner.
(4) Seeing this they further complimented me and I felt ecstatic.
(5) Of course I accepted and visited their palace.
(6) They were all full of admiration when they saw how much I could eat.

26. Which of the following should be the SECOND sentence after rearrangement ?
(A) 2
(B) 3
(C) 4
(D) 5
(E) 6
Ans : (D)
27. Which of the following should be the FOURTH sentence after rearrangement ?
(A) 2
(B) 3
(C) 4
(D) 6
(E) 5
Ans : (A)

28. Which of the following should be the FIRST sentence after rearrangement ?
(A) 1
(B) 2
(C) 3
(D) 4
(E) 6
Ans : (C)

29. Which of the following should be the LAST (SIXTH) sentence after rearrangement ?
(A) 2
(B) 3
(C) 4
(D) 6
(E) 5
Ans : (C)

30. Which of the following should be the THIRD sentence after rearrangement ?
(A) 1
(B) 2
(C) 3
(D) 4
(E) 6
Ans : (A)

Directions—(Q. 31–40) Read each sentence to find out whether there is any grammatical error
or idiomatic error in it. The error, if any, will be in one part of the sentence. The letter of that part
is the answer. If there is no error, the answer is (E). (Ignore errors of punctuation, if any.)

31. One of the best way (A) / of improving (B) / your English is to (C) / read the language. (D)
No error (E)
Ans : (A)

32. I was halting (A) /at the station for (B) / the train to arrive when (C) / suddenly there was a
loud noise. (D) No error (E)
Ans : (E)

33. I reached (A) / lately for the meeting (B) / as I got (C) / stuck in traffic. (D) No error (E)
Ans : (B)

34. Unless you (A) / work hard you (B) /will not achieve (C) / success in life. (D) No error (E)
Ans : (E)

35. The labourers working at (A) / the construction site were (B) / very tiring and (C) / wanted to
rest. (D) No error (E)
Ans : (C)

36. As it was (A) / the boss’s birthday (B) / all the employees wanted (C) / to buy a gift to him.
(D) No error (E)
Ans : (D)

37. Priya got a (A) / new job and she (B) / happily told everyone (C) / about then. (D) No error
(E)
Ans : (D)

38. Being new to the country (A) / myself decided to first take a (B) / quick tour around (C) / the
place I was staying. (D) No error (E)
Ans : (B)

39. Since I had bought (A) / quite a few items (B) / the store keeper smiles (C) / and gave me a
discount. (D) No error (E)
Ans : (C)

40. After waiting for (A) /a very long time (B) / the man finally(C) / returned home frustrated.
(D) No error (E)
Ans : (E)

Directions—(Q. 41–50) In the following passage there are blanks, each of which has been
numbered. These numbers are printed below the passage and against each, five words are
suggested, one of which fits the blank appropriately. Find out the appropriate word in each case.

My father …(41)… a small estate in Punjab; I …(42)… the third of five sons. When, I was
fourteen years …(43) …, my father sent me to college. But as it was expensive, I …(44)… after
staying there for three years. I was then apprenticed to Mr. Bose, a well known surgeon. I
worked …(45)…him for four years. During this period, my father would send me some money,
which I used to study navigation, because I always felt that I would one day …(46)… to faraway
lands.

When I …(47)… Mr. Bose, my father sent me to medical college. After …(48)… three years
there, Mr. Bose got me the position of doctor on a ship. I …(49)… on the ship for three and a
half years, after which I thought I would settle down in Kolkata. My master Mr. Bose used to
send me some of his patients and I bought a small house. After a …(50)…, I got married to a
local girl.
41. (A) live
(B) sell
(C) owned
(D) own
(E) lived
Ans : (C)

42. (A) was


(B) had
(C) for
(D) is
(E) wanted
Ans : (A)

43. (A) aged


(B) older
(C) elder
(D) age
(E) old
Ans : (E)

44. (A) leave


(B) went
(C) gone
(D) left
(E) go
Ans : (D)

45. (A) of
(B) under
(C) within
(D) to
(E) off
Ans : (B)

46. (A) visited


(B) travel
(C) flying
(D) visit
(E) travelling
Ans : (B)

47. (A) with


(B) finished
(C) leave
(D) wait
(E) left
Ans : (E)

48. (A) spent


(B) study
(C) complete
(D) nearly
(E) casual
Ans : (D)

49. (A) ran


(B) work
(C) stayed
(D) sleep
(E) travel
Ans : (C)

50. (A) while


(B) brief
(C) pause
(D) time
(E) stay
Ans : (A)

32. The Buffer Stock of foodgrains at the beginning of February 2009 stood at—
(A) 16•2 million tonnes
(B) 20•0 million tonnes
(C) 37•4 million tonnes
(D) 36•2 million tonnes
(E) None of these
Ans : (D)

33. Public Private Partnership came to be introduced in India to—


(A) Mitigate the financial burden of the governments
(B) Respond to the technological change that allow unbundling of infrastructure
(C) Alter the character of goods and services from pure public to private ones
(D) All of the above
(E) None of these
Ans : (A)

34. Heteroscedasticity in econometric analysis means—


(A) The variance for each disturbance term (Vi) is the same for all i’s
(B) The variance for each disturbance term (Vi) is not the same for all i’s
(C) The co-variance for each pair of disturbance term is the same
(D) The co-variance for each pair of disturbance term is not the same
(E) None of these
Ans : (B)
35. Autocorrelation in econometric analysis refers to—
(A) The correlation between the values of different variables
(B) The correlation between the values of alternative variables
(C) The correlation between successive values of the same variable
(D) All of the above
(E) None of these
Ans : (A)

36. Durbin-Watson Test is applied in econometric models to test—


(A) Presence of heteroscedasticity in a model
(B) For autocorrelated errors in the sample data
(C) For multicorrelinearity
(D) For autoregressive disturbances
(E) None of these
Ans : (B)

37. A weakly stationary stochastic process is so called, if—


(A) Its mean and variance are constant over time
(B) The value of covariance between two time periods depends only on lag between
two time periods
(C) The value of covariance does not depend on the actual time at which the
covariance is computed
(D) All the above
(E) None of these
Ans : (D)

38. Dummy variables are used in regression models—


(A) To represent important variables that are not qualitatively measurable
(B) To capture the effect of specific attributes
(C) As binary variables
(D) To distinguish between two groups of population
(E) All of the above
Ans : (C)

39. Principal components in econometric analysis—


(A) Are linear combinations
(B) Help to increase the degrees of freedom
(C) Help to reduce high degree of multicorrelinearity
(D) All of the above
(E) None of these
Ans : (A)

40. If world population grows at 2•6 per cent, then it will double in—
(A) 44•34 years
(B) 36•92 years
(C) 29•32 years
(D) 26•66 years
(E) None of these
Ans : (E)
United India Insurance Co. Administrative Officers Exam., 2009
(Held on 24-5-2009)
General Knowledge and Current Affairs : Solved Paper

1. What is Siam now known as ?


(A) Laos
(B) Thailand
(C) Vietnam
(D) Cambodia
Ans : (B)

2. The correct group of cities, through which National Highway No. 8 passes is—
(A) Bikaner, Ajmer, Kota
(B) Jodhpur, Jaipur, Bikaner
(C) Jaipur, Udaipur, Ahmedabad
(D) Jaipur, Pune, Udaipur
Ans : (C)

3. Through its power of judicial review, the Supreme Court ensures—


(A) Judicial Supremacy
(B) A democrtic government in the country
(C) A constitutional government in the country
(D) That Parliament Sovereignty is curbed
Ans : (D)

4. Which unit of valuation is known as ‘Paper Gold’ ?


(A) Petrodollar
(B) SDR
(C) Eurodollar
(D) GDR
Ans : (B)

5. What is the playing time of the full version of the Indian National Anthem ?
(A) 47 seconds
(B) 50 seconds
(C) 52 seconds
(D) 60 seconds
Ans : (C)

6. When was Barack Obama sworn in as the 44th President of U.S.A. ?


(A) January 15, 2009
(B) January 20, 2009
(C) January 25, 2009
(D) January 30, 2009
Ans : (B)

7. The Movie ‘Slumdog Millionaire’ is based on a novel ‘Q and A’ written by an Indian—


(A) Khuswant Singh
(B) Khalid Mohammad
(C) Sabarjit Singh
(D) Vikas Swarup
Ans : (D)

8. Who has been appointed the first Director-General of the ‘National Investigation Agency
(NIA)’ ?
(A) P. S. Sharma
(B) A. Gopalswami
(C) Radha Vinod Raju
(D) K. C. Verma
Ans : (C)

9. Under which Article of the Indian Constitution, can the President of India withhold his assent
to a Bill passed by both the houses of Parliament ?
(A) Article, 100
(B) Article, 111
(C) Article, 200
(D) Article, 222
Ans : (B)

10. Which language is spoken by the people of Tripura ?


(A) Bengali
(B) Telugu
(C) Assamese
(D) Malayalam
Ans : (A)

11. Surya Shekhar Ganguly is a player of—


(A) Chess
(B) Cricket
(C) Golf
(D) Hockey
Ans : (A)

12. We very frequently read about Special Economic Zones (SEZs) in newspapers. These SEZs
were established with which of the following objectives ?
1. To protect domestic market from direct competition from multinationals
2. To provide more capital to agricultural and allied activities
3. To attract foreign investment directly
(A) Only 1
(B) Only 2
(C) Only 3
(D) 1, 2 and 3 all
Ans : (C)

13. With which is ‘Uro Cup’ associated ?


(A) Cricket
(B) Football
(C) Polo
(D) Wrestling
Ans : (B)

14. World Environment Day is on—


(A) June, 5
(B) June, 11
(C) June, 17
(D) June, 30
Ans : (A)

15. Which rock-shelter in India bears largest number of paintings ?


(A) Adamgarh
(B) Bhimbetka
(C) Ghagharia
(D) Lekhia
Ans : (B)

16. Heat received by earth from the sun is known as—


(A) Isolation
(B) Solar heat
(C) Solar radiation
(D) Thermal radiation
Ans : (A)

17. Who invented Coronograph which allowed the study of the sun without waiting for a total
solar eclipse ?
(A) Donald Menzel
(B) Pierre Jenseen
(C) John Evershed
(D) Bernard Lyot
Ans : (D)

18. Who wrote ‘The Structure of the Universe’, giving in detail the origin of the universe and its
present status, among other things ?
(A) Robert Jastrow
(B) S. Chandrashekhar
(C) Jayant V. Narlikar
(D) Carl Sagan
Ans : (C)

19. Name the Sri Lankan town which has been the Tamil Tiger’s administrative and political
headquarters and has been captured by the army recently ?
(A) Kandy
(B) Jaffana
(C) Kilinochchi
(D) Mullaithivu
Ans : (C)

20. Which committee recommended abolition of tax rebates under Section, 88 ?


(A) Kelkar Committee
(B) Rangarajan Committee
(C) Chelliah Committee
(D) Shome Committee
Ans : (D)

Andhra Bank Probationary Officers Exam., 2009


(Held on 5-7-2009)
General Awareness : Solved Paper

1. The Govt. of India decided to scrap the import duty on cotton and also withdraw incentives on
the export of the same. How will this action of the govt. affect the cotton market in India ?
1. More and more traders will be able to import cotton as there is no import duty on it.
2. The price of the cotton in domestic market will come down.
3. More and more farmers will no start growing cotton.
(A) Only 1
(B) Only 2
(C) Only 3
(D) All 1, 2 and 3
(E) None of these
Ans : (B)

2. As per the reports in the financial newspapers CCEA cleared many FDI proposals recently.
What is the full form of CCEA ?
(A) Cabinet Committee on Economic Affairs
(B) Congress Committee on Economic Affairs
(C) Cabinet Committee on Emergency Accounts
(D) Clearance Committee on Export Accounts
(E) None of these
Ans : (A)
3. The Vishnugad Pipalkoti Hydel power project is being developed on which of the following
rivers ?
(A) Ganga
(B) Alaknanda
(C) Yamuna
(D) Sabarmati
(E) None of these
Ans : (B)

4. Despite the rampant volatility in the market trends and rising costs all over the world, some
countries were able to grab the business of outsourcing IT and BPO services from some big
countries. Which of the following had been the most favoured nation during 2008-09 for the
same ?
(A) China
(B) South Korea
(C) India
(D) Brazil
(E) None of these
Ans : (C)

5. The facility which allows mobile subscribers to switch from one service provider to another
without changing their mobile number is known as Mobile Number—
(A) Protectability
(B) Portability
(C) Promotability
(D) Permeability
(E) None of these
Ans : (B)

6. Which of the following economies is not in the top 10 economies of the world ?
(A) USA
(B) Japan
(C) Germany
(D) China
(E) South Africa
Ans : (E)

7. Which of the following banks has opened the country’s first ‘Cash Factory’ in Lucknow which
will issue currency notes to all its branches and ATM in that area ?
(A) Bank of India
(B) Bank of Baroda
(C) State Bank of India
(D) Union Bank of India
(E) None of these
Ans : (C)

8. Many times we read in newspapers that the RBI has revised certain rates/ratios under LAF.
What is the full form of LAF ?
(A) Legal Adjustment Facility
(B) Liquidity Adjustment Facility
(C) Longterm Achievement Facility
(D) Legal Adjustment Formality
(E) None of these
Ans : (B)

9. Which of the following statement(s) is/are correct about the Aam Aadmi Bima Yojana
(AABY) launched by the Union Govt. ?
1. The scheme is drawn to include another one crore beneficiaries in the scheme by the end of
September 2009.
2. The scheme was announced to provide insurance cover to those women workers who come for
job under National Rural Employment Guarantee Act.
3. Under the scheme, the central govt. bears the entire amount of the premium which is around
Rs. 6,000 per year for each beneficiary.
(A) Only 1
(B) Only 2
(C) Only 3
(D) All 1, 2 and 3
(E) None of these
Ans : (E)

10. As per the Interim Budget of the Union Govt. for 2009-10, around Rs. 1,31,000 crores were
allocated for various flagship programmes of the Govt. of India. Amongst these, one particular
programme is given at Rs. 41,000 crores as it is aimed at development of infrastructure in the
rural areas of the country. What is that programme ?
(A) Rajiv Gandhi Rural Drinking Water Mission
(B) Rural Employment Guarantee Scheme
(C) Jawaharlal Nehru Urban Renewal Scheme
(D) National Rural Health Mission
(E) None of these
Ans : (E)

11. Who amongst the following was the chairperson of the committee set up for Financial
SectorAssessment
by the Govt. of India and RBI ?
(A) Dr. Rakesh Mohan
(B) Dr. D. Subbarao
(C) Mr. Pranab Mukherjee
(D) Mr. P. Chidambaram
(E) None of these
Ans : (A)
12. As per the reports published in the newspapers the use of credit cards in India has gone down
by about 8% during 2008-09. What are the main reasons for the same ?
1. High interest rate charged by the card companies.
2. Rise in delinquency level related to use of credit cards.
3. Govt. policies which do not allow the launch of new credit cards by the banks.
(A) Only 1
(B) Only 2
(C) Only 3
(D) All 1, 2 and 3
(E) None of these
Ans : (B)

13. As per the reports in the media the exports from India has gone down by 13% in the last few
months. What steps has the Union Govt. taken to boost the same ?
1. Customs duty under export promotion capital goods scheme was cut to 3% from the earlier
5%.
2. Special incentives of Rs. 325 crore were announced for sectors like handmade carpets, leather
and textiles.
3. Excise duty on the goods produced exclusively for exports is withdrawn fully.
(A) Only 1
(B) Only 2
(C) Only 3
(D) Both 1 and 2
(E) All 1, 2, and 3
Ans : (D)

14. Which of the following states in India is at the top of the Investment Index prepared by the
World Bank ?
(A) Gujarat
(B) Maharashtra
(C) Karnataka
(D) Tamil Nadu
(E) None of these
Ans : (C)

15. The G-20 Summit was organized in London on 2nd April 2009. Which of the following
announcement(s) was/were made in the summit ?
1. US $ 1•1 trillion will be provided to restore credit, growth and jobs in the world economy.
2. A wider global regulation will be launched to protect banks from the toxic assets.
3. A Financial Stability Forum will be established with the assistance of IMF.
(A) Only 1
(B) Only 2
(C) Only 3
(D) Both 1 and 2
(E) All 1, 2, and 3
Ans : (A)

16. The Govt. of India recently provided an amount of Rs. 65,300 crores to about 3•6 crore
farmers in India. What was the purpose of the distribution of this much amount to the farmers ?
(A) To enable them to purchase tractors and other hi-tech agricultural machines
(B) This was given to them as a debt waiver and relief package
(C) To provide them fertilizers and advanced high yielding seeds for the next harvesting season
(D) To help them to avail internet services and update their knowledge about the farming
techniques and adopt new methods of the same
(E) None of these
Ans : (B)

17. Which of the following is true about Value Added Tax (VAT) ?
1. It is a consumption tax levied on value added to the product on every stage of its production.
2. VAT is levied by the Union Govt. and only a small part goes to State Govts.
3. The income tax on retailers is now decided on the basis of their VAT collection in a year.
(A) Only 1
(B) Only 2
(C) Only 3
(D) All 1, 2 and 3
(E) None of these
Ans : (A)

18. Mr. Najib Razak is the new Prime Minister of —


(A) Myanmar
(B) Indonesia
(C) Malaysia
(D) Thailand
(E) None of these
Ans : (C)

19. India recently signed an agreement with the European patent office. How would this
agreement be helpful to our country ?
1. It will help India to protect its traditional knowledge of medicines.
2. India’s Traditional Knowledge Library which is in digital form will now be available to the
Patent Examiners at the European Patent Office.
3. India will have the right to grant patents for all its traditional medicines and herbs.
(A) Only 1
(B) Only 2
(C) Only 3
(D) All the three
(E) None of these
Ans : (B)

20. The 21st Arab League Summit was held in which of the following countries in March 2009 ?
(A) Saudi Arabia
(B) Iran
(C) Qatar
(D) UAE
(E) None of these
Ans : (C)

21. India recently got a consignment of 60 tonnes of uranium from which of the following
countries to be used in Rajasthan Atomic Power Station ?
(A) USA
(B) Canada
(C) France
(D) South Korea
(E) China
Ans : (C)

22. India and Chile recently agreed to allow Free Trade between both the countries. What does
it really mean ?
1. India can sell any product to Chile and Chile can also sell any product to India.
2. India and Chile both will sell goods or services to each other but the payment for the same will
be only in return goods. No hard cash will be paid for such transactions.
3. Whatever goods both of them will import or export, the payment will be made only in US
dollars and only cash transactions will take place.
(A) Only 1
(B) Only 2
(C) Only 3
(D) Both 1 and 2
(E) None of these
Ans : (B)

23. Which of the following statement(s) is/are correct about the Global Hunger Index published
recently ?
1. The Index is prepared every year by the United Nations World Food Programme (WFP).
2. As per the report about 27% of world’s undernourished population lives in India.
3. The number of undernourished is more in urban areas than the rural one.
(A) Only 1
(B) Only 2
(C) Only 3
(D) All the three
(E) None of these
Ans : (B)

24. The President’s Rule was imposed in which of the following North-Eastern States in March
2009 ?
(A) Nagaland
(B) Assam
(C) Arunachal Pradesh
(D) Meghalaya
(E) None of these
Ans : (D)

25. Arvind Bhat won the 23rd National Badminton Championship held in Indore by defeating —
(A) Nikhil Kanetkar
(B) P. Gopichand
(C) Nandu Natekar
(D) P. Kashyap
(E) None of these
Ans : (D)

26. Who amongst the following became the first woman pilot in the world to fly MIG-35 fighter
plane ?
(A) Suman Sharma
(B) Saudamini Deshmukh
(C) Kirsty Moore
(D) Nicole Malachowski
(E) None of these
Ans : (A)

27. The 14th Summit of the Association of South-East Asian Nations (ASEAN) was organized in
which of the following countries ?
(A) Vietnam
(B) Myanmar
(C) Indonesia
(D) Thailand
(E) None of these
Ans : (D)

28. India recently signed a Defence Deal with Israel. Which of the following statement(s) is/are
true about the same ?
1. India will purchase a stateof-the-art air defence system from Israel at a whopping cost of US $
1•4 billion.
2. A complex of 5 ordinance plants would be established in Nalanda.
3. Israel will develop a seaborne and shore-based system to protect India from missile attacks on
it.
(A) Only 1
(B) Only 2
(C) Only 3
(D) Both 1 and 3
(E) All 1, 2 and 3
Ans : (A)

29. Mr. Morgan Tsvangirai is the new —


(A) President of Zimbabwe
(B) Prime Minister of Zimbabwe
(C) President of Kenya
(D) Prime Minister of Kenya
(E) None of these
Ans : (B)

30. Which of the following teams won the ICC Women’s World Cup Cricket Finals held in
March 2009 ?
(A) Australia
(B) India
(C) England
(D) New Zealand
(E) None of these
Ans : (C)

31. As we all can notice, the nature of agribusiness in India has changed a lot in the last few
years. This is because the food habits of the people have changed. What are these changes ?
1. The consumption of high value food like fruits, vegetables, fishery and livestock has
increased. Hence, farmers are shifting attention to these.
2. The demand for processed food and ready-to-eat food has increased. Hence, agro business has
become a lucrative option.
3. The use of genetically modified seeds has started giving bumper crops and 60% of the same is
sold as processed food in local and foreign markets.
(A) Only 1
(B) Only 2
(C) Only 3
(D) All the three
(E) None of these
Ans : (B)

32. Which of the following countries is not a member of the Shanghai Cooperation Organisation
which was in news recently ?
(A) Russia
(B) India
(C) China
(D) Kyrgyzstan
(E) Tajikistan
Ans : (B)

33. Under the Project 2011, the All India Football Federation is preparing players for the
matches to be held in the year 2011 for which of the following events ?
(A) Olympics
(B) Asian Cup
(C) FIFA World Cup
(D) National Championship
(E) None of these
Ans : (B)

34. As per reports some state governments in India have put a ban on the sale of Ammonium
Nitrate or its other combinations in the open market as the same is being used by some people as
an explosive. Ban on open sale of such chemicals can be put under which of the following Acts ?
(A) Explosive Substance Act
(B) Narcotics Act
(C) The Trade and Merchandise Act
(D) Dangerous Drugs Act
(E) None of these
Ans : (A)

35. Who amongst the following is the author of the book ‘Flight into Fear’ ?
(A) Capt. Devi Sharan
(B) Admiral Vishnu Bhagwat
(C) Dr. Kiran Bedi
(D) Sri Jaswant Singh
(E) None of these
Ans : (A)

36. Which of the following is an international forum to discuss and implement supervisory norm
in all the banks of the world ?
(A) Basel Committee
(B) UN Charter on Commercial Laws
(C) International Board of Banks and Finance
(D) World Trade Organisation
(E) None of these
Ans : (A)

37. Which of the following states ranked first in implementation of the Central Govt. sponsored
scheme “Artificial Recharge of Ground Water Through dug Wells in Hard Rock Areas” ?
(A) Madhya Pradesh
(B) Maharashtra
(C) Karnataka
(D) Tamil Nadu
(E) None of these
Ans : (B)

38. Which of the following is/are true about the Competition Commission which is set up
recently ?
1. The Commission is established to prevent practices which do not support healthy business
competition.
2. The Commission will have five members including the Chairman.
3. The Commission has to ensure that the financial operations of any business entity does not get
concentrated in the hands of few people.
(A) Only 1
(B) Only 2
(C) Only 3
(D) All the three
(E) None of these
Ans : (D)

39. ‘Anti Tobacco Day’ is observed in India on which one of the following days ?
(A) 11th April
(B) 11th May
(C) 11th June
(D) 31st May
(E) 30th June
Ans : (D)

40. Which of the following teams won the National Women’s Football Championship Final
(Seniors) held in March 2009 ?
(A) Manipur
(B) West Bengal
(C) Tamil Nadu
(D) Maharashtra
(E) None of these
Ans : (A)

41. Who amongst the following was chosen for the award of Hoover Medal 2008 for his/her
contribution to upliftment of poor and rural people ?
(A) Shri Rahul Gandhi
(B) Dr. A. P. J. Abdul Kalam
(C) Mrs. Sonia Gandhi
(D) Dr. Manmohan Singh
(E) None of these
Ans : (B)

42. Who amongst the following is the author of the book ‘Cricket My Style’ ?
(A) Sunil Gavaskar
(B) Ravi Shastri
(C) Kapil Dev
(D) Sachin Tendulkar
(E) Saurav Ganguly
Ans : (C)

43. Many times we read about Foreign Direct Investment in the country for various project
schemes. Why should government promote the idea of Foreign Direct Investment particularly
when it can provide funds from its own resources ?
1. It is a supplement to domestic investments and is preferred to bridge the investment-savings
gap.
2. Foreign Direct Investment helps in providing better and advanced technology which is
sometimes not available domestically.
3. It generates new employment opportunities.
(A) Only 1
(B) Only 2
(C) Only 3
(D) Both 1 and 2
(E) All 1, 2 and 3
Ans : (C)

44. The name of Mr. Jacob Zuma was in news recently as his party has won the general election
in his country with landslide victory. Mr. Zuma is from which country ?
(A) Algeria
(B) Lebanon
(C) South Africa
(D) Venezuela
(E) None of these
Ans : (C)

45. As we all know, lot of people from underdeveloped states migrate to various progressive
states as agricultural labour. What are the major problems of agriculture in India owing to which
people have to leave their own land and work as labourer on somebody else’s farm/land ?
1. Lack of irrigation facilities, low productivity of land and uncertain monsoon.
2. Non-availability of fertilizers, etc. on subsidized rates.
3. Single crop cultivation.
(A) Only 1
(B) Only 2
(C) Only 3
(D) All the three
(E) None of these
Ans : (A)

46. Yuki Bhambri who won an international tournament recently is a famous ………
(A) Badminton player
(B) Chess player
(C) Pistol shooter
(D) Lawn Tennis player
(E) Table Tennis player
Ans : (D)

47. Who amongst the following is not a recipient of the Padma Shri Award given away recently ?
(A) Penaz Masani
(B) Aishwarya Rai Bachchan
(C) Sonam Kapur
(D) Kumar Sanu
(E) M. S. Dhoni
Ans : (C)
48. As we read these days, many world economies are passing through recession at present.
When can an economy be called ‘in recession’ ?
1. When a decline occurs in almost all major economic activities.
2. When the inflation reaches at a very high rate or becomes double digit inflation.
3. When big financial scams and frauds start taking place.
(A) Only 1
(B) Only 2
(C) Only 3
(D) All the three
(E) None of these
Ans : (A)

49. Vyas Samman is given for excellence in the field of ………


(A) Film
(B) Literature
(C) Dance
(D) Music
(E) Science and Technology
Ans : (B)

50. Which of the following is true about the ‘Village Grain Bank Scheme’ ?
1. The Scheme was launched to provide safeguards against starvation during the period of
natural calamities or lean season.
2. The grain banks are set up in food scarce areas like drought prone areas, hot and cold desert
areas, tribal areas and inaccessible hilly areas.
3. Village Panchayats who were running village grain banks earlier are now not authorized to run
the same. Instead the authority is now given to NGOs and SHGs in the village.
(A) Only 1
(B) Only 2
(C) Only 3
(D) Only 1 and 2
(E) All 1, 2 and 3
Ans : (A)

Andhra Bank Marketing Associates, (IInd Shift) Exam., 2009


(Held on 5-7-2009)
English Language : Solved Paper

Directions—(Q. 1–8) Read the following passage carefully and answer the questions given
below it. Certain words/phrases have been printed in bold to help you locate them while
answering some of the questions.
A new analysis has determined that the threat of global warming can still be greatly diminished
if nations cut emissions of heat-trapping greenhouse gases by 70% this century. The analysis was
done by scientists at the National Center for Atmospheric Research (NCAR). While global
temperatures would rise, the most dangerous potential aspects of climate change, including
massive losses of Arctic sea ice and permafrost and significant sea-level rise, could be partially
avoided.

“This research indicates that we can no longer avoid significant warming during this century,”
said NCAR scientist Warren Washington, the study paper’s lead author. “But, if the world were
to implement this level of emission cuts, we could stabilize the threat of climate change”, he
added.

Average global temperatures have warmed by close to 1 degree Celsius since the pre-industrial
era. Much of the warming is due to human-produced emissions of greenhouse gases,
predominantly carbon dioxide. This heat-trapping gas has increased from a preindustrial level
of about 284 parts per million (ppm) in the atmosphere to more than 380 ppm today. With
research showing that additional warming of about 1 degree C may be the threshold for
dangerous climate change, the European Union has called for dramatic cuts in emissions of
carbon dioxide and other greenhouse gases.

To examine the impact of such cuts on the world’s climate, Washington and his colleagues ran a
series of global studies with the NCAR-based Community Climate System Model (CCSM).
They assumed that carbon dioxide levels could be held to 450 ppm at the end of this century. In
contrast, emissions are now on track to reach about 750 ppm by 2100 if unchecked. The team’s
results showed that if carbon dioxide were held to 450 ppm, global temperatures would increase
by 0•6 degrees Celsius above current readings by the end of the century. In contrast, the study
showed that temperatures would rise by almost four times that amount, to 2•2 degrees Celsius
above current readings, if emissions were allowed to continue on their present course. Holding
carbon dioxide levels to 450 ppm would have other impacts, according to the climate modeling
study.

Sea-level rise due to thermal expansion as water temperatures warmed would be 14 centimetres
(about 5•5 inches) instead of 22 centimetres (8•7 inches). Also, Arctic ice in the summertime
would shrink by about a quarter in volume and stabilize by 2100, as opposed to shrinking
atleast three-quarters and continuing to melt, and Arctic warming would be reduced by almost
half.

1. Why has the European Union called for dramatic cuts in carbon dioxide and greenhouse gas
emissions ?
(A) As global warming is not an issue of concern
(B) As the temperatures may rise almost by an additional one degree and this may lead to severe
climate change
(C) As the NCAR has forced the European Union to announce the cuts
(D) As all the nations have decided to cut emissions of carbon dioxide
(E) None of these
Ans : (B)

2. What would not be one of the impacts of cutting greenhouse gas emissions ?
(A) Temperatures will stop soaring
(B) Ice in the Arctic sea would melt at a slower pace
(C) The rise in sea level would be lesser
(D) All of the above would be the impact
(E) None of these
Ans : (D)

3. What would be the impact of holding the carbon dioxide level at 450 ppm at the end of this
century ?
1. Global temperatures would increase by 0•6 degrees Celcius.
2. Arctic warming would be reduced by half.
3. Thermal expansion will stop completely.
(A) Only 1
(B) Only 1 and 2
(C) Only 2 and 3
(D) All the three 1, 2 and 3
(E) None of these
Ans : (A)

4. What does the scientist Warren Washington mean when he says “we could stabilize the threat
of climate change” ?
(A) Climate change can be stopped completely
(B) Climate change can be regularized
(C) Climate change and its effects can be studied extensively
(D) The ill-effects of the change in climate can be minimized
(E) None of these
Ans : (D)

5. Why did Washington and his colleagues conduct a series of studies ?


(A) Because they realized that the temperature increase was almost about 1 degree
(B) So that they could stabilize the climate change
(C) So that they could help the European Union in cutting the carbon dioxide emissions
(D) Because they found out that the greenhouse gas emissions could be cut by 70%
(E) None of these
Ans : (E)

6. What would be the impact of unchecked greenhouse gas and carbon dioxide emissions ?
(A) The temperature would rise from the current temperature by 2•2 degrees Celsius
(B) The sea-level would rise by about 5•5 inches
(C) The arctic ice would stabilize by 2100
(D) The arctic ice would reduce by one-fourth
(E) None of these
Ans : (A)
7. What can be the most appropriate title of the above passage ?
(A) A study of the rise in water level
(B) A study of rise in temperatures
(C) A study of the effects of greenhouse gas emissions
(D) A study of the Arctic region
(E) A study of change in seasons
Ans : (C)

8. Which of the following statements is true in context of the passage ?


(A) At present the carbon dioxide emission is about 284 ppm
(B) The carbon dioxide emissions will be about 450 ppm at the end of this century if unchecked
(C) The carbon dioxide emission was about 380 ppm during the pre-industrial era
(D) The carbon dioxide emissions will be about 750 ppm at the end of this century if unchecked
(E) None of these
Ans : (D)

Directions—(Q. 9–12) Choose the word which is MOST SIMILAR in MEANING to the word
printed in bold as used in the passage.

9. predominantly
(A) clearly
(B) aggressively
(C) mainly
(D) firstly
(E) faintly
Ans : (C)

10. massive
(A) tall
(B) tough
(C) total
(D) little
(E) severe
Ans : (A)

11. shrink
(A) contract
(B) physician
(C) wither
(D) shrivel
(E) reduce
Ans : (E)

12. dramatic
(A) unprecedented
(B) thrilling
(C) spectacular
(D) effective
(E) feeble
Ans : (D)

Directions—(Q. 13–15) Choose the word which is MOST OPPOSITE in MEANING to the
word printed in bold as used in the passage.

13. opposed
(A) resistant
(B) against
(C) favouring
(D) similar
(E) agree
Ans : (D)

14. diminished
(A) created
(B) rose
(C) increased
(D) lessen
(E) finished
Ans : (C)

15. significant
(A) substantial
(B) miniscule
(C) incoherent
(D) unimportant
(E) irrelevant
Ans : (B)

Directions—(Q. 16–20) Which of the phrases (A), (B), (C) and (D) given below each sentence
should replace the phrase printed in bold in the sentence to make it grammatically correct ? If the
sentence is correct as it is given and No Correction is Required, mark (E) as the answer.

16. Naturally, with everything gone so well for them, it was time for celebration.
(A) go so well
(B) going so well
(C) gone as well
(D) going as well
(E) No Correction Required
Ans : (B)

17. The ban was imposed by the state’s commercial taxes department last Friday after protests by
a certain community, which had threat to burn cinema halls screening the controversial movie.
(A) had threats of burning
(B) had threated to burn
(C) had threatened to burn
(D) had threatened to burning
(E) No Correction Required
Ans : (C)

18. Rakesh, an avid football player who captained his team in school and college, will
inaugurate the match tomorrow in Pune.
(A) will be inaugurate
(B) is inauguration
(C) will inaugurating
(D) is inaugurate
(E) No Correction Required
Ans : (E)

19. At a musical night organised for them, the artistic side of the doctors came as forward, as
they sang beautifully and made the evening truly memorable.
(A) come forward
(B) come to the fore
(C) came to the forth
(D) came to the fore
(E) No Correction Required
Ans : (C)

20. Although scared of heights, she gather all her courage and stood atop the 24-storey building
to participate in the activities.
(A) gathered all her courage
(B) gathered all courageous
(C) gather all courageous
(D) is gathered all courage
(E) No Correction Required
Ans : (A)

Directions—(Q. 21–25) Each question below has two blanks, each blank indicating that
something has been omitted. Choose the set of words for each blank which best fits the meaning
of the sentence as a whole.

21. Along with a sharp rise in ……, a recession would eventually result in more men, women
and children living in ……
(A) crime, apathy
(B) fatalities, poor
(C) deaths, slums
(D) unemployment, poverty
(E) migrations streets
Ans : (D)

22. Behaving in a …… and serious way, even in a …… situation, makes people respect you.
(A) calm, difficult
(B) steady, angry
(C) flamboyant, tricky
(D) cool, astounding
(E) silly, sound
Ans : (A)

23. An airplane with …… passengers on board made an unscheduled …… as the airport to


which it was heading was covered with thick fog.
(A) irritable, slip
(B) faulty, stop
(C) variety, halt
(D) tons, wait
(E) numerous, landing
Ans : (C)

24. The government has …… to provide financial aid to the ones …… by severe floods in the
city.
(A) desired, troubled
(B) promised, havoc
(C) failed, affected
(D) wanted, struck
(E) decided, ill
Ans : (C)

25. Deemed universities …… huge fees, but have not been successful in providing ……
education to our students.
(A) collect, maintaining
(B) pay, better
(C) ask, good
(D) charge, quality
(E) demand, quantitative
Ans : (D)

Directions—(Q. 26–30) Rearrange the following six sentences 1, 2, 3, 4, 5 and 6 in the proper
sequence to form a meaningful paragraph; then answer the questions given below them.
1. In turn the buyer is called the franchisee.
2. These two parties are called the franchisor and franchisee.
3. This means that it gives permission for the buyer to use its name and sell its products.
4. He pays money to the franchisor, and agrees to obey the rules the franchisor makes.
5. A franchising agreement includes two parties.
6. The franchisor is the business house/entity which grants the franchisee license.
26. Which of the following should be the LAST (SIXTH) sentence after rearrangement ?
(A) 2
(B) 3
(C) 4
(D) 5
(E) 6
Ans : (C)

27. Which of the following should be the THIRD sentence after rearrangement ?
(A) 5
(B) 6
(C) 1
(D) 4
(E) 2
Ans : (B)

28. Which of the following should be the FOURTH sentence after rearrangement ?
(A) 2
(B) 3
(C) 4
(D) 5
(E) 6
Ans : (B)

29. Which of the following should be the FIRST sentence after rearrangement ?
(A) 1
(B) 2
(C) 3
(D) 4
(E) 5
Ans : (E)

30. Which of the following should be the SECOND sentence after rearrangement ?
(A) 2
(B) 3
(C) 4
(D) 5
(E) 6
Ans : (A)

Directions—(Q. 31–40) Read each sentence to find out whether there is any grammatical error
or idiomatic error in it. The error, if any, will be in one part of the sentence. The letter of that
part is the answer. If there is no error, the answer is (E). (Ignore errors of punctuation, if any).

31. The angry at being (A) / left out of the bonanza (B) / is palpable among (C) / employees of
the organization. (D) No error (E)
Ans : (A)

32. There are just too few trains (A) / for the ever-grow (B) / number of passengers (C) / in the
city. (D) No error (E)
Ans : (B)

33. If all goes well, (A) / the examination scheduled for next month (B) / is all set to be
completely free (C) / from annoying power cuts and disruptions. (D) No error (E)
Ans : (E)

34. His comments came after (A) / the research group said that its (B) / consumer confidence
index were (C) / slumped to its lowest level. (D) No error (E)
Ans : (C)

35. The buzz at the party was (A) / that a famous (B) / filmstar and politician, would (C) /
probable drop by for a while. (D) No error (E)
Ans : (D)

36. The President has denied (A) / that the economy is in recession (B) / or was go into one (C) /
despite a spate of downcast reports. (D) No error (E)
Ans : (C)

37. Aggression in some teenage boys (A) / may be linkage to overly (B) / large glands in their
brains, (C) / a new study has found. (D) No error (E)
Ans : (B)

38. The opposition disrupted proceedings (A) / in both Houses of Parliament (B) / for the second
consecutive day (C) / above the plight of farmers in the country. (D) No error (E)
Ans : (D)

39. In response to the growing crisis (A) / the agency is urgently asking for (B) / more
contributions, to make up for (C) / its sharp decline in purchasing power. (D) No error (E)
Ans : (E)

40. The tennis player easy through (A) / the opening set before her opponent, (B) / rallied to take
the final two sets (C) / for the biggest victory of her young career. (D) No error (E)
Ans : (A)

Directions—(Q. 41–50) In the following passage there are blanks, each of which has been
numbered. These numbers are printed below the passage and against each, five words are
suggested, one of which fits the blank appropriately. Find out the appropriate word in each case.

In economics, the term recession generally describes the reduction of a country’s Gross
Domestic Product (GDP) for atleast two quarters. A recession is …(41)… by rising
unemployment, increase in government borrowing, …(42)… of share and stock prices and
falling investment. All of these characteristics have effects on people. Some recessions have
been anticipated by stock market declines. The real-estate market also usually …(43)… before a
recession. However real-estate declines can last much longer than recessions. During an
economic decline, high …(44)… stocks such as financial services, pharmaceuticals and tobacco
…(45)… to hold up better. However when the economy starts to recover growth, stocks tend to
recover faster. There is significant disagreement about how health care and utilities tend to …
(46)….

In 2008, an economic recession was suggested by several important indicators of economic


downturn. These …(47)… high oil prices, which led to …(48)… high food prices due to a
dependence of food production on petroleum, as well as using food crop products such as ethanol
and biodiesel as an …(49)… to petroleum and global inflation; a substantial credit crisis leading
to the drastic bankruptcy of large and well …(50)… investment banks as well as commercial
banks in various, diverse nations around the world; increased unemployment and signs of
contemporaneous economic downturns in major economies of the world, a global recession.

41. (A) visualized


(B) characterized
(C) imagined
(D) depict
(E) shown
Ans : (B)

42. (A) decrease


(B) abundance
(C) increase
(D) variance
(E) more
Ans : (A)

43. (A) strengthens


(B) volatile
(C) weakens
(D) initiates
(E) awakens
Ans : (C)

44. (A) result


(B) payment
(C) maintained
(D) yield
(E) heavy
Ans : (D)

45. (A) yearn


(B) made
(C) are
(D) want
(E) tend
Ans : (E)

46. (A) increased


(B) fight
(C) distribute
(D) recover
(E) wait
Ans : (D)

47. (A) included


(B) encompass
(C) meant
(D) show
(E) numbered
Ans : (A)

48. (A) healthy


(B) nutritious
(C) fearful
(D) dangerous
(E) abnormally
Ans : (E)

49. (A) element


(B) integral
(C) alternative
(D) variant
(E) substitute
Ans : (C)

50. (A) created


(B) established
(C) wealthy
(D) costly
(E) stand
Ans : (B)

United Bank of India (P.O.) Exam., 2009


(Held on 21-6-2009)
English Language : Solved Paper

Directions—(Q. 1–15) Read the following passage carefully and answer the questions given
below it. Certain words have been printed in bold to help you locate them while answering some
of the questions.

Indeed the western recession is really the beginning of good news for India ! But to understand
that we will have to move away for a while from the topic of western recession ……to the
Japanese recession ! For years the Japanese style of management has been admired. However,
over the last decade or so, one key question has sprung up ‘if Japanese management style is as
wonderful as described then why has Japan been in a recession for more than a decade ?’

The answer to this question is very simple. Culture plays a very important part in shaping up
economics. What succeeds in one culture fails in another. Japanese are basically non-
materialistic. And however rich they become, unlike others, they cannot just keep throwing and
buying endlessly. And once they have everthing they need, there is a saturation point. It was only
when companies like Toyota realized that they cannot keep selling cars endlessly to their home
market that they went really aggressive in the western markets – and the rest is history. Japanese
companies grew bigger by catering to the world markets when their home markets shrank.

And the markets have to shrink finally after attaining a level of affluence ! And that’s great for
the world because earth needs sustainable development. It does not need monstrous consumers
who keep consuming at the cost of the environment and the earth. There should be limits to
growth so that consumers are not converted into material dustbins for the profit of a handful of
corporations.

Owing to the materialistic culture elsewhere, it was possible to keep selling newer products to
the consumers despite having existing ones which served equally well. They were lured through
advertising and marketing techniques of ‘dustbinisation’ of the customer; and then finally, once
they became ready customers, they were given loans and credits to help them buy more and
more. When all the creditworthy people were given loans to a logical limit, they ceased to be a
part of the market. Even this would have been understandable if it could work as an eye opener.
Instead of taking the ‘Right Step’ as Toyota did, they preferred to take a ‘shortcut’. Now banks
went to the non creditworthy people and gave them loans. The people expectedly defaulted and
the entire system collapsed.

Now like Toyota western companies will learn to find new markets. They will now lean towards
India because of its common man ! The billion plus population in the next 25 years will become
a consuming middleclass. Finally, the world’s attention will shift to the developing world.
Finally, there will be a real surge in income of these people and in next fifty odd years, one can
really hope to see an equal world in terms of material plenty, with poverty being almost
nonexistent ! And this will happen not by selling more cars to Americans and Europeans. It will
happen by creating markets in India, China, Latin America and Africa, by giving their people
purchasing power and by making products for them.

The recession has made us realize that it is not because of worse management techniques, but
because of limits to growth. And they will realize that it is great for planet earth. After all, how
many cars and houses must the rich own before calling it enough ? It’s time for them to look at
others as well. Many years back, to increase his own profits, Henry Ford had started paying his
workers more, so that they could buy his cars. In similar fashion, now the developed world will
pay the developing world people so that they can buy their cars and washing machines.

The recession will kick-start the process of making the entire world more prosperous, and lay
the foundation of limits to growth in the west and the foundation of real globalization in the
world – the globalization of prosperity. And one of its first beneficiaries will be India !

1. What does the author mean by the ‘Right Step’ in the passage ?
(A) Giving loans to creditworthy people only
(B) Considering market growth along with environment protection
(C) Restricting people to buy only such products as are needed by them
(D) To start looking at newer avenues and markets
(E) None of these
Ans : (D)

2. Although admired since years, why did the scepticism over the Japanese management style
start since the last decade ?
(A) Japanese companies have been moving out of their home markets since the last decade
(B) Japenese banks have provided loans indiscriminately to the creditworthy as well as non
creditworthy people
(C) Because Japanese markets have been going through a period of continuous recession since
the last decade
(D) The unlimited growth of the Japanese markets has come at the cost of the western market
(E) None of these
Ans : (A)

3. Why does the author foresee the market being created in the developing countries instead of
America and Europe ?
(A) All developing countries have materialistic culture
(B) Developed countries are willing to make an effort to achieve globalization
(C) American and European markets have had a large number of credit defaulters
(D) Recession has not hit the markets of developing countries yet
(E) None of these
Ans : (C)

4. According to the author, what is the main cause of Japanese recession ?


(A) Only a handful of corporations earned profits and not the people in general
(B) Non creditworthy people deafulted which lad to collapse of the entire system
(C) Consumers were sold newer products which were similar in quality to the existing ones
(D) Japanese do not purchase endlessly and thus when products had been sold to every customer,
the markets slowed down
(E) None of these
Ans : (D)

5. How does the author foresee the future globalization as an analogy to Henry Ford’s example ?
1. Car companies would start selling cars in developing countries as well
2. By paying the developing world the developed world would increase its own profit, in turn
bringing affluence to developing world as well.
3. To earn profit, the companies in developing countries would move to foreign land
(A) Only 1
(B) Only 2
(C) Only 3
(D) Only 1 and 3
(E) None of these
Ans : (A)

6. According to the passage, which of the following was NOT an effect of providing loans and
credits to the customers ?
1. The non creditworthy people defaulted.
2. People bought new products which were not needed.
3. Poverty became nonexistent.
(A) Only 1
(B) Only 2
(C) Only 1 and 2
(D) Only 2 and 3
(E) One 3
Ans : (E)

7. Why is recession the beginning of good news for India in the author’s view ?
1. India can provide an attractive market to the western companies.
2. India has remained largely unaffected by recession owing to its huge population.
3. Indians keep purchasing products despite owning equally good products.
(A) Only 3
(B) Only 2
(C) Only 1
(D) Only 2 and 3
(E) None of these
Ans : (E)

8. What does the author mean by ‘Dustbinisation’ of the customer ?


(A) Convincing the customer to buy products he does not need
(B) Denying the non creditworthy people of any loans
(C) Denying more loans to people of any loans
(D) Moving from old customers at the home market to foreign markets
(E) None of these
Ans : (A)

9. Why according to the author is the current recession great for ‘Planet Earth’ ?
1. It will make people nonmaterialistic like the Japanese.
2. The unlimited market growth which caused hazards to the environment would be checked to a
certain extent.
3. Banks will now provide loans only to the creditworthy people.
4. Developing countries will also be benefited by shifted markets.
(A) Only 1
(B) Only 2 and 4
(C) Only 1 and 2
(D) Only 2
(E) None of these
Ans : (B)

Directions—(Q. 10 –17) Choose the word which is MOST SIMILAR in meaning to the word
printed in BOLD as used in the passage.

10. catering
(A) considering
(B) lending
(C) supplying
(D) working
(E) indulging
Ans : (C)

11. key
(A) foundation
(B) solution
(C) requisite
(D) difficult
(E) important
Ans : (E)

12. aggressive
(A) violent
(B) determined
(C) demanding
(D) offensive
(E) brutish
Ans : (D)

Directions—(Q. 13 –15) Choose the word/phrase which is MOST OPPOSITE in meaning to


the word printed in BOLD as used in the passage.

13. prosperous
(A) distressed
(B) helpless
(C) worse
(D) worthless
(E) underprivileged
Ans : (A)
14. consuming
(A) destroying
(B) exhausting
(C) greedy
(D) curtailing
(E) spending
Ans : (D)

15. surge
(A) decrease
(B) deteriorating
(C) weakening
(D) atrophy
(E) crumble
Ans : (A)

Directions—(Q. 16 – 20) Read each sentence to find out whether there is any grammatical error
in it. The error if any will be in one part of the sentence, the letter of that part will be the answer.
If there is no error, mark (E) as the answer. (Ignore errors of punctuation, if any.)

16. To be a king and (A) / wear a crown are (B) / more glamorous to (C) / see than to bear. (D)
No error (E)
Ans : (B)

17. None of the student (A) / in the class (B) / scored below the (C) / given cut-off marks. (D) No
error (E)
Ans : (A)

18. Ashok is among the (A) / few people in the world (B) / which did not blindly follow (C) / the
path of others. (D) No error (E)
Ans : (C)

19. Most people like to (A) / rest after a day’s hard work (B) / but he seemed to have (C) / an
inexhaustive supply of energy. (D) No error (E)
Ans : (C)

20. Ancient artefacts are (A) / a part of global heritage (B) / and should not be (C) / sold to the
highest bidder. (D) No error (E)
Ans : (E)

Directions—(Q. 21–25) Which of the phrases (A), (B), (C) and (D) given below each statement
should replace the phrase printed in bold in the sentence to make it grammatically correct ? If the
sentence is correct as it is given and no correction is required, mark (E) as the answer.

21. Alcohol in moderate quantity boosts concentration of good cholesterol and inhibiting blood
clots.
(A) inhibits blood clots
(B) inhibit blood clots
(C) inhibited blood clots
(D) inhabiting blood clots
(E) No correction required
Ans : (B)

22. One of the main function of the State is maintenance of law and order.
(A) main function for
(B) main function of
(C) main functions for
(D) main functions off
(E) No correction required
Ans : (B)

23. We must realize that learning from mistakes is an important part of life.
(A) mistakes are an
(B) mistakes are a
(C) mistake are a
(D) mistakes has an
(E) No correction required
Ans : (E)

24. The sword of Tipu Sultan was recently brought at an auction by an Indian for Rs. 2 crores.
(A) brought in a
(B) brought in an
(C) bought in an
(D) bought at a
(E) No correction required
Ans : (C)

25. Setbacks and failures has always been an integral part of science.
(A) has always being
(B) were always been
(C) has been always
(D) have always been
(E) No correction required
Ans : (D)

Directions—(Q. 26–30) Rearrange the following sentences (1), (2), (3), (4), (5) and (6) to make
a meaningful paragraph and then answer the questions which follow—
(1) However while reading they would not know when to pause and what to emphasize.
(2) Since then their use has been regularized and the punctuation rules have been followed by all.
(3) In earlier days, people learnt by reading out loud.
(4) But not everybody used the same punctuations for the same thing.
(5) To address this problem, various signs depicting various punctuations were introduced.
(6) Thus firmer guidelines regarding punctuations were framed so that everyone used them in
similar way.

26. Which of the following sentence should be the SECOND after rearrangement ?
(A) (1)
(B) (2)
(C) (4)
(D) (5)
(E) (6)
Ans : (A)

27. Which of the following sentence should be the THIRD after rearrangement ?
(A) (1)
(B) (5)
(C) (4)
(D) (6)
(E) (3)
Ans : (B)

28. Which of the following sentence should be the FIFTH after rearrangement ?
(A) (2)
(B) (3)
(C) (1)
(D) (5)
(E) (6)
Ans : (A)

29. Which of the following sentence should be the SIXTH (LAST) after rearrangment ?
(A) (3)
(B) (5)
(C) (4)
(D) (2)
(E) (6)
Ans : (E)

30. Which of the following sentence should be the FIRST after rearrangement ?
(A) (1)
(B) (2)
(C) (3)
(D) (4)
(E) (5)
Ans : (C)

Directions—(Q. 31 – 45) Each question below has two blanks, each blank indicating that
something has been omitted. Choose the set of words for each blank that best fits the meaning of
the sentence as a whole.

31. Many teachers ……… the lack of professional freedom as the ……… for leaving the job.
(A) cited, reason
(B) explained, force
(C) claimed, understanding
(D) argued, culprit
(E) believe, ground
Ans : (A)

32. Skeptics would not ……… that the earth actually moves, let alone that it ……… around the
sun.
(A) permit, orbits
(B) accept, revolves
(C) experience, circles
(D) assume, went
(E) challenge, spins
Ans : (B)

33. Unpredictable …… of the child could not lead the consultants to any ………
(A) performance, setting
(B) belief, judgement
(C) operation, purpose
(D) behaviour, conclusion
(E) react, decision
Ans : (D)

34. A public servant who is guilty will not ……… punishment and no …… person will be
punished.
(A) be sincere
(B) flee, guilty
(C) defend, common
(D) avoid, uninformed
(E) escape, innocent
Ans : (E)

35. Few professions can ……… the sheer variety and constant ……… of being a doctor.
(A) like, struggle
(B) share, enthusiast
(C) match, challenge
(D) draw, work-load
(E) justify, exception
Ans : (C)

Directions—(Q. 36– 40) In each of the following questions four words are given of which two
words are most nearly the same or opposite in meaning. Find the two words which are most
nearly the same or opposite in meaning and indicate the letter of the correct letter combination.

36. (1) unite


(2) association
(3) separate
(4) distant
(A) 1 – 3
(B) 1 – 2
(C) 2 – 3
(D) 2 – 4
(E) 1 – 4
Ans : (A)

37. (1) explicit


(2) cautious
(3) introvert
(4) clear
(A) 1 – 2
(B) 2 – 4
(C) 1 – 3
(D) 1 – 4
(E) 3 – 4
Ans : (D)

38. (1) fearful


(2) beautiful
(3) hostile
(4) amicable
(A) 2 – 4
(B) 3 – 4
(C) 1 – 2
(D) 2 – 3
(E) 1 – 4
Ans : (B)

39. (1) fraud


(2) barbarian
(3) guilty
(4) civilized
(A) 1 – 3
(B) 1 – 2
(C) 2 – 4
(D) 1 – 4
(E) 3 – 4
Ans : (C)
40. (1) loud
(2) prominent
(3) salient
(4) legible
(A) 1 – 3
(B) 2 – 4
(C) 3 – 4
(D) 1 – 4
(E) 2 – 3
Ans : (E)

Directions—(Q. 41–50) In the following passage there are blanks, each of which has been
numbered. These numbers are printed below the passage and against each, five words are
suggested, one of which best fits the blank appropriately. Find out the appropriate word in each
case.

Twenty years …(41)… now, nearly 60% of the world’s population will live in urban areas. The
impact of urbanization might not all be positive on India as urban expansion is happening at a
much …(42)… rate than infrastructure expansion.

Sustainability issues need to be …(43)… so that economic development is not at the …(44)… of
public health. Some urban services that ought to be in …(45)… in a city like water, electricity,
transport etc. need special consideration.

TERI has put together a detailed report that …(46)… sustainability in the provision of basic
urban services in Indian cities.

…(47)… public transport is a major reason for the proliferation of private vehicles on the road.
Respiratory illness in children living in urban areas is on the …(48)… with more cases of
Ashthma being …(49)… because of pollution. The future of cities of Indian dreams depends on
…(50)… we can build better cities today.

41. (A) till


(B) since
(C) from
(D) after
(E) on
Ans : (C)

42. (A) faster


(B) slower
(C) changed
(D) speed
(E) quick
Ans : (A)
43. (A) speculated
(B) believed
(C) imagined
(D) considered
(E) understand
Ans : (D)

44. (A) payment


(B) rate
(C) cost
(D) charge
(E) expense
Ans : (C)

45. (A) abundance


(B) large
(C) functional
(D) vicinity
(E) location
Ans : (A)

46. (A) bring


(B) emphasizes
(C) speculates
(D) postulates
(E) requests
Ans : (B)

47. (A) good


(B) competent
(C) absence
(D) inadequate
(E) sufficient
Ans : (D)

48. (A) multiplication


(B) expansion
(C) rise
(D) inflation
(E) grow
Ans : (C)

49. (A) produced


(B) develop
(C) composed
(D) resulted
(E) reported
Ans : (E)

50. (A) if
(B) whether
(C) unless
(D) provided
(E) weather
Ans : (B)

Nabard Bank Officers Exam., 2009


(Held on 29-3-2009)
Reasoning : Solved Paper
1. In a certain code DESPAIR is written as TFEQSJB. How is NUMERAL written in that code ?
(A) OVNFMBS
(B) NVOFSBN
(C) NVOMFBS
(D) NVOFMBS
(E) None of these
Ans : (D)

2. If it is possible to make only one meaningful word with the first, second, sixth and tenth letters
of the word DISCLAIMER, which of the following will be the third letter from the left ? If no
such word can be made, give ‘X’ as your answer and if more than one such words can be made,
give ‘Y’ as the answer.
(A) I
(B) R
(C) D
(D) X
(E) Y
Ans : (A)

3. How many such digits are there in the number 27561493 each of which is as far away from the
beginning of the number as when the digits are arranged in descending order within the number ?
(A) None
(B) One
(C) Two
(D) Three
(E) More than three
Ans : (B)

4. If ‘yellow’ is called ‘red’, ‘red’ is called ‘blue’, ‘blue’ is called ‘white’, ‘white’ is called
‘black’, ‘black’ is called ‘green’, and ‘green’ is called ‘violet’, what is the colour of clear sky ?
(A) Green
(B) Violet
(C) Yellow
(D) Red
(E) None of these
Ans : (E)

5. It was 9•35 a.m. in Rakhi’s watch, which kept correct time, when Reena informed her that the
last bus left the bus stop at 9•25 a.m. Reena’s watch is 5 minutes fast. The frequency of the bus is
every 20 minutes. For how long Rakhi must wait to catch the next bus ?
(A) 5 minutes
(B) 10 minutes
(C) 15 minutes
(D) 25 minutes
(E) None of these
Ans : (A)

6. Which of the following should come next in the given sequence of numbers ?
243243124312243123243123
(A) 2
(B) 3
(C) 4
(D) 5
(E) None of these
Ans : (C)

7. How many meaningful English words can be formed with the letters EDOM using each letter
only once in each word ?
(A) None
(B) One
(C) Two
(D) Three
(E) More than three
Ans : (C)

8. Four of the following five are alike in a certain way and so form a group. Which is the one that
does not belong to that group ?
(A) Nephew
(B) Cousin
(C) Mother
(D) Brother
(E) Sister
Ans : (B)

9. How many pairs of letters are there in the word SECURITY each of which has as many
alphabets between them as there are in the English alphabetical series ?
(A) None
(B) Two
(C) Three
(D) Four
(E) More than four
Ans : (C)

10. In a certain code STAR is written as 5$*2 and TORE is written as $32@. How is OATS
written in that code ?
(A) 3*5$
(B) 3*$5
(C) 3$*5
(D) 35*$
(E) None of these
Ans : (B)

Directions—(Q. 11–15) In each of the questions below are given four statements followed by
four conclusions numbered I, II, III and IV. You have to take the given statements to be true
even if they seem to be at variance from commonly known facts. Read all the conclusions and
then decide which of the given conclusions logically follows from the given statements
disregarding commonly known facts.

11. Statements : Some doctors are lawyers. All teachers are lawyers. Some engineers are
lawyers. All engineers are businessmen.
Conclusions :
I. Some teachers are doctors.
II. Some businessmen are lawyers.
III. Some businessmen are teachers.
IV. Some lawyers are teachers.
(A) None follows
(B) Only II follows
(C) Only III follows
(D) Only II and IV follow
(E) None of these
Ans : (D)

12. Statements : All plastics are glasses. Some sponges are glasses. All sponges are clothes. All
clothes are liquids.
Conclusions :
I. All liquids are sponges.
II. Some plastics are clothes
III. All glasses are plastics
IV. All liquids are clothes.
(A) None follows
(B) Only either II or IV follows
(C) Only IV follows
(D) Only III and IV follow
(E) None of these
Ans : (A)

13. Statements : All sands are beaches. All shores are beaches. Some beaches are trees. All trees
are hotels.
Conclusions :
I. Some shores are hotels.
II. All beaches are shores.
III. Some beaches are hotels.
IV. Some sands are trees.
(A) None follows
(B) Only II follows
(C) Only either I or III follows
(D) Only IV follows
(E) None of these
Ans : (E)

14. Statements : All parrots are pigeons. Some crows are pigeons. Some sparrows are crows. All
sparrows are koels.
Conclusions :
I. Some koels are crows.
II. Some parrots are crows.
III. Some sparrows are pigeons.
IV. No crow is a parrot.
(A) Only I follows
(B) Only III follows
(C) Only I and either II or IV follow
(D) Only either I or III follows
(E) None of these
Ans : (C)

15. Statements : All chairs are tables. All tables are cushions. Some cushions are trolleys. All
trolleys are lamps.
Conclusions :
I. Some lamps are tables.
II. Some trolleys are chairs.
III. Some cushions are lamps.
IV. All chairs are cushions.
(A) Only I follows
(B) Only III and IV follow
(C) Only either I or II follows
(D) All follow
(E) None of these
Ans : (B)
Directions—(Q. 16–20) Study the following information carefully and answer the given
questions—

A, B, C, D, E, F, G and H are eight friends sitting around a circular table facing the centre. A sits
second to the left of D who is third to the left of E. C sits third to the right of G who is not an
immediate neighbour of E.
H sits third to the right of B who sits second of the right of G.

16. Who sits between D and C ?


(A) Only B
(B) Only C and A
(C) Only G
(D) Only E
(E) Only G and E
Ans : (A)

17. Who sits second to the right of E ?


(A) B
(B) F
(C) G
(D) C
(E) None of these
Ans : (B)

18. What is the position of A with respect to H ?


(A) Third to the left
(B) Third to the right
(C) Second to the left
(D) Second to the right
(E) Fourth to the left
Ans : (D)

19. Four of the following five are alike based upon their seating arrangements and so form a
group. Which is the one that does not belong to that group ?
(A) CH
(B) FG
(C) DA
(D) BE
(E) GB
Ans : (C)

20. Which of the following pairs has the second person sitting to the immediate left of the first
person ?
(A) DB
(B) EH
(C) FA
(D) GD
(E) None of these
Ans : (E)

Directions—(Q. 21–25) These questions are based on the following letter/number/symbol


arrangement. Study it carefully and answer the questions.
8#B2©EK7%5A1$GD94U*C6H@I3

21. How many such numbers are there in the above arrangement each of which is immediately
preceded by a consonant and also followed by a symbol ?
(A) None
(B) One
(C) Two
(D) Three
(E) More than three
Ans : (D)

22. Which of the following should replace the question mark in the following series based on the
above arrangement ?
2EK, %A1, G94, ?
(A) *6H
(B) UC6
(C) *CH
(D) *6@
(E) None of these
Ans : (A)

23. Which of the following is the fifth to the left of seventeenth from the left end of the
arrangement ?
(A) $
(B) G
(C) A
(D) D
(E) None of these
Ans : (E)

24. If all the numbers are deleted from the above arrangement, which of the following will be the
twelfth from the right end ?
(A) %
(B) K
(C) *
(D) C
(E) None of these
Ans : (B)
25. How many such symbols are there in the above arrangement each of which is immediately
preceded by a number and also immediately followed by a letter ?
(A) None
(B) One
(C) Two
(D) Three
(E) More than three
Ans : (D)

Directions—(Q. 26–30) Read the following information carefully and answer the questions,
which follow :
‘P ÷ Q’ means ‘P is son of Q’.
Q’ ‘P × means ‘P is sister of Q’.
‘P + Q’ means ‘P is brother of Q’.
‘P – Q’ means ‘P is mother of Q’.

26. How is T related to S in the R + V ÷ S’ ? expression ‘T ×


(A) Sister
(B) Mother
(C) Aunt
(D) Uncle
(E) None of these
Ans : (E)

27. How is T related to S R ÷ V – S’ ? in the expression ‘T ×


(A) Father
(B) Sister
(C) Daughter
(D) Aunt
(E) None of these
Ans : (B)

28. How is S related to T in the expression ‘T + R – V + S’ ?


(A) Uncle
(B) Nephew
(C) Son
(D) Cannot be determined
(E) None of these
Ans : (D)

29. Which of the following means that ‘S is the husband of T’ ?


(A) T × R – V + S
(B) T – R ÷ V × S
(C) T – R + V ÷ S
(D) T ÷ R × V + S
(E) None of these
Ans : (C)

30. How is V related to T in the expression ‘T ÷ R + V × S’ ?


(A) Aunt
(B) Nephew
(C) Niece
(D) Uncle
(E) None of these
Ans : (A)

Directions—(Q. 31–35) Each of the questions below consists of a question and two statements
numbered I and II are given below it. You have to decide whether the data provided in the
statements are sufficient to answer the question. Read both the statements and Give answer—

(A) If the data in Statement I alone are sufficient to answer the question, while the data in
Statement II alone are not sufficient to answer the question.
(B) If the data in Statement II alone are sufficient to answer the question, while the data in
Statement I alone are not sufficient to answer the question.
(C) If the data in Statement I alone or in Statement II alone are sufficient to answer the question.
(D) If the data in both the Statements I and II are not sufficient to answer the question.
(E) If the data in both the Statements I and II together are necessary to answer the question.

31. How is A related to B ?


I. A is sister-in-law of C who is the daughter-in-law of B who is the wife of D.
II. B is the mother of A’s son’s only uncle’s son.
Ans : (B)

32. Amongst A, B, C, D, E and F each are having a different height, who is the shortest ?
I. C is shorter than only B.
II. A is taller than only D and F.
Ans : (D)

33. Point X is in which direction with respect to Y ?


I. Point Z is at equal distance from both point X and point Y.
II. Walking 5 km to the East of point X and taking two consecutive right turns after walking 5
kms before each turn leads to point Y.
Ans : (D)

34. How is ‘must’ written in a code language ?


I. ‘you must see’ is written as ‘la pa ni’ and ‘did you see’ is written as ‘jo ni pa’ in that code
language.
II. ‘you did that’ is written as ‘pa si jo’ in that code language.
Ans : (A)

35. On which day of the week does Arti’s birthday fall ?


I. Sonu correctly remembers that Arti’s birthday falls after Wednesday but before Sunday.
II. Raj correctly remembers that Arti’s birthday falls before Friday but after Tuesday.
Ans : (E)

Directions—(Q. 36–40) Below in each question are given two statements I and II. These
statements may be either independent causes or may be effects of independent causes or a
common cause. One of these statements may be the effect of the other statement. Read both the
statements and decide which of the following answer choice correctly depicts the relationship
between these two statements.
Mark answer—
(A) If statement I is the cause and statement II is its effect.
(B) If statement II is the cause and statement I is its effect.
(C) If both the statements I and II are independent causes.
(D) If both the statements I and II are effects of independent causes.
(E) If both the statements I and II are effects of some common cause.

36. I. Computer education has been made compulsory for all the classes by many schools.
II. The current job market prefers computer literate workforce.
Ans : (B)

37. I. The standard of education in evening colleges of the State has been deteriorating.
II. The standard of school education has been fast deteriorating in the State.
Ans : (E)

38. I. All domestic airlines increased the fares in all sectors with immediate effect.
II. Railways increased the fare of all its classes with immediate effect.
Ans : (E)

39. I. The prices of fruits and vegetables fell substantially over the last few days.
II. The quality of fruits and vegetables improved considerably over the last few days.
Ans : (D)

40. I. Recent floods in the area changed the nutritional contents of the soil.
II. Farmers in the area switched over to cultivating rice instead of wheat.
Ans : (A)

Directions—(Q. 41–45) In the following questions, the symbols #, $, %, * and @ are used with
the following meaning as illustrated below :

‘A # B’ means ‘A is not greater than B’.


‘A $ B’ means ‘A is neither smaller nor equal to B’.
‘A % B’ means ‘A is neither smaller nor greater than B’.
‘A * B’ means ‘A is neither greater nor equal to B’.
‘A @ B’ means ‘A is not smaller than B’.

Now in each of the following questions assuming the given statements to be true, find which of
the two conclusions I and II given below them is/are definitely true and give your answer
accordingly. Mark answer—
(A) If only conclusion I is true.
(B) If only conclusion II is true.
(C) If either conclusion I or II is true.
(D) If neither conclusion I nor II is true.
(E) If both conclusion I and II are true.

41. Statements :
A # B, B % C, C * D
Conclusions :
I. C @ A
II. A # D
Ans : (A)

42. Statements :
P $ Q, Q @ S, S % R
Conclusions :
I. P @ R
II. R $ Q
Ans : (D)

43. Statements :
W * X, X # Y, Y $ Z
Conclusions :
I. W * Z
II. W @ Z
Ans : (C)

44. Statements :
G @ H, H $ J, J % K
Conclusions :
I. K * G
II. J * G
Ans : (E)

45. Statements :
N @ M, M $ P, P # T
Conclusions :
I. T # N
II. P * N
Ans : (B)

Directions—(Q. 46–50) Study the following information carefully and answer the given
questions :

A, B, C, D, E, F and G are seven friends studying seven different branches of engineering,


namely Mechanical, Chemical, Electrical, Electronics, Civil, Computer and Aeronautical
Engineering, not necessarily in this order. Each of them studies in three different colleges, X, Y
and Z. Not less than two study in any college. D studies Electrical engineering in College X. The
one who studies Chemical Engineering does not study in college Z. F studies Aeronautical
engineering in college Y with only B. A does not study in college X and does not study Civil
engineering. E studies Computer engineering and does not study in college X. G studies
Electronics engineering but not in college X. None in college X studies Mechanical or Civil
engineering.

46. Which of the following groups represents the persons studying in college Z ?
(A) D, B
(B) C, E, G
(C) A, G
(D) G, E, A
(E) None of these
Ans : (D)

47. In which of the following colleges does C study ?


(A) X
(B) Y
(C) Z
(D) Either X or Z
(E) Cannot be determined
Ans : (A)

48. Which of the following combinations is correct ?


(A) A—Civil—Z
(B) B—Chemical—Y
(C) C—Chemical—Z
(D) G—Electronics—Y
(E) None of these
Ans : (E)

49. B studies in which of the following branches of engineering ?


(A) Chemical
(B) Mechanical
(C) Civil
(D) Cannot eb determined
(E) None of these
Ans : (C)

50. Who studies Chemical engineering ?


(A) B
(B) C
(C) E
(D) A
(E) None of these
Ans : (B)

Bank of Baroda Agriculture Officers Exam., 2008


(Held on 14-12-2008)
Reasoning : Solved Paper
1. In a certain code SOUTHERN is written as UVPTMQDG. How is MARIGOLD written in
that code?
(A) JSBCNFKS
(B) JSBNHPME
(C) JSBNCKNF
(D) NBSKCJNF
(E) None of these
Ans : (C)

2. Four of the following five are alike in a certain way and so form a group. Which is the one that
does not belong to the group ?
(A) Tornado
(B) Volcano
(C) Storm
(D) Hurricane
(E) Cyclone
Ans : (B)

3. How many such pairs of letters are there in the word EVAPORATE each of which has as
many letters between them in the word as in the English alphabet ?
(A) One
(B) Two
(C) Three
(D) Four
(E) More than four
Ans : (E)

4. How many meaningful four letter English words can be formed with the letters ITED using
each letter only once in each word ?
(A) None
(B) One
(C) Two
(D) Three
(E) More than three
Ans : (E)

Directions—(Q. 5–6) Study the following information carefully to answer these questions.
A vehicle starts from point P and runs 10 kms. towards North. It takes a right turn and runs 15
kms. It now runs 6 kms. after taking a left turn. It finally takes a left turn, runs 15 kms. and stops
at point Q.

5. How far is point Q with respect to point P ?


(A) 16 km
(B) 25 km
(C) 4 km
(D) 0 km
(E) None of these
Ans : (A)

6. Towards which direction was the vehicle moving before it stopped at point Q ?
(A) North
(B) East
(C) South
(D) West
(E) North-West
Ans : (D)

7. If each of the consonants of the word PURCHASED is substituted by the letter preceding it
and each vowel is substituted by the letter following it, which of the following letters would have
the same letter on either sides ?
(A) B
(B) G
(C) F
(D) Q
(E) None of these
Ans : (B)

8. If all the letters in the word NEIGHBOURS are rearranged as they appear in the English
alphabetical order, the position of how many letters will remain unchanged after the
rearrangement ?
(A) None
(B) One
(C) Two
(D) Three
(E) More than three
Ans : (C)

9. Asha ranks 16th from the top and 15th from the bottom in an examination. How many
students are there in the class ?
(A) 30
(B) 31
(C) 32
(D) Cannot be determined
(E) None of these
Ans : (A)

10. Five students participated in an examination and each scored different marks. Nidhi scored
higher than Mamta. Kavita scored lower than Prashant but higher than Nidhi. Anil’s score was
between Mamta and Nidhi. Which of the following pairs represents the highest and the lowest
scorers respectively ?
(A) Nidhi, Kavita
(B) Kavita, Mamta
(C) Anil, Kavita
(D) Prashant, Mamta
(E) None of these
Ans : (D)

Directions—(Q. 11–15) In each of these questions a group of letters is given followed by four
combinations of digit/symbol lettered (A), (B), (C) and (D). Letters are to be coded as per the
scheme and conditions given
below. You have to find out the serial letter of the combination, which represents the letter
group. Serial letter of that combination is your answer. If none of the combinations is correct,
your answer is (E) i.e. ‘None of these’.
Letter :
ERJLNAQDPUBFIKTH
Digit/Symbol :
*2©96#51%$£4@738
Conditions :
(i) If the first letter is followed by a vowel and the last letter is preceded by a consonant, the last
letter is to be coded as the code for vowel following the first letter.
(ii) If there are no vowels in the group of letters the codes for the fifth letter from the right and
the fifth letter from the left are to be interchanged.
(iii) If the group of letters contains more than one vowel, the codes for both the vowels are to be
interchanged.

11. PNEBJI
(A) %6@£©*
(B) %6*@£©
(C) %6*£©@
(D) @%£©6*
(E) None of these
Ans : (A)

12. FABLNK
(A) #4£96#
(B) £4#96#
(C) 4#£96#
(D) 49#6#£
(E) None of these
Ans : (C)

13. LPKNJB
(A) 9%£67©
(B) 9©76%£
(C) 9%76©£
(D) 96©7%£
(E) None of these
Ans : (B)

14. DTQKNH
(A) 135768
(B) 137568
(C) 138765
(D) 165783
(E) None of these
Ans : (E)

15. TNAPUB
(A) 36%$#£
(B) 36#%$£
(C) 63#%$£
(D) 36$%#£
(E) None of these
Ans : (D)

Directions—(Q. 16-20) In each of the questions below are given four statements followed by
four conclusions numbered I, II, III and IV. You have to take the given statements to be true
even if they seem to be at variance from commonly known facts. Read all the conclusions and
then decide which of the given conclusions logically follows from the given statements
disregarding commonly known facts.

16. Statements :
Some jeeps are trains.
All trains are buses.
Some boats are jeeps.
Some scooters are buses.
Conclusions :
I. Some scooters are trains.
II. Some bots are buses.
III. Some jeeps are scooters.
IV. All buses are trains.
(A) None follows
(B) Only IV follows
(C) Only II and IV follow
(D) Only III follows
(E) None of these
Ans : (A)

17. Statements :
All teachers are engineers.
All engineers are cooks.
Some cooks are merchants.
All merchants are poets.
Conclusions :
I. Some cooks are teachers.
II. Some merchants are engineers.
III. All cooks are engineers.
IV. Some cooks are poets.
(A) None follows
(B) Only I follows
(C) Only II and IV follow
(D) Only I and IV follow
(E) None of these
Ans : (D)

18. Statements :
Some tools are hammers.
Some hammers are nails.
All nails are screws.
All screws are nuts.
Conclusions :
I. All nuts are screws.
II. Some nuts are tools.
III. Some hammers are screws.
IV. All nuts are nails.
(A) All follow
(B) Only I follows
(C) Only II follows
(D) Only II and III follow
(E) None of these
Ans : (E)

19. Statements :
All pens are bags.
All bags are glasses.
No glass is a spoon.
All spoons are books.
Conclusions :
I. Some glasses are pens.
II. Some books are bags.
III. No spoon is a pen.
IV. No bag is a book.
(A) Only II and III follow
(B) Only I, III and either II or IV follow
(C) Either II or IV follows
(D) All follow
(E) None of these
Ans : (B)

20. Statements :
All petals are flowers.
All thorns are flowers.
Some leaves are thorns.
Some stems are flowers.
Conclusions :
I. Some petals are leaves.
II. All leaves are flowers.
III. Some stems are petals.
IV. No petal is a leaf.
(A) None follows
(B) Only II follows
(C) Only II and either I or IV follow
(D) Only either I or IV follows
(E) None of these
Ans : (D)

Directions—(Q. 21-25) Study the sets of numbers given below and answer the questions, which
follow—
129 642 921 476 308

21. Which of the following numbers will be obtained if the second digit of the lowest number is
added to third digit of the highest number after adding 4 to each number ?
(A) 5
(B) 6
(C) 7
(D) 8
(E) None of these
Ans : (D)

22. If the digits in each of the five numbers are arranged in descending order, the position of how
many numbers will not change ?
(A) None
(B) One
(C) Two
(D) Three
(E) Four
Ans : (C)
23. Based upon the given set of numbers, four of the following five are alike in a certain way and
so form a group. Which is the one that does not belong to that group ?
(A) 647
(B) 264
(C) 912
(D) 192
(E) 380
Ans : (E)

24. If in each number, the digits are arranged in ascending order, what will be sum of middle
digits of each of the numbers ?
(A) 14
(B) 12
(C) 26
(D) 9
(E) None of these
Ans : (E)

25. If in each number, the first and the last digits are interchanged, which will be the second
lowest number ?
(A) 129
(B) 642
(C) 921
(D) 476
(E) 308
Ans : (B)

Directions—(Q. 26-30) These questions are based on the following letter/number/symbol


arranement. Study it carefully and answer the questions.
@BDS3WA*V69QJ%H8UNI2T4#75$K

26. How many such symbols are there in the above arrangement each of which is immediately
preceded by a vowel and also immediately followed by a number ?
(A) None
(B) One
(C) Two
(D) Three
(E) None of these
Ans : (A)

27. Four of the following are alike in a certain way based upon their positions in the above
arrangement and so form a group. Which is the one that does not belong to that group ?
(A) BD3
(B) %HU
(C) *V9
(D) T45
(E) S3A
Ans : (D)

28. Which of the following is exactly in the middle of the 17th from the right and 19th from the
left end ?
(A) %
(B) H
(C) J
(D) #
(E) None of these
Ans : (B)

29. What should come in the place of the question mark (?) in the following series based on the
above arrangement ?
*6Q9JH%8NUIT?
(A) N
(B) 4
(C) #
(D) 7
(E) None of these
Ans : (E)

30. How many consonants are there in the above arrangement each of which is immediately
preceded by a number but not immediately followed by another consonant ?
(A) None
(B) One
(C) Two
(D) Three
(E) None of these
Ans : (C)

Directions—(Q. 31-35) Study the following information carefully to answer these questions.

A, B, C, D, E, F, G and H are sitting around a circle, facing the centre. F sits to the immediate
right of D and third to the left of A. G sits third to the left of D who does not sit next to E. B sits
next to G but not next to D. C does not sit next to either D or A.

31. Who sits to the immediate left of a ?


(A) E
(B) F
(C) G
(D) H
(E) None of these
Ans : (A)
32. What is the position of H with respect to C ?
(A) Second to the left
(B) First to the right
(C) Third to the right
(D) Second to the right
(E) None of these
Ans : (E)

33. Which of the following pairs sits between G and D ?


(A) AC
(B) CF
(C) HB
(D) FA
(E) None of these
Ans : (C)

34. Starting from A’s position, if all the eight are arranged in alphabetical order in clockwise
direction, the seating position of which of the following (excluding A) would not change ?
(A) B
(B) C
(C) D
(D) H
(E) None of these
Ans : (B)

35. Four of the following are alike based upon their seating arrangement around the circle, which
is the one that does not belong to that group ?
(A) FH
(B) GE
(C) CD
(D) BG
(E) EF
Ans : (D)

Directions—(Q. 36-40) Read the following information carefully and answers the questions
which follow—
‘A × B’ means ‘A is brother of B’
‘A – B’ means ‘A is daughter of B’.
‘A ÷ B’ means ‘A is wife of B’.
‘A + B’ means ‘A is son of B’.

36. How is T related to P in the expression ‘P + Q – R ÷ T’ ?


(A) Maternal grandmother
(B) Maternal grandfather
(C) Paternal grandmother
(D) Grandson
(E) None of these
Ans : (B)

37. Which of the following means ‘P is sister of S’ ?


(A) P + Q ÷ R – S
(B) P + Q ÷ R ×S
(C) P × Q – R + S
(D) P ÷ Q + R × S
(E) None of these
Ans : (E)

38. How is P R + Q ÷ P’ ? related to S in the expression ‘S ×


(A) Father
(B) Grandson
(C) Son
(D) Grandfather
(E) None of these
Ans : (A)

39. R – T’ ? How is R related to P in the expression ‘P + Q ×


(A) Niece
(B) Paternal Uncle
(C) Paternal Aunt
(D) Either (B) or (C)
(E) None of these
Ans : (C)

40. How is S related to P in the S’ ? expression ‘P – Q ÷ R ×


(A) Nephew
(B) Uncle
(C) Aunt
(D) Either (B) or (C)
(E) None of these
Ans : (D)

Directions—(Q. 41-45) In making decisions about important questions, it is desirable to be able


to distinguish between ‘strong’ arguments and ‘weak’ arguments. ‘Strong’ arguments must be
both important and directly related to the question. ‘Weak’ arguments may not be directly related
to the question and may be of minor importance or may be related to the trivial aspects of the
question. Each question below is followed by two arguments numbered I and II. You have to
decide which of the arguments is a ‘strong’ argument and which is a ‘weak’ argument. Give
answers—
(A) If only argument I is strong.
(B) If only argument II is strong.
(C) If either argument I or II is strong.
(D) If neither argument I nor II is strong.
(E) If both arguments I and II are strong.

41. Statement : Should the number of holidays given to Govt. employees be reduced to only five
in a year ?
Arguments :
I. Yes, such holidays subsequently reduce working hours, thus adversely affecting the economy
of nation.
II. No, employees require intermittent rest from hectic work schedule.
Ans : (B)

42. Statement : Should all correspondence courses at graduate level be stopped ?


Arguments :
I. No, correspondence courses help needy students to persue studies and earn at the same time.
II. Yes, quality education is not possible without teachers and classrooms.
Ans : (E)

43. Statement : Should only nuclear power be used to generate electricity ?


Arguments :
I. Yes, this will help reduce air pollution to a great extent.
II. No, Radioactive material used in nuclear plants is unsafe for large scale use.
Ans : (B)

44. Statement : Should the Govt. remove all the slums in major cities ?
Arguments :
I. Yes, slums are a nuisance to the people living in big cities.
II. No inhabitants of slums are also citizens of the country and they contribute towards the
growth of the nation.
Ans : (D)

45. Statement : Should cricket replace hockey as the national sport of India.
Arguments :
I. Yes, the performance of hockey team has been dismal since last few years.
II. No, cricket is the national sport of Australia and no two countries must have the same national
sport.
Ans : (B)

Directions—(Q. 46-50) Each of the questions below consists of a question and two statements
numbered I and II are given below it. You have to decide whether the data provided in the
statements are sufficient to answer the question. Read both the statements and give answers—

(A) If the data in Statement I alone are sufficient to answer the question, while the data in
Statement II alone are not sufficient to answer the question.
(B) If the data in Statement II alone are sufficient to answer the question, while the data in
Statement I alone are not sufficient to answer the question.
(C) If the data in Statement I alone or in Statement II alone are sufficient to answer the question.
(D) If the data in both the Statements I and II are not sufficient to answer the question.
(E) If the data in both the Statements I and II together are necessary to answer the question.

46. On which month of the year was Divya born ?


I. Her mother correctly remembers that Divya was born after June and before September.
II. Her father correctly remembers that she was born after March and before August.
Ans : (E)

47. What is the code for ‘those’ if in a certain language ‘those lovely red roses’ is written as ‘pe
so la ti’ ?
I. ‘ni jo ke pe’ means ‘stopped at red light’.
II. ‘ba di ti ga’ means ‘roses are very pretty and ‘fo hi la’ means ‘lovely day outside’.
Ans : (E)

48. What is Sumit’s rank from the top in a class of 50 students ?


I. Ajay is 6 ranks below Sumit and is twenty-ninth from the bottom.
II. Ashu is 4 ranks above Sumit and is thirty-fifth from the bottom.
Ans : (C)

49. How many sisters does Mala have ?


I. The only brother of Mala’s father has only one niece.
II. The son of the husband of Mala’s mother has two siblings.
Ans : (D)

50. Who among A, B, C and D is sitting next to A if all the four are sitting in a straight line
facing North ?
I. A does not sit next to D who does not sit next on the extreme right.
II. None sit to the left of A and on the right of B, while only one person sits between C and B.
Ans : (B)

Directions—(Q. 51-55) Study the following information carefully and answer the given
questions.

P, Q, R, S, T, W and Z are seven students studying in three different institutes – A, B and C.


There are three girls among the seven students who study in each of the three institutes. Two of
the seven students study BCA, two study medicine and one each studies Aviation Technology,
Journalism and MBA. R studies in the same college as P who studies MBA in college B. No girl
studies journalism or MBA. T studies BCA in college A and his brother W studies Aviation
Technology in college C. S studies journalism in the same college as Q. Neither R nor Z study
BCA. The girl who studies BCA does not study in college C.

51. Which of the following pairs of students study medicine ?


(A) QZ
(B) WZ
(C) PZ
(D) SZ
(E) None of these
Ans : (E)

52. In which college does Q study ?


(A) A
(B) B
(C) C
(D) Data inadequate
(E) None of these
Ans : (A)

53. In which of the colleges do three of them study ?


(A) A
(B) B
(C) A and B
(D) C
(E) None of these
Ans : (A)

54. What is the field of study of Z ?


(A) Aviation Technology
(B) BCA
(C) MBA
(D) Medicines
(E) None of these
Ans : (D)

55. Which of the following three represents girls ?


(A) SQR
(B) QRZ
(C) SQZ
(D) Data inadequate
(E) None of these
Ans : (B)

Directions—(Q. 56-65) Study the following information carefully and answer the given
questions.

For a recruitment process in an organization, the candidates need to possess the following
qualifications/criteria.
(i) A graduate in science with atleast 60% marks.
(ii) An age of atleast 25 years and not more than 40 years as on 1.7.2008.
(iii) Have a post-qualification work experience of atleast 2 years.
(iv) Should have secured 55% marks in the selection process.

However, if the candidates fulfills above mentioned criteria except—


(a) At (i) if the candidate is not a graduate in Science but has a post graduation degree with
minimum of 60% marks, he/she should be referred to the HRManager of the organization.
(b) At (iii) if the candidate fulfils all the eligibility criteria but of postqualification work
experience but has secured 75% marks in the selection process, he/she may be referred to the
Director of the organization.

Based on the above information study carefully whether following candidates are eligible for the
recruitment process and mark your answer as follows. You are not to assume anything other than
the information provided in each question. All the cases are given to you as on 1.7.2008.
Mark answers—

(A) If the candidate is to be selected.


(B) If the candidates is not to be selected.
(C) If the candidate may be referred to the HR-Manager.
(D) If the candidate may be referred to the Director.
(E) If the data provided is inadequate to take the decision.

56. Shruti Walia has 4 years of post qualification work experience in a top organization. She has
secured 59% marks in the selection process. Born on 5.10.1981, she had completed her
Bachelor’s degree in Physics in 2004 and secured 66% marks in it.
Ans : (A)

57. Pradeep Kumar has 6 years of post qualification work experience. His date of birth is
12.4.1972. He has secured 58% marks in the selection process. He has completed graduation in
Science and scored 76% in it.
Ans : (A)

58. Zaheer Ahmed has completed graduation and post graduate in Economics from Bhopal in
1999 and has secureds 51% and 68% marks respectively. His date of birth is 26.11.1976. He has
been working as an executive in a reputed firm since 2003 till date. He has secured 56% marks in
the selection process.
Ans : (C)

59. Harpreet Kaur has done graduation in Physics and is a post graduate in Science and has
secured 57% and 65% marks respectively. She has secured 59% marks in the selection process,
and has a post qualification work experience of 3 years.
Ans : (E)

60. Deepak Agarwal has completed graduation and post graduation in Geography with 68% and
has secured 57% marks in the selection process. His date of birth is 11.11.1979.
Ans : (E)

61. Varun Arora has secured 79% marks in the selection process. He has completed his degree in
B.Sc securing 67% marks in it. His date of birth is 9.3.1975, and has a post qualification work
experience of one year.
Ans : (D)
62. Varsha Nath’s date of birth is 6.2.1979. She has done her graduation and post-graduation in
Commerce and secured 59% and 62% marks respectively. She has a post qualification work
experience as a senior executive of 3 years and has secured 59% marks in the selection process.
Ans : (C)

63. Asha Walia has 4 years of post qualification work experience in a reputed firm. She has done
graduation in Botany scoring 71% marks in it. Born on 22.1.1973, she has secured 53% marks in
the selection process.
Ans : (B)

64. Nitesh Burman has done Grduation in Chemistry and has secured 69% in it. His date of birth
is 9.12.1981. He has ranked first in the selection process and has secured 82% marks in it. He has
four months of post qualification work experience.
Ans : (D)

65. Tanya Shetty has been working in a leading organization since the completion of post
graduation in 1990. She was born on 1.5.1968. She is a graduate and a post graduate in Science
and has secured more than 60% at both graduate as well as post graduate level. She has secured
71% marks in the selection process.
Ans : (B)

66. How many meaningful English words can be made with the letters ODME using each letter
only once in each word ?
(A) None
(B) One
(C) Two
(D) Three
(E) More than three
Ans : (D)

67. How many such digits are there in the number 84315269 each of which is as far away from
the beginning of the number as when the digits are rearranged in ascending order ?
(A) None
(B) One
(C) Two
(D) Three
(E) More than three
Ans : (D)

68. Four of the following five are alike in a certain way and so form a group. Which is the one
tthat does not belong to that group ?
(A) 19
(B) 35
(C) 15
(D) 21
(E) 27
Ans : (A)

69. Four of the following five are alike in a certain way and so form a group. Which is the one
that does not belong to that group ?
(A) Mars
(B) Jupiter
(C) Earth
(D) Uranus
(E) Moon
Ans : (E)

70. If only each of the consonants in the word GROUNDS is changed to the next letter in the
English alphabet, which of the following will be the third letter from the right end ?
(A) O
(B) U
(C) S
(D) V
(E) None of these
Ans : (A)

71. What should come next in the following letter series ?


AACACEACEGACEGIACEGI
(A) J
(B) K
(C) A
(D) L
(E) None of these
Ans : (B)

72. How many such pairs of letters are there in the word REFORM each of which has as many
letters between them in the word as in the English alphabet ?
(A) None
(B) One
(C) Two
(D) Three
(E) More than three
Ans : (E)

73. In a certain code BOARD is written as ‘53798’ and TRAIN is written as ‘29714’. How is
BAIT written in that code ?
(A) 5714
(B) 5412
(C) 5712
(D) 5912
(E) None of these
Ans : (C)
74. ‘BE’ is related to ‘FI’ and ‘LO’ is related to ‘PS’ in the same way as ‘RU’ is related to
……….
(A) VY
(B) WZ
(C) VZ
(D) VX
(E) None of these
Ans : (A)

75. Mohan walked 20 metres towards North, took a left turn and walked 10 metres, then he took
a right turn and walked 20 metres, again he took a right turn and walked 10 metres. How far is he
now from the starting point ?
(A) 20 metres
(B) 30 metres
(C) 40 metres
(D) Cannot be determined
(E) None of these
Ans : (C)

United Bank of India Probationary Officers Exam., 2009


(Held on 21-6-2009)
General Awareness : Solved Paper
1. Very often we see in the advertisements published by Financing Institutes/Agencies stating
that their products are given high or average Ratings. These Rating Agencies classify
bonds/investments in how many categories ?
1. Low Risk
2. Average Risk
3. High Risk
(A) Only 1
(B) Only 2
(C) Only 3
(D) All 1, 2 and 3
(E) None of these
Ans : (D)

2. As per the reports published in various newspapers/journals etc. the economic map of the
world is being redesigned. What is/are the new emerging trends of this new economic map of the
world ?
1. With India and Asia at the forefront the centre of economic gravity has shifted to East.
2. Emerging markets, with India among the leaders, are growing faster than old established
markets.
3. Central banks of the countries have recognized that case-by-case liquidity solutions are not the
answers and a market wide and globally synchronized approach is needed to solve the present
crisis.
(A) Only 1
(B) Only 2
(C) Only 3
(D) All 1, 2 and 3
(E) None of these
Ans : (A)

3. As per the decision taken by the Govt. of India, the exporters of which of the following
products to USA and European countries will get an incentive of 2% on their exports ?
1. Leather goods
2. Garments
3. Software
(A) Only 1
(B) Only 2
(C) Only 3
(D) Both 1 and 2 only
(E) All 1, 2 and 3
Ans : (D)

4. Which of the following is/are the key features of Indian Economy which were highlighted
during the presentation of the Interim Budget 2009-10 ?
1. Despite global financial crisis the GDP growth rate in current financial year has been 7•1 per
cent.
2. Indian economy was adjudged as second fastest growing economy in the world
3. A provision of Rs. 100 crores is made in the annual plan 2009-10 for setting up Unique
Identification Authority of India.
(A) Only 1
(B) Only 2
(C) Only 3
(D) All 1, 2 and 3
(E) None of these
Ans : (D)

5. Many a times we read a term in financial news papers ‘SEPA’. What is the full form of the
term ?
(A) Single Exchange Processing Agency
(B) Single Euro Payments Area
(C) Single Electronic Processing Agency
(D) Super Electronic Purchase Agency
(E) None of these
Ans : (B)

6. As per the newly launched ‘Indira Gandhi National Widow Pension Scheme’ a monthly
pension will be given to the widows. What will be the amount of the pension ?
(A) Rs. 200
(B) Rs. 400
(C) Rs. 600
(D) Rs. 800
(E) None of these
Ans : (A)

7. As per the decision taken by the Govt. of India all the Public Sector Banks (PSBs) will be
recapitalised over the next two years so that they can maintain a Capital Adequacy Ratio (CAR)
of ……….
(A) 9%
(B) 12%
(C) 11%
(D) 22%
(E) None of these
Ans : (B)

8. As per the Industrial Development Report 2009 India’s rank in Industrial Development was 54
on the Development Index. Which of the following organizations/agencies compiles Industrial
Development Index of the world ?
(A) International Labour Organisation
(B) United Nations’ Industrial Development Organisation
(C) World Bank
(D) World Trade Organisation
(E) The Economic and Social Council of UNO
Ans : (B)

9. As per the news published in major news papers/magazines the International Finance
Corporation has offered a loan for providing clean drinking water to 30 lakhs people in rural
India. What is the amount of the loan ?
(A) 5 million US $
(B) 10 million US $
(C) 15 million US $
(D) 20 million US $
(E) None of these
Ans : (C)

10. The merger of which of the following two Indian companies took place in recent past which
is termed as “Largest ever merger in India’s Corporate History” ?
(A) Nuclear Power Corporation and National Thermal Power Corporation
(B) State Bank of India and its seven associate banks
(C) Hind Aluminium Company and Century Mills
(D) Tata Motors and Ashok Leyland
(E) Reliance Industries and Reliance Petroleum Ltd.
Ans : (E)

11. As per the agreements signed by the Nuclear Power Corporation of India Ltd. (NPCL) and
National Thermal Power Corporation (NTPC) a joint Venture will be launched for setting up
various Nuclear Power Plants in India. If this is materialized all these establishments will be
required to function necessarily under the framework of which of the following existing acts ?
1. Industrial Disputes Act
2. Monopolies and Restrictive Trade Practices Act
3. Atomic Energy Act
(A) Only 1
(B) Only 2
(C) Only 3
(D) All 1, 2 and 3
(E) None of these
Ans : (C)

12. As we all know more and more countries/organizations are now going for Non-Cash
Transactions and accordingly Banks have launched many new products in the market for the
same. Which of the following products is a non-cash transaction product ?
(A) Only ATM Card
(B) Only Credit Card
(C) Only Prepaid Card
(D) Only Debit Card
(E) All are non-cash transaction products
Ans : (E)

13. The Prime Minister of India while addressing a meeting of top industrialists of India in
March 2009 advised the RBI to further cut interest rates. Which of the following is/are the
reasons owing to which the Prime Minister who himself is an economist gave this advise to
RBI ?
1. All major economies of the world are passing through a crisis situation. Prime Minister wants
RBI to take preventive measures to save Indian economy.
2. In India ample liquidity is available in the market but there are no takers.
3. Inflation is at a comparatively low level at present in the country
(A) Only 1
(B) Only 2
(C) Both 2 and 3 only
(D) Only 3
(E) None of these
Ans : (A)

14. As per the reports published in the newspapers India has signed an agreement to supply its
Dhruv Advanced Light Helicopter (ALH) to ……….
(A) Mauritius
(B) Bangladesh
(C) Nepal
(D) Zimbabwe
(E) Vietnam
Ans : (A)
15. Which of the following is/are TRUE about the present situation of Sugar Market in India
which has undergone a dramatic change in last 12 months ?
1. From being a major exporter of sugar, India is desperately looking for imports today to meet
its domestic needs.
2. Domestic prices have spurted by 60-70 per cent with no sign of respite.
3. The present situation is largely due to sharp contraction in cane acreage in the country.
(A) Only 1
(B) Only 2
(C) Only 3
(D) All 1, 2 and 3
(E) None of these
Ans : (D)

16. As per the recent reports, the per capita Income of the country has gone up during 2008-09.
What is the per capita income at present ?
About ……….
(A) Rs. 15,000
(B) Rs. 33,000
(C) Rs. 40,000
(D) Rs. 23,000
(E) Rs. 53,000
Ans : (C)

17. A major Financial Newspaper while writing about the present status of economy in India
wrote “the outlook in the agricultural sector gives room for optimism”. What does it really
mean ?
[Pick up most appropriate statement(s)]
1. Agricultural Sector which was not playing any significant role in Indian economy is now
growing very fast and significantly.
2. Agricultural sector is not going to play any major role in economy as its progress is still very
slow.
3. Govt. will not require to provide any boost up package to agricultural sector as it is likely to be
satisfactory this year.
(A) Only 1
(B) Only 2
(C) Only 3
(D) Either 1 or 3
(E) None of these
Ans : (C)

18. While addressing a conference of Bankers and top Industrialists, the Prime Minister made a
statement that “Domestic Credit Flow for Productive Needs had to be maintained at a
reasonable Cost”. What does it really mean ?
(Pick up the most appropriate statements).
1. Banks should further raise their deposit base and provide very low cost credit to Industry
which is badly in need of the same.
2. Banks should neither reduce their lending rates further nor increase the rates on deposits as
this may create a critical imbalance in the financial/money market of the country.
3. Banks should see that ample credit is provided to the industries at an affordable cost without
diluting the credit norms.
(A) Only 1
(B) Only 2
(C) Only 3
(D) Both 1 and 2 only
(E) None of these
Ans : (C)

19. A major financial newspaper recently published a report wherein it is said that “the Govt.
was aware of the problems in certain sectors, particularly where Export Dependence was
High.’ Keeping general trend of India’s present economic situation in mind how would you
interpret the above statement ?
[Pick up the most appropriate statement(s)].
1. The industry which produces mainly for export purposes is not doing well at present and needs
Govt. support.
2. The industry which imports raw material and exports processed goods is in very bad shape
these days. Govt. wishes to help them so that they can come out of it.
3. Indian exports were never in comfortable situations and this year they are in their worst
situation. Govt. knows it but cannot help.
(A) Only 1
(B) Only 2
(C) Only 3
(D) Both 1 and 3 only
(E) None of these
Ans : (A)

20. Who amongst the following is an Economist of international fame and was in news recently ?
(A) John Key
(B) Paul Krugman
(C) Nicolas Sarkozy
(D) Mohamed Elbaradei
(E) Mohamed Nasheed
Ans : (B)

21. Which of the following taxes is not levied by the Union Govt. ?
(A) Customs
(B) Corporate Tax
(C) Land Revenue
(D) Income Tax
(E) Surcharge on Income Tax
Ans : (C)
22. Service Tax was introduced in India for the first time in the year ……….
(A) 1990-91
(B) 1991-92
(C) 1994-95
(D) 1980-81
(E) 2000-01
Ans : (C)

23. Which of the following countries launched world’s first satellite for monitoring Greenhouse
Gases ?
(A) USA
(B) Britain
(C) Japan
(D) China
(E) India
Ans : (C)

24. Which of the following countries launched its indigenous Satellite ‘Omid’ in February
2009 ?
(A) Spain
(B) Australia
(C) Turkey
(D) China
(E) Iran
Ans : (E)

25. The Govt. of India allowed the Income Tax Department to set up its centralized IT
processing centre in ……….
(A) Bangalore
(B) Chennai
(C) Mumbai
(D) Kolkata
(E) Hyderabad
Ans : (A)

26. As per news reports MTNL recently launched it “3G Services” and became the first telecom
operator to launch the same in India. What is the full form of ‘G’ in 3G ?
(A) Global
(B) Generation
(C) Growth
(D) Gravity
(E) None of these
Ans : (B)

27. India recently signed an agreement with which of the following countries so that it can
import Uranium from it ?
(A) China
(B) Tunisia
(C) Brazil
(D) Taiwan
(E) None of these
Ans : (E)

28. The annual meeting of the world Economic Forum was organized in January–February 2009
in ……
(A) Davos
(B) Brasilia
(C) Washington DC
(D) London
(E) Tokyo
Ans : (A)

29. What is the minimum support price announced by the Govt. of India for Rabi crops of
wheat ?
(A) Rs. 900 quintal
(B) Rs. 950 quintal
(C) Rs. 1,000 quintal
(D) Rs. 1,080 quintal
(E) Rs. 1,200 quintal
Ans : (D)

30. Which of the following is/are the highlights of the ‘project Snow Leopard’ launched in recent
past ?
1. It is launched to safeguard and conserve India’s unique natural heritage and high altitude wild
life.
2. The project is launched to make another hitech permanent Lab on South pole.
3. It is nothing but a project to protect snow leopards found in Himalayan region which is facing
the fear of extinction.
(A) Only 1
(B) Only 2
(C) Only 3
(D) Both 1 and 3 only
(E) None of these
Ans : (A)

31. Which of the following has now became the Third Largest Economy of the World ?
(A) Brazil
(B) India
(C) China
(D) Japan
(E) Russia
Ans : (C)
32. Ms. Johanna Sigurdardottir has taken over as the first women Prime Minister of ……
(A) Brazil
(B) Iceland
(C) Peru
(D) New Zealand
(E) Tunisia
Ans : (B)

33. Govt. of India has decided to set up two more All India Institute of Medical Sciences
(AIIMS) like institutes in near future. These new institutions would be established in ……….
(A) West Bengal and Uttar Pradesh
(B) Haryana & Punjab
(C) Punjab & Tamil Nadu
(D) Tamil Nadu and Andhra Pradesh
(E) Gujarat and Maharashtra
Ans : (A)

34. Duleep Trophy is associated with the game of ……….


(A) Hockey
(B) Badminton
(C) Football
(D) Lawn Tennis
(E) Cricket
Ans : (E)

35. Who amongst the following is the winner of Saraswati Samman 2008 ?
(A) Gulzar
(B) Nida Fazli
(C) Srilal Shukla
(D) Govind Mishra
(E) Lakhmi Nandan Bora
Ans : (E)

36. Who amongst the following is the New Prime Minister of Israel after the general election
held there in recent past ?
(A) Lieberman
(B) Tzipi Livini
(C) Shimon Peres
(D) Benjamin Netanyahu
(E) None of these
Ans : (D)

37. Who amongst the following is the highest Wicket taker in the history of cricket ?
(Both ODI and Test Cricket)
(A) Wasim Akram
(B) Anil Kumble
(C) Stephan Fleming
(D) Shane Warne
(E) Muttiah Muralidharan
Ans : (E)

38. Which of the following commissions set up by the President of India decides the distribution
of tax incomes between the Central and State Govts. ?
(A) Central Law Commission
(B) Pay Commission for Govt. Employees
(C) Administrative Reforms Commission
(D) Planning Commission
(E) Finance Commission
Ans : (E)

39. Who amongst the following became the first Indian to win ‘Golden Globe Award 2009’ ?
(A) Shivkumar Sharma
(B) Anu Malik
(C) Zakir Hussain
(D) A. R. Rahman
(E) None of these
Ans : (D)

40. Sania Mirza won the Mixed Doubles of which of the following tennis matches alongwith
Mahesh Bhupati ?
(A) US Open 2009
(B) Australian Open 2009
(C) South Africa Tennis Open 2009
(D) ATP Challenger Tennis Championship 2009
(E) None of these
Ans : (B)

41. We very frequently read about the activities of the Foreign Exchange Market in
newspapers/magazines. Which of the following is/are the major functions of the same ?
1. Transfer of purchasing power from domestic to foreign market.
2. Providing credit for financing foreign trade.
3. Power to purchase gold from foreign countries as most of the nations still work on Gold
Standards.
(A) Only 1
(B) Only 2
(C) Only 3
(D) Both 1 and 2 only
(E) Both 2 and 3 only
Ans : (D)

42. Who amongst the following players won the National Shooting Championship held in
January 2009 ?
(A) Abhinav Bindra
(B) Omkar Singh
(C) Gurpreet Singh
(D) Anjali Bhagwat
(E) None of these
Ans : (E)

43. Who amontst the following is the author of the book “Termites in the Trading system” ?
(A) M. S. Swaminathan
(B) Jagdish Bhagwati
(C) Jaswant Singh
(D) M. Chelapati Rao
(E) None of these
Ans : (B)

44. The Govt. of India recently introduced Money Laundering Bill 2009. Under the bill which of
the following agencies is empowered to search the premises after the offence is committed and a
case is filed ?
(A) Income Tax Department
(B) Vigilance Department of the Reserve Bank of India
(C) Enforcement Directorate
(D) Central Bureau of Investigation
(E) None of these
Ans : (C)

45. Which of the following is/are the components of the Fiscal Deficit ?
1. Budgetary Deficit
2. Market Borrowings
3. Expenditure made from Pradhan Mantri Rahat Kosh
(A) Only 1
(B) Only 2
(C) Only 3
(D) All 1, 2 and 3
(E) None of these
Ans : (A)

46. Which of the following is the Trophy/Cups associated with the game of Hockey ?
(A) Derby
(B) Agha Khan Cup
(C) Merdeka
(D) Vizzy Trophy
(E) Rovers Cup
Ans : (B)

47. Which of the following is not a Welfare scheme launched by the Govt. of India ?
(A) Village Grain Bank Scheme
(B) Sampoorna Gramin Rozgar Yojana
(C) Annapurna Scheme
(D) Midday Meal Scheme
(E) Bharat Nirman Yojana
Ans : (E)

48. Which of the following are the instruments of Credit Control in the hands of the RBI ?
1. Lowering or raising the discount and interest rates.
2. Raising the minimum support price of the major agro products.
3. Lowering or raising the minimum cash reserves maintained by the commercial banks.
(A) Only 1
(B) Only 2
(C) Only 3
(D) Both 1 and 3 only
(E) Both 2 and 3 only
Ans : (C)

49. Who amongst the following is the author of the Book “Dreams from My Father : A Story of
Race and Inheritance” ?
(A) Barack Obama
(B) John Evans Aatta Mills
(C) Nelson Mandela
(D) Danny Boyle
(E) None of these
Ans : (A)

50. Which of the following programmes was launched to further improve the facilities of
irrigation in rural India ?
(A) National Social Assistance programme
(B) Construction of Kiosks for poors
(C) Sampoorna Grameen Rozgar Yojana
(D) Annapurna Scheme
(E) National Watershed Development Programme
Ans : (E)

Indian Overseas Bank Probationary Officers Exam., 2009


(Held on 5-4-2009)
General Awareness : Solved Paper
1. Which one of the following is the percentage upto which FDI in defense sector is allowed ?
(A) 26%
(B) 41%
(C) 31%
(D) 49%
(E) None of these
Ans : (A)

2. Whenever RBI does some Open Market Operation Transactions, actually it wishes to regulate
which of the following ?
(A) Inflation only
(B) Liquidity in economy
(C) Borrowing powers of the banks
(D) Flow of Foreign Direct Investments
(E) None of these
Ans : (C)

3. The maximum amount of the Total Revenue earned by the Govt. of India comes from—
(A) Income Tax
(B) Customs Duty
(C) Excise Duty
(D) Value Added Tax
(E) Corporate Tax
Ans : (E)

4. In economics it is generally believed that the main objective of a Public Sector Financial
Company like Bank is to—
(A) Employ more and more people
(B) Maximize total profits
(C) Maximize total production
(D) Provide financial service to the people of the nation of its origin across the country
(E) Sell the goods at subsidized cost
Ans : (D)

5. Which of the following is TRUE about the ‘Single—EURO Payment Area’ a concept very
much in news these days ?
1. It is a concept floated by European Union.
2. It aims to create an integrated market for payment services in EURO.
3. It is a contract to convert all US dollar payments into EURO, which is a currency almost free
from fluctuations in its value.
(A) Only 1
(B) Only 2
(C) Only 3
(D) Only 1 and 2
(E) None of these
Ans : (D)

6. What is a Repo Rate ?


(A) It is a rate of which RBI sell Govt. securities to banks
(B) It is a rate at which RBI buys Govt. securities from banks
(C) It is a rate at which RBI allows small loans in the market
(D) It is a rate which is offered by Banks to their most valued customers or prime customers
(E) None of these
Ans : (A)

7. In a Company the use of price sensitive corporate information by the Company-people to


make gains or cover losses is known as—
(A) Insider trading
(B) Future trading
(C) Foreign trading
(D) Stock trading
(E) None of these
Ans : (A)

8. As per the directives issued by the Prime Minister’s Economic Advisory Council (EAC), the
percentage of the current account deficit (of the total GDP) during 2008-2009 should not be
more than—
(A) 2•8%
(B) 3•0%
(C) 3•1%
(D) 3•2%
(E) None of these
Ans : (D)

9. The agricultural census is done at which of the following intervals ?


(A) There is no fixed term
(B) Every year
(C) Once in three years
(D) Once in ten years
(E) Once in five years
Ans : (E)

10. Which of the following cannot be called as a Debt Instrument as referred in financial
transactions ?
(A) Certificate of Deposits
(B) Bonds
(C) Stocks
(D) Commercial Papers
(E) Loans
Ans : (C)

11. The amount of which of the following reflects the overall budgetary position of the Govt. of
India at a given time ?
(A) Revenue Deficit
(B) Total Amount of Income Tax collected
(C) Capital Deficit
(D) Fiscal Deficit
(E) None of these
Ans : (D)

12. The 11th Five Year Plan is termed as plan for—


(A) India’s Health
(B) Eradication of poverty from India
(C) India’s Education
(D) Development of Rural India
(E) None of these
Ans : (C)

13. Who amongst the following gave the concept of PURA (Providing Urban facilities in Rural
Areas) ?
(A) Rajiv Gandhi
(B) Manmohan Singh
(C) A.P.J. Abdul Kalam
(D) Sonia Gandhi
(E) None of these
Ans : (C)

14. The higher rate of growth of economy certainly reduces the—


(A) Gender inequalities
(B) Poverty
(C) Population of a country
(D) Flow of foreign direct investment
(E) None of these
Ans : (B)

15. Bharat Nirman does not cover which of the following areas ?
(A) Rural employment
(B) Rural Housing
(C) Rural Water supply
(D) Irrigation facilities
(E) It covers all the above areas
Ans : (A)

16. Which of the following committees has given its recommendations on ‘Financial Inclusion’ ?
(A) Rakesh Mohan Committee
(B) Rangarajan Committee
(C) Sinha Committee
(D) Kelkar Committee
(E) None of these
Ans : (B)

17. Which of the following correctly describes what sub-prime lending is ?


1. Lending to the people who cannot return the loans.
2. Lending to the people who are high value customers of the banks.
3. Lending to those who are not a regular customer of a bank.
(A) Only 1
(B) Only 2
(C) Only 3
(D) All 1, 2 and 3
(E) None of these
Ans : (A)

18. Which of the following is/are the major concepts visible in today’s banking industry in
India ?
1. Risk Based Management
2. Growing competition
3. IT Initiatives
(A) Only 1
(B) Only 2
(C) Only 3
(D) Both 2 and 3
(E) All 1, 2 and 3
Ans : (A)

19. Which of the following services provided by a bank in India is not liable for Service Tax as
per existing laws ?
(A) Safe Deposit Lockers
(B) Merchant Banking Services
(C) Credit cards
(D) Discount earned on certain discounted bills
(E) None of these
Ans : (D)

20. The actual return of an investor is reduced sometimes as the prices of the commodities go up
all of a sudden. In financial sector this type of phenomenon is known as—
(A) Probability risk
(B) Market risk
(C) Inflation risk
(D) Credit risk
(E) None of these
Ans : (B)

21. In India Minimum Support Prices are announced by the Govt. of India every year. Which of
the following commodities is not covered under this policy ?
(A) Sugar
(B) Wheat
(C) Cotton
(D) White goods
(E) All are covered under the policy
Ans : (D)

22. Which of the following policies of a country brings a crucial impact on the economic
growth of the nation ?
1. Fiscal policy
2. Foreign policy
3. Social policies
(A) Only 1
(B) Only 2
(C) Only 1 and 2
(D) Only 2 and 3
(E) All 1, 2 and 3
Ans : (C)

23. Which of the following financial products is not much popular in India as they are launched
recently only ?
(A) Development Bonds
(B) Insurance Policies
(C) Mutual funds
(D) Sovereign Wealth Funds
(E) All are very popular products in India
Ans : (D)

24. Which of the following countries is given permanent membership of the World Trade
Organisation recently ?
(A) Ukrain
(B) India
(C) Pakistan
(D) Sri Lanka
(E) None of these
Ans : (E)

25. When the prices of commodities, goods and services start declining consistently, the
phenomenon is known as—
(A) Discount yield
(B) Deflation
(C) Negative growth
(D) Market capitalization
(E) None of these
Ans : (B)

26. Which of the following products launched by most of the banks help farmers in getting
instant credit for various agicultural purposes ?
(A) Kissan Credit Card
(B) Personal Loan
(C) Business Loan
(D) ATM Card
(E) None of these
Ans : (A)

27. Which of the following policies of the financial sectors is basically designed to transferring
local financial assets into foreign financial asset freely and at market determined exchange
rates ? Policy of—
(A) Capital Account convertibility
(B) Financial Deficit Management
(C) Minimum Support Price
(D) Restrictive Trade Practices
(E) None of these
Ans : (A)

28. A customer wishes to purchase some US dollars in India. He/ She should go to—
(A) Public Debt Division of the RBI only
(B) American Express Bank Only
(C) RBI or any branch of a bank which is authorized for such business
(D) Ministry of Foreign Affairs
(E) None of these
Ans : (C)

29. India’s Look East Policy was launched in the year—


(A) 1990
(B) 1992
(C) 1995
(D) 2000
(E) None of these
Ans : (B)

30. Which of the following products of a bank is specifically designed to provide financial help
to children in their higher studies in India or in a foreign nation ?
(A) Personal Loan
(B) Corporate Loan
(C) Educational Loan
(D) Mortgage Loan
(E) None of these
Ans : (C)

31. A Bank/Financial Organization these days relies heavily on e-commerce for its transaction.
As a part of system security, it has introduced organization’s security awareness manual. This
step of the organization can be classified under which one of the following categories of
measures for a business ?
(A) Preventive
(B) Compliance
(C) Corrective
(D) Detective
(E) None of these
Ans : (B)

32. Which of the following is the limitation of the ATMs owing to which people are required to
visit branches of the bank ?
1. It does not accept deposits.
2. It has a limited cash disbursement capacity.
3. Lack of human interface
(A) Only 1
(B) Only 2
(C) Only 3
(D) All 1, 2 and 3
(E) None of these are limitations
Ans : (B)

33. Which of the following is/are the objectives of developing Special Economic Zones (SEZs) ?
1. Generate additional economic activities.
2. Generate additional employment opportunities.
3. Promotion of export of goods and services.
(A) Only 1
(B) Only 2
(C) Only 3
(D) Only 1 and 2
(E) All 1, 2 and 3
Ans : (E)

34. An industry which is fighting hard to increase its market share in existing market (with new
popular products) is known as—
(A) Market vendor
(B) Market operator
(C) Market leader
(D) Market follower
(E) Market challenger
Ans : (D)

35. Maruti view Tata Motors as prominent competitor but does not view Volvo as a competitor.
This type of competition in the market is known as—
(A) Product competition
(B) Production flow competition
(C) Cost Control competition
(D) Service competition
(E) None of these
Ans : (C)

36. Which of the following is not a social assistance programme launched by the Govt. of India ?
(A) National Old Age Pension Scheme
(B) Annapurna Scheme
(C) National Family Benefit Scheme
(D) Indira Awaas Yojana
(E) All are social assistance programmes
Ans : (D)

37. What is the Statutory Liquidity Ratio (SLR) at present ?


(A) 14%
(B) 18%
(C) 20%
(D) 24%
(E) None of these
Ans : (D)

38. Mr. Raman Singh has taken over as the Chief Minister of which of the following States after
a General Election in November/December 2008 ?
(A) Jammu & Kashmir
(B) Madhya Pradesh
(C) Delhi
(D) Chhattisgarh
(E) None of these
Ans : (D)

39. The World Development Report- 2009 is released a few months earlier, by which of the
following Organisations ?
(A) UNESCO
(B) ADB
(C) IMF
(D) WTO
(E) World Bank
Ans : (E)

40. As per news published in newspapers, how much money Govt. of India has decided to invest
in Infrastructure Sector as an initiative ? About—
(A) Rs. 30,000 crores
(B) Rs. 40,000 crores
(C) Rs. 50,000 crores
(D) Rs. 60,000 crores
(E) Rs. 70,000 crores
Ans : (E)

41. Mr. Hosni Mubarak who is the recipient of the Jawaharlal Nehru Award for International
Understanding recently is the—
(A) President of South Africa
(B) Prime Minister of South Africa
(C) President of Egypt
(D) Prime Minister of Egypt
(E) Secretary General of UNO
Ans : (C)

42. Mr. Paul Krugman whose name was in news recently is a famous—
(A) Medical Scientist
(B) Economist
(C) Author
(D) Astrophysicist
(E) Sports Personality
Ans : (B)

43. The Buckingham Canal which is declared a National Waterway of India recently is a Canal
running from—
(A) Punjab to Haryana
(B) Mumbai to Goa
(C) Vijayawada Villupuram
(D) Cochin to Kozhikode
(E) Kanyakumari to Rameshwaram
Ans : (C)

44. Many times we see in newspapers that some projects are launched by the Govt. Authorities
on ‘PPP’ basis. What is the full form of ‘PPP’ ?
(A) Preferential Payment Plan
(B) Public Private Partnership
(C) Partial Payment Project
(D) Popular Private Project
(E) Public Private Plan
Ans : (B)

45. Which of the following countries is not the member of Asia Pacific Economic Cooperation
(APEC) which was in news recently ?
(A) South Korea
(B) Canada
(C) China
(D) India
(E) U.S.A.
Ans : (D)

46. The compulsory Education Act will ensure education to the children upto the age of—
(A) 10 years
(B) 12 years
(C) 8 years
(D) 6 years
(E) None of these
Ans : (E)

47. Saniya Mirza and Mahesh Bhupati won which of the following titles recently ?
(A) Australia Open
(B) U.S. Open
(C) Wimbledon Finals
(D) Davis Cup Finals
(E) None of these
Ans : (A)

48. Mr. Kunwar Narayan who was selected for the Jnanpith Award (for 2005) recently is a
famous poet and writer in—
(A) Urdu
(B) English
(C) Gujarathi
(D) Punjabi
(E) Hindi
Ans : (E)

49. Which of the following countries is planning to launch its Moon Mission Space Vehicle
named as Moon-LITE ?
(A) Pakistan
(B) Germany
(C) Russia
(D) France
(E) Britain
Ans : (E)

50. Which of the following books is written by Mr. Nandan Nilekani ?


(A) Imagining India
(B) A Good Woman
(C) Between the Assassinations
(D) The White Tiger
(E) None of three
Ans : (A)

Gurgaon Gramin Bank Officers Exam., 2009


(Held on 12-7-2009)
General Awareness : Solved Paper
1. Which of the following is a programme to provide electric connections to those living below
poverty line ?
(A) MPLADS
(B) Kutir Jyoti
(C) Ganga Kalyan Yojana
(D) Sangam Yojana
(E) None of these
Ans : (B)

2. As per the guidelines issued by the SEBI the Permanent Account Number (PAN) is a must for
which of the following ?
(A) Demat Accounts
(B) All Saving Banks Accounts
(C) All Housing Loan Accounts
(D) All Current Accounts
(E) None of these
Ans : (A)

3. For getting National Old Age Pension the age of the person should not be less than—
(A) 50 years
(B) 55 years
(C) 58 years
(D) 60 years
(E) 65 years
Ans : (E)

4. Many a times we read about the priority sector in newspapers. Which of the following is/are
considered segment(s) of Priority Sector in India ?
1. Agriculture
2. Small Scale Industry
3. Textile Industry
(A) Only 1
(B) Only 2
(C) Only 3
(D) All 1, 2 and 3
(E) None of these
Ans : (A)

5. The average size of holdings in India is about 2 hectares or 5 acres. What problems do farmers
normally face due to this ?
1. Proper irrigation facilities are not possible for these farms.
2. Scientific and advanced techniques are very expensive for these farms.
3. It does not pose a problem for them. These farms are ideal in size. Hence all sorts of advanced
methods can easily be used for them.
(A) Only 1
(B) Only 2 is correct
(C) Only 3 is correct
(D) Both 1 and 2 are correct
(E) None is correct
Ans : (B)
6. Which of the following is considered a technological factor in the productivity of agricultural
land ?
(A) Inadequate irrigation facilities
(B) Size of holding
(C) Pattern of Land Tenure
(D) Type of marketing facilities in the area
(E) All the above
Ans : (A)

7. Many times we read a term ‘Free Market’ in newspapers. What does it mean ?
[Pick up the correct statement(s)]
1. It is a market where pricing is driven by the laws of supply and demand.
2. In free market things are available without restraints of the government
3. In such markets goods are free from extraneous influences such as tariff or quotas.
(A) Only 1
(B) Only 2
(C) Only 3
(D) 1 and 2 both
(E) 1, 2 and 3 all
Ans : (E)

8. Earlier it was thought safe to allow each financial institution to carry out only one type of
financial service. But now banks are selling shares, insurance companies can sell banking
services.
These new financial conglomerates are colloquially referred to as—
(A) Multinationals
(B) Financial supermarkets
(C) Market regulators
(D) Brand Promoters
(E) None of these
Ans : (B)

9. Marketing of agro products and also of the cottage industry is most important for economic
development. Which of the following should be the major consideration for the marketing of
rural products ?
(A) Proper storing facility only
(B) Proper holding capacity only
(C) Adequate and cheap transport facility only
(D) Small number of intermediaries only
(E) All the above are needed
Ans : (E)

10. Many people travel to different parts of India to work as agricultural labour as the wages
given to them are different in different parts. Which of the following is/are determining factor(s)
in deciding the structure of agricultural wages in India ?
1. Land to Labour ratio i.e. The number of people employed on a land holding of a particular
size.
2. Nature of crop sown on the land. Food crops do not attract good wages whereas commercial
crops do.
3. Nature of the area as a high productivity area will have higher wages and vice-versa.
(A) Both 1 and 2
(B) Both 1 and 3
(C) Only 2
(D) All 1, 2 and 3
(E) None of these
Ans : (D)

11. The structure of the financial system in India can be divided into many segments. Which of
the following is not one of these segments ?
(A) Industrial Finance
(B) Agriculture Finance
(C) Developmental Finance
(D) Government Finance
(E) Personal Finance
Ans : (E)

12. Which of the following statements about the changes in cropping patterns is/are correct ?
1. The Govt. of India can influence crop pattern through legislative and administrative measures.
2. The Cropping pattern normally depends upon the climatic conditions of the area.
3. As 80% of the agro products are being exported from India the cropping pattern depends upon
the WTO stipulations in the field.
(A) Only 1
(B) Only 2
(C) Both 1 and 2 are correct
(D) Only 2 and 3 are correct
(E) All 1, 2 and 3 are correct
Ans : (E)

13. The Head of the Hindu Undivided Family (HUF) is called—


(A) Swami
(B) Karta
(C) Bharta
(D) Agraj
(E) Jyeshthi
Ans : (B)

14. The Banking Industry and financial sector in India has come a long way in the past few
years. Which of the following were the major drawbacks of the banks in India, inherited from the
British Raj owing to which the Govt. of India decided to nationalize them in 1969 and put them
through a series of reforms again ?
1. Interlocking of directorship of big business houses.
2. Paucity of credit to socially and economically important sectors of the economy.
3. Shortage of trained manpower to handle the major task of economic development.
(A) Only 1
(B) Only 2
(C) Only 3
(D) Both 1 and 2
(E) All 1, 2 and 3
Ans : (D)

15. A national policy to provide rural credit was launched by the Govt. of India in general and by
RBI and NABARD in particular. Which of the following cannot be considered as one of the
objectives of this National Policy ?
1. To ensure timely and increased flow of credit to the rural sector.
2. To make available credit facilities to every region of the country so that there are no regional
imbalances.
3. To establish a well organized network of informal/non institutional credit system in the
country.
(A) Only 1
(B) Both 1 and 2
(C) Only 3
(D) Both 1 and 3
(E) All 1, 2 and 3
Ans : (C)

16. Under Bharat Nirman Programme of the Govt. of India action is proposed in various areas.
Which of the following is not one of these areas ?
(A) Irrigation
(B) Rural Roads
(C) Rural Housing
(D) Rural Water Supply
(E) Rural Employment
Ans : (E)

17. Which of the following is the name of the legislation, which provides employment to the
poors in remote areas of the country ?
(A) NREGA
(B) POTA
(C) FEMA
(D) COFEPOSA
(E) None of these
Ans : (A)

18. The Reserve Bank of India issues coins and notes of various denominations. At present RBI
does not issue coins of which of the following denominations ?
(A) 10 paise
(B) 25 paise
(C) 50 paise
(D) 1•00 Rupee
(E) 5•00 Rupee
Ans : (A)

19. Many a times we come across a term ‘Know Your Customer’ in area of banking transactions.
Which of the following is the main purpose of issuing Know Your Customer guidelines by the
RBI ?
(A) To provide better customer service
(B) To keep a check on money laundering
(C) To bring more and more people in the Income Tax net
(D) To make high value transactions faster
(E) None of these
Ans : (B)

20. Which of the following is not correct about the various targets set for 11th Five Year Plan ?
(Whole plan period 2007-12).
(A) Farm sector growth to be increased to 8%
(B) GDP growth rate to be increased to 10%
(C) Create 7 Crore new jobs
(D) Literacy rate is to be increased to 80%
(E) Reduction in unemployment amongst the educated youth
Ans : (A)

21. Many a times we come across a term read as ‘GM’ in reference to certain crops. What is the
full form of the term ‘GM’ ?
(A) Geographically Moderated
(B) Genetically Modified
(C) Globally Marketed
(D) Grown after Modifications
(E) None of these
Ans : (B)

22. Many a times we see a term in newspapers ‘IPO’. What is the full form of the same ?
(A) Indian Public Offer
(B) Institutional Purchase Offer
(C) Industrial Purchase Order
(D) Indian Purchase Offer
(E) Initial Public Offer
Ans : (E)

23. Which of the following is not the part of the Charter of the National Rural Health Mission
(NRHM) ?
1. NRHM has to pay special attention to 18 States, which have weak Public Health Indicators.
2. Mission provides for appointment of Accredited Social Health Activist (ASHA) in each one of
four metros so that these metros become free from Polio, Malaria, and HIV infections by 2010 ?
3. It aims at effective integration of Private Sector in health sector so that government burden can
be reduced to the level of mere policy maker.
(A) Only 1
(B) Only 2
(C) Both 2 and 3
(D) Only 3
(E) Both 1 and 3
Ans : (C)

24. Who amongst the following had said ‘small aim is crime’ ?
(A) Jawaharlal Nehru
(B) Lal Bahadur Shastri
(C) Smt. Indira Gandhi
(D) Dr. A. P. J. Kalam
(E) Dr. Manmohan Singh
Ans : (D)

25. The rate on which banks lend to RBI is known as—


(A) CRR
(B) PLR
(C) Bank Rate
(D) REPO Rate
(E) SLR
Ans : (A)

26. Which of the following name is not associated with the insurance business in India ?
(A) Bajaj Allianz
(B) LIC
(C) GIC
(D) Tata AIG
(E) GE Money
Ans : (E)

27. Which of the following awards is given for excellence in the field of sports ?
(A) Kalidas Samman
(B) Dhyanchand Award
(C) Shram Vir Award
(D) Shanti Swarup Bhatnagar Award
(E) None of these
Ans : (B)

28. Which of the following does not represent the name of a bank working in India ?
(A) YES
(B) HDFC
(C) TATA
(D) Kotak Mahindra
(E) Axis
Ans : (C)

29. Who amongst the following addresses the nation on the Independence Day from the ramparts
of the Red Fort, Delhi ?
(A) President of India
(B) UPA Chair person
(C) Speaker of the Lok Sabha
(D) Prime Minister of India
(E) None of these
Ans : (D)

30. Which of the following is considered an informal method of getting credit/finance ?


(A) Internet Banking
(B) Branch visits
(C) Going to money lenders
(D) Tele Banking
(E) All of these
Ans : (C)

31. Which of the countries represent letter ‘C’ in the category known as BRIC countries ?
(A) Canada
(B) Chile
(C) Cuba
(D) Croatia
(E) China
Ans : (E)

32. Nobel Prizes are not given for the performance in the area of—
(A) Literature
(B) Physics
(C) Chemistry
(D) Music
(E) Medical Science
Ans : (D)

33. Which of the following countries has not made any significant investment in India ?
(A) Japan
(B) USA
(C) Nepal
(D) Britain
(E) France
Ans : (C)

34. Twenty-20 matches are played in the game of—


(A) Hockey
(B) Football
(C) Badminton
(D) Tennis
(E) Cricket
Ans : (E)

35. Which of the following is known as Plastic money ?


1. Demand Draft
2. Credit Card
3. Debit Card
(A) Only 1
(B) Only 2
(C) Only 3
(D) Both 2 and 3
(E) All 1, 2 and 3
Ans : (D)

36. In which of the following Hindi films actor Shahrukh Khan has played the role of a Hockey
Coach ?
(A) Guru
(B) Chak De India
(C) Om Shanti Om
(D) Jab We Met
(E) None of these
Ans : (B)

37. Which amongst the following nations is not a member of the SAARC ?
(A) Bangladesh
(B) Bhutan
(C) Nepal
(D) Maldives
(E) South Africa
Ans : (E)

38. Which of the following organizations/bank has done a commendable work in the field of
micro finance and was awarded Nobel Prize also in the past ?
(A) Gramin Bank of Bangladesh
(B) CRY
(C) ASHA
(D) NABARD
(E) None of these
Ans : (A)

39. Which of the following is not a Public Sector Unit/Undertaking/ Agency ?


(A) ECGC
(B) SEBI
(C) SIDBI
(D) Axis Bank
(E) BHEL
Ans : (D)

40. The European Union has adopted which of the following as a common currency ?
(A) Dollar
(B) Dinar
(C) Yen
(D) Peso
(E) Euro
Ans : (E)

41. Various Govt. agencies/organizations are given responsibilities for implementing various
policies/decisions of the Govt. of India. Policies about credit flow to the agricultural/priority
sector are framed/implemented through which of the following apex bodies ?
(A) ECGC
(B) NABARD
(C) UTI
(D) IDBI Bank
(E) None of these
Ans : (B)

42. Which of the following is not a foodgrain ?


(A) Wheat
(B) Rice
(C) Maize
(D) Jowar
(E) Cotton
Ans : (E)

43. Many a times we read in newspapers about ‘Financial Inclusion’. What does it really mean ?
[Pick up correct statement(s)]
1. Allow the merger and acquisition of banks so that only few big banks exist and continue to
cater to the need of corporate sector.
2. Expending the network of banks in such a way that people from lower strata of society also
get the benefit of services provided by banks.
3. Providing Insurance cover to each and every citizen so that he/she can live a healthy and long
life.
(A) Only 1
(B) Only 2
(C) Only 3
(D) Both 1 and 2
(E) All 1, 2 and 3
Ans : (B)

44. Which of the following is not a gallantry Award ?


(A) Vir Chakra
(B) Jeevan Raksha Padak
(C) Ashok Chakra
(D) Mahavir Chakra
(E) Saraswati Samman
Ans : (E)

45. Heavy Water is normally used by which of the following industries ?


(A) Nuclear Power generation plants
(B) Pharma Industry
(C) Paper Industry
(D) Sugar plants
(E) None of these
Ans : (A)

46. As per news items published in various newspapers, Italy has offered its help in developing
‘SMEs’ in India. What is the full form of ‘SME’ ?
(A) Small and Monopolistic Economy
(B) Small and Medium Enterprises
(C) Speedy and Mechanical
(D) Small and Medium Level Economy
(E) None of these
Ans : (B)

47. Which of the following indexes is developed to measure life expectancy, level of literacy,
education and standard of living of people in a country ?
(A) Inflation
(B) Sensex
(C) Human Development Index
(D) SLR
(E) None of these
Ans : (C)

48. Which of the following is a type of tax levied by the Govt. on goods and services ?
(A) SAT
(B) NET
(C) PAN
(D) VAT
(E) None of these
Ans : (D)

49. Who amongst the following was made the Chairman of the National Land Reforms Council ?
(A) Smt. Sonia Gandhi
(B) Arjun Singh
(C) Pranav Mukherjee
(D) Kamal Nath
(E) Manmohan Singh
Ans : (E)

50. Which of the following is not a foreign bank working in India ?


(A) HSBC
(B) Barclays
(C) Standard Chartered
(D) Yes Bank
(E) All are foreign banks
Ans : (E)

Indian Bank (Clerk) Exam., 2009


(Held on 21-6-2009)
English Language : Solved Paper
Directions—(Q. 1–15) Read the following passage carefully and answer the questions given
below it. Certain words/phrases have been printed in bold to help you locate them while
answering some of the questions.

Once upon a time, there was a rich merchant who had four wives. He loved the fourth wife the
most and adorned her with rich robes and treated her to delicacies. He took great care of her and
gave her nothing but the best. He also loved the third wife very much. He was very proud of her
and always wanted to show her off to his friends. However, the merchant was always in great
fear that she might run away with some other men. He loved his second wife too. She was a very
considerate person, always patient and in fact was the merchant’s confidant. Whenever the
merchant faced some problems, he always turned to his second wife and she would always help
him out and tide him through difficult times. Now, the merchant’s first wife was a very loyal
partner and had made great contributions in maintaining his wealth and business as well taking
care of the household. However, the merchant did not love the first wife and although she loved
him deeply, he hardly took notice of her.

One day, the merchant fell ill. Before long, he knew that he was going to die soon. He thought of
his luxurious life and told himself, “Now I have four wives with me. But when I die, I’ll be
alone. How lonely I’ll be !” Thus, he asked the fourth wife, “I loved you most, endowed you
with the finest clothing and showered great care over you. Now that I’m dying will you follow
me and keep me company ?” “No way !” replied the fourth wife and she walked away without
another word. The answer cut like a sharp knife right into the merchant’s heart. The sad
merchant then asked the third wife, “I have loved you so much for all my life. Now that I’m
dying will you follow me and keep me company ?” “No !” replied the third wife, “Life is so
good over here ! I’m going to remarry when you die !” The merchant’s heart sank and turned
cold. He then asked the second wife. “I always turned to you for help and you’ve always helped
me out. Now I need your help again. When I die, will you follow me and keep me company ?”
“I’m sorry, I can’t help you out this time !” replied the second wife. “At the very most, I can only
send you to your grave.” The answer came like a bolt of thunder and the merchant was
devastated. Then a voice called out, “I’ll leave with you. I’ll follow you no matter where you
go.” The merchant looked up and there was his first wife. She was so skinny, almost like she
suffered from malnutition. Greatly grieved, the merchant said, “I should have taken much better
care of you while I could have !”

Actually, we all have four wives in our lives. The fourth wife is our body. No matter how much
time and effort we lavish in making it look good, it’ll leave us when we die. Our third wife is our
possessions, status and wealth. When we die, they all go to others. The second wife is our family
and friends. No matter how close they had been to us when we’re alive, the furthest they can
stay by us is up to the grave. The first wife is in fact our soul, often neglected in our pursuit of
material wealth and sensual pleasure. It is actually the only thing that follows us wherever we go.
Perhaps it’s a good idea to cultivate and strengthen it now rather than to wait until we’re on our
deathbed to lament.

1. What did the merchant want to do with his third wife ?


(A) To buy good clothes for her
(B) To give her away to his friends
(C) To show her off to his friends
(D) To share his problems with her
(E) None of these

2. What was the second wife’s quality ?


(A) She helped the merchant during tough times
(B) She helped the merchant in his business
(C) She made the merchant feel proud
(D) She contributed to the merchant’s wealth
(E) None of these

3. What does the phrase ‘cut like a sharp knife’ as used in the passage mean ?
(A) Injure
(B) Bleed
(C) Dangerous
(D) Hurt
(E) None of these

4. Which of the following is possibly NOT a characteristic of the first wife as given in the
passage ?
(A) Caring
(B) Beautiful
(C) Thin
(D) Loyal
(E) All of the above are characteristics of the first wife

5. What fear did the merchant have about his third wife ?
(A) That she would kill him
(B) That she would not accompany him when he died
(C) That she would run away with all his money
(D) That she would not look beautiful anymore
(E) None of these

6. What explanation did the fourth wife give for not accompanying the merchant when he was
dying ?
(A) She wanted more money
(B) She did not give any explanation
(C) She wanted to marry someone else after the merchant’s death
(D) She was afraid to go with him
(E) None of these

7. Why did the first wife offer to go with the merchant when he died ?
(A) Because she loved him
(B) Because she was afraid to stay alone
(C) Because she hated the other wives
(D) Because she was undernourished and was going to die anyway
(E) None of these

8. The author has compared the merchant’s fourth wife to our ………
(A) Soul
(B) Wealth
(C) Friends
(D) Status
(E) None of these

9. What does the author have to say about our soul ?


(A) That it is well taken care of
(B) That it can give us company only till our death
(C) That it has been ignored in the chase for worldly pleasures
(D) That it belongs to someone else after our death
(E) None of these

10. What does the phrase ‘like a bolt of thunder’ used in the passage mean ?
(A) like a very bright light
(B) like a pleasant feeling
(C) like a loud noise
(D) like a shock
(E) None of these

Directions—(Q. 11–13) Choose the word which is MOST SIMILAR IN MEANING to the
word printed in bold as used in the passage.

11. adorned
(A) spent
(B) dressed
(C) tried
(D) carried
(E) necklace

12. showered
(A) bestowed
(B) cleaned
(C) pleased
(D) bathed
(E) threw

13. lament
(A) fear
(B) tell
(C) grieve
(D) care
(E) forget

Directions—(Q. 14–15) Choose the word which is MOST OPPOSITE IN MEANING to the
word printed in bold as used in the passage.

14. considerate
(A) ignorant
(B) lazy
(C) angry
(D) thoughtful
(E) insensitive

15. furthest
(A) longest
(B) closest
(C) greatest
(D) shortest
(E) wildest

Directions—(Q. 16–20) Which of the phrases (A), (B), (C) and (D) given below each sentence
should replace the phrase printed in bold in the sentence to make it grammatically correct ? If the
sentence is correct as it is given and ‘No correction is required’, mark (E) as the answer.

16. I woke up early in the morning and had a steamer cup of coffee.
(A) has a steamer
(B) has a steaming
(C) had a steaming
(D) had a steam
(E) No correction required
17. Tired of being harassed by the goons, I finally called the police and complained—
(A) in being harassed
(B) of being harass
(C) in be harassed
(D) of be harass
(E) No correction required

18. A friendship founded on business is best than a business founded on friendship.


(A) is good than
(B) is better than
(C) is bestest than
(D) is better then
(E) No correction required

19. Right action cannot came out of nothing, it must be preceded by thought.
(A) come out of
(B) came out off
(C) come off
(D) coming out of
(E) No correction required

20. One of the base laws of nature is that adaptability is the price of survival.
(A) basis laws of
(B) basic law of
(C) base law of
(D) basic laws of
(E) No correction required

Directions—(Q. 21–25) In each question below, a sentence with four words printed in bold type
is given. These are lettered as (A), (B), (C) and (D). One of these four words printed in bold may
be either wrongly spelt or inappropriate in the context of the sentence. Find out the word
which is wrongly spelt or inappropriate, if any. The letter of that word is your answer. If all the
words printed in bold are correctly spelt and also appropriate in the context of the sentence, mark
(E) i.e., ‘All correct’ as your answer.

21. Oportunities (A) multiply (B) when they are seized (C) and die (D) when they are not. All
correct (E)

22. He realized (A) he was alone (B) in the house and rushed (C) to bolt (D) all the doors and
windows. All correct (E)

23. The information (A) provided (B) to the staff was not adecuate (C) and everyone
retaliated. (D) All correct (E)

24. Worried (A) that he will fail in the exams, Satish stayed (B) up the whole kinght (C) and
studied. (D) All correct (E)

25. Thomas could not cook (A) very well and thus had to stay (B) food from (C) a restaurant.
(D) All correct (E)

Directions—(Q. 26–30) Rearrange the following six sentences (1), (2), (3), (4), (5) and (6) in the
proper sequence to form a meaningful paragraph; then answer the questions given below them.
(1) She eased out something from her waist folds.
(2) Maganlal welcomed a customer early in the day.
(3) Once visible, Maganlal realized it was a pair of gold bangles.
(4) The woman carefully counted the money and then left.
(5) He took the bangles and placed some money in the woman’s palm.
(6) This customer was a peasant woman wearing a discoloured sari and old anklets.

26. Which of the following should be the FIRST sentence after rearrangement ?
(A) 1
(B) 2
(C) 3
(D) 4
(E) 5

27. Which of the following should be the SECOND sentence after rearrangement ?
(A) 2
(B) 3
(C) 4
(D) 5
(E) 6

28. Which of the following should be the THIRD sentence after rearrangement ?
(A) 1
(B) 2
(C) 3
(D) 4
(E) 5

29. Which of the following should be the FOURTH sentence after rearrangement ?
(A) 2
(B) 3
(C) 4
(D) 5
(E) 6

30. Which of the following should be the LAST (SIXTH) sentence after rearrangement ?
(A) 2
(B) 3
(C) 4
(D) 5
(E) 6

Directions—(Q. 31– 40) Read each sentence to find out whether there is any grammatical error
or idiomatic error in it. The error, if any, will be in one part of the sentence. The letter of that
part is the answer. If there is no error, the answer is (E). (Ignore errors of punctuation, if any.)

31. Progress is impossible (A) / without change, and those (B) / who cannot change their minds
(C) / cannot change nothing. (D) No error (E)

32. She was allergic (A) / to some medicines (B) / and informed (C) / the doctor about it. (D) No
error (E)

33. He walked as faster (A) / as he could so that (B) / he would not (C) / miss the train to work.
(D) No error (E)

34. She shared (A) / all her secrets with (B) / Suman as they had been (C) / friends with
childhood. (D) No error (E)

35. Gautam did not care (A) / so many about (B) / anything else as much (C) / as he cared for his
dog. (D) No error (E)

36. You cannot change (A) / people, but you (B) / can definitely (C) / change own. (D) No error
(E)

37. Veena wanted to (A) / become a surgeon (B) / and worked very hardly (C) / to achieve this.
(D) No error (E)

38. Krishna ran to the (A) / nearing grocery store to (B) / buy biscuits as his parents (C) / were
expecting guests. (D) No error (E)

39. As soon so (A) / he came home, (B) / he showered and got (C) / ready to go out again. (D)
No error (E)

40. She failed to remember (A) / that it was Rema’s birthday (B) / and did not buy (C) / a present
for her. (D) No error (E)

Directions—(Q. 41–50) In the following passage there are blanks, each of which has been
numbered. These numbers are printed below the passage and against each, five words are
suggested, one of which fits the blank appropriately. Find out the appropriate word in each case.

One of the good things that happened to me …(41)… in Patliputra, …(42)… the friendship of
the farmer’s daughter. This nine year old girl …(43)… became very fond of me and her parents
…(44)… her that she could keep me forever as her toy. She was very good at needlecraft and …
(45)… clothes for her doll. In fact, on my first night there and …(46)… the rest of my stay in the
farmer’s house, I slept in her doll’s cradle. That first night, they put the cradle on …(47)… of a
shelf far away from the danger of rats. As I slowly got to learn their language, I was …(48)… to
talk to the girl and let her know my needs and she was able to make me …(49)… comfortable.
She made me seven shirts and was my teacher of the language. When I pointed to anything, she
would …(50)… it by name, and soon I was able to talk easily with her.

41. (A) waiting


(B) truly
(C) till
(D) still
(E) while

42. (A) for


(B) was
(C) because
(D) is
(E) it

43. (A) soon


(B) had
(C) was
(D) has
(E) forever

44. (A) handled


(B) worried
(C) taught
(D) promised
(E) carried

45. (A) tore


(B) tearing
(C) making
(D) wore
(E) make

46. (A) to
(B) as
(C) for
(D) of
(E) if

47. (A) bottom


(B) top
(C) coating
(D) height
(E) wide
48. (A) possible
(B) happiness
(C) eagerly
(D) able
(E) request

49. (A) every


(B) thorough
(C) total
(D) high
(E) more

50. (A) call


(B) ask
(C) throw
(D) help
(E) hit

Answers :
1. (C) 2. (A) 3. (D) 4. (B) 5. (E) 6. (E) 7. (A) 8. (E) 9. (C) 10. (D)
11. (B) 12. (A) 13. (C) 14. (E) 15. (B) 16. (C) 17. (E) 18. (B) 19. (A) 20. (D)
21. (E) 22. (E) 23. (C) 24. (C) 25. (B) 26. (B) 27. (E) 28. (A) 29. (B) 30. (C)
31. (D) 32. (E) 33. (A) 34. (D) 35. (B) 36. (D) 37. (C) 38. (B) 39. (A) 40. (E)
41. (E) 42. (B) 43. (A) 44. (D) 45. (C) 46. (A) 47. (B) 48. (D) 49. (E) 50. (A)

Andhra Bank (Clerk) Exam., 2009


(Held on 14-6-2009)
English Language : Solved Paper
Directions—(Q. 1–15) Read the following passage carefully and answer the questions given
below it. Certain words have been printed in bold to help you locate them while answering some
of the questions.

The great sage once had a group of disciples. They were all very bright and eager students and
the sage had all the reasons to be proud of them. One day the sage realized that he had imparted
enough knowledge to his disciples. Now they were all very learned. There was only one thing the
sage had not taught them, and that was the special verse that could bring the dead back to life.
The sage knew that such knowledge was too wonderful and could prove to be a dangerous thing
in the hands of someone who was not very wise. The sage pondered over this for a long time.
But he also knew that if he did not pass on this secret verse, it could die with him. So, at last he
called his cleverest disciple aside and said “I am going to teach you a very special verse. If you
chant this you can bring to life a dead person or animal. Use only when you need it to and never
misuse or test your powers vainly.” He then called all the disciples together and said, “I am
sending you all into the forest for forty days. Go together and come back together. Each one of
you has to guide one another and do good things.”

So the disciples started out together into the forest. They were all united. But the clever disciple
who knew the verse wanted to show he was better than the others. As they walked into the forest,
they came across a dead tiger on the way. It was huge and looked wickedly fierce even when
dead. The clever disciple stopped and said to the others. “Now I am going to show you what our
teacher has taught me alone. He has taught me how to bring life back into the dead.” The others
would not believe him and he said, “I would prove it to you by bringing this tiger back to life.
“But other disciple said “do not do anything to prove your knowledge vainly. Moreover, if you
put life into this tiger, it will only turn on us and kill us all. This will not be a wise thing to do.”

But the clever disciple had decided to prove himself and prepared to recite the verse. But before
he did so, the other disciples scrambled up to the topmost branches of a big tree nearby. The
disciple then recited the magical verse. The tiger slowly began to breathe. “Its working” cried
the disciple in excitement and joy. The tiger opened its eyes and saw him jumping and shouting
in front of him. Roaring loudly, the tiger pounced on the poor disciple and killed him.

The other disciples on the tree watched helplessly as the tiger threw down the dead body of the
disciple and went away into the forest. After some time the disciples came down, took the body
and went to the sage. The sage looked at them and said, “Now you see what can happen if you
don’t use your learning wisely. Let this be a lesson for you.” With that, the sage uttered the
magic verse and brought the dead disciple back to life. The sage then taught the verse to all his
disciples and sent them into the world to do good. He was sure that after such a lesson, they
would be wiser and use their knowledge and learning only for doing good.

1. Why did the other disciples climb the highest branches of the nearby tree ?
(A) To save their lives from the tiger
(B) They disliked the clever disciple and were hiding away from him
(C) To enjoy watching the tiger kill the disciple from a safe distance
(D) Because the sage had not taught the other disciples the secret verse
(E) None of these

2. Why did the sage send his disciples to the forest ?


(A) He wanted to get rid of them
(B) He had imparted all the knowledge he had and was left with nothing to teach
(C) He wanted them to be killed by a tiger so that he could test his secret verse
(D) So that the disciples could practice his secret verse on the wild animals
(E) None of these

3. Which of the following morals can be drawn from the above passage ?
(A) A teacher must pass on all his knowledge to others before he dies
(B) Advice of true friends should always be taken
(C) Teachers should always be impartial and should impart knowledge to all uniformly
(D) The knowledge and learning gained should never the misused
(E) None of these

4. Why did the sage decide to pass on his secret verse to his disciples ?
(A) So that the cleverest of the disciples may teach other disciples as well
(B) He did not want the verse to die with him
(C) So that the tiger may eat the cleverest disciple
(D) So that he could prove his genius to his disciples
(E) None of these

5. What did the sage say to his disciple while teaching his secret verse ?
(A) Never to misuse the power achieved with the help of the verse
(B) Not to use it on the wild animals
(C) Not to share it with other disciples
(D) To practice it on animals before humans
(E) None of these

6. How did the other disciples react when the clever disciple decided to bring the tiger back to
life ?
(A) They felt jealous of him
(B) They all felt that it was a wise thing to do
(C) They felt sorry for the tiger
(D) They tried to stop him as he could put their lives in danger
(E) None of these

7. Why was the sage proud of his disciples ?


(A) Because they had agreed to go to the forest for a long period of 40 days
(B) Because they had fought against the tiger very bravely
(C) Because they could bring the dead tiger back to life
(D) Because they were very bright and eager students
(E) None of these

8. Who/what is described as wicked and fierce by the author ?


(A) The clever disciple
(B) The sage
(C) The tiger
(D) The forest
(E) None of these

9. Which of the following is NOT TRUE in the context of the passage ?


(A) The clever disciple was brought back to life by the sage
(B) The sage wanted his disciples to use their learning only for doing good
(C) The sage was biased towards the clever disciple and disliked others
(D) The sage ultimately taught the secret verse to all his disciples
(E) All are true

10. Why did the clever disciple recite the verse to the dead tiger ?
(A) So that the other disciples may be eaten up by the tiger
(B) To flaunt the power of the verse to the other disciples
(C) Because the sage told him to do so
(D) So that he could himself to the sage
(E) None of these

Directions—(Q. 11–13) Choose the word which is MOST SIMILAR in MEANING to the
word printed in bold as used in the passage.

11. pondered
(A) guessed
(B) puzzled
(C) studied
(D) thought
(E) attended

12. pounced
(A) climbed
(B) looked
(C) roared
(D) plunged
(E) jumped

13. cried
(A) wept
(B) screamed
(C) protested
(D) tearful
(E) saddened

Directions—(Q. 14 –15) Choose the word which is MOST OPPOSITE in MEANING to the
word printed in bold as used in the passage.

14. bright
(A) dim
(B) soft
(C) dull
(D) faint
(E) vague

15. fierce
(A) timid
(B) emotional
(C) civilized
(D) pleased
(E) domesticated
Directions—(Q. 16 –20) Which of the phrases (A), (B), (C) and (D) given below each sentences
should replace the phrase printed in bold in the sentence to make it grammatically correct ? If the
sentence is correct as it is given and No Correction is Required, mark (E) as the answer.

16. He wants a start a new business but he did not have any money.
(A) Thought to start
(B) Want to started
(C) Wanted to start
(D) Wants for start
(E) No Correction Required

17. As Rohan was already late, he had his breakfast while walk down the road.
(A) walking down the
(B) walking at the
(C) walked on the
(D) walked down the
(E) No Correction Required

18. The poor bullock do not move since the cart was overloaded with heavy goods.
(A) can not move
(B) unable to move
(C) was not moved
(D) could not move
(E) No Correction Required

19. He covered the expensive furniture at sheet before he started painting the ceiling.
(A) with sheet ahead
(B) with sheet before
(C) in sheet prior
(D) in sheet earlier
(E) No Correction Required

20. The smoke if inhaled proves to be more fatal than the burn itself.
(A) be most fatal
(B) kill more fatal
(C) be additional fatal
(D) be greater fatal
(E) No Correction Required

Directions—(Q. 21–25) In each question below a sentences with four words printed in bold type
is given. These are lettered as (A), (B), (C) and (D). One of these four printed in bold may be
either misspelt or inappropriate in the context of the sentence. Find out the word which is
wrongly spelt or inappropriate if any. The letter of that word is your answer. If all the words
printed in bold are correctly spelt and also appropriate in the context of the sentence, mark (E)
i.e., ‘All correct’ as your answer.
21. Indian peoples (A) invest (B) as much (C) in gold as in bank savings accounts. (D) All
correct (E)

22. The issues (A) of hunger and poorty (B) are left behind (C) as we have progressed (D) in
technology. All correct (E)

23. Rajan would (A) pick (B) up his children from (C) the school and superwise (D) their
homework. All correct (E)

24. A good (A) leader is of prime (B) importance for develop (C) of any organisation. (D) All
correct (E)

25. The milk vendor (A) studied hard (B) for four years before topped (C) the national level
(D) exam. All correct (E)

Directions—(Q. 26–30) Rearrange the following six sentences 1, 2, 3, 4, 5 and 6 in the proper
sequence to form a meaningful paragraph; then answer the questions given below them—
1. The emperor was impressed with me and rewarded me suitably.
2. He then asked me to make it shorter without erasing its ends.
3. One fine day the king decided to test my intelligence.
4. By doing so, I could make the line shorter without erasing the ends.
5. After thinking over it for some time, I drew longer lines on both the ends of the line that the
emperor had drawn.
6. He drew a line on the floor with the help of a chalk

26. Which of the following should be the FIRST sentence after rearrangement ?
(A) 1
(B) 2
(C) 3
(D) 4
(E) 5

27. Which of the following should be the SECOND sentence after rearrangement ?
(A) 2
(B) 3
(C) 4
(D) 5
(E) 6

28. Which of the following should be the THIRD sentence after rearrangement ?
(A) 1
(B) 2
(C) 3
(D) 4
(E) 5
29. Which of the following should be the FOURTH sentence after rearrangement ?
(A) 2
(B) 3
(C) 4
(D) 5
(E) 6

30. Which of the following should be the LAST (SIXTH) sentence after rearrangement ?
(A) 1
(B) 2
(C) 3
(D) 4
(E) 5

Directions—(Q. 31– 40) Read each sentence to find out whether there is any grammatical error
or idiomatic error in it. The error, if any, will be in one part of the sentence. The letter of that
part is the answer. If there is no error, the answer is (E). (Ignore errors of punctuation, if any.)

31. She considered herself (A) / very fortunated to (B) / have had a (C) / very good education.
(D) No error (E)

32. A lot number of people (A) / donate money to the organisation (B) / at this time (C) / of the
year. (D) No error (E)

33. The leader of the opposition (A) / is in the danger (B) / of lost his seat (C) / in the next
elections. (D) No error (E)

34. He is the same (A) / ice-cream vendor (B) / which sold ice-creams (C) / when we were kids.
(D) No error (E)

35. The police received (A) / tremendous support from (B) / the general public (C) / over that
issue. (D) No error (E)

36. I have bought for my sister (A) / a bottle of perfume (B) / on her birthday (C) / and I hope she
liked it. (D) No error (E)

37. Throughout his tenure (A) / at the office he (B) / have proven himself to be a (C) / talented
and hard-working employee. (D) No error (E)

38. The committee did not (A) / approve the plan (B) / since it were (C) / neither practical nor
efficient. (D) No error (E)

39. The bus driver drove rashly (A) / and we had (B) / a narrow escape (C) / on a number of
occasions. (D) No error (E)
40. He never remember (A) / to pick up the (B) / milk bottles while (C) / picking up the
newspaper. (D) No error (E)

Directions—(Q. 41–50) In the following passage there are blanks, each of which has been
numbered. These numbers are printed below the passage and against each, five words are
suggested, one of which fits the blank appropriately. Find out the appropriate word in each case.

The visit to the planet earth museum is always exciting and teaches some interesting …(41)…
about the world we live in.

Forests are home to nearly half of the world’s plants and animals. Trees clean the air and …(42)
… oxygen for us to breathe. Everyday we cut …(43)… trees and destroy more than 36 football
fields of forests.

Rivers collect rainwater and …(44)… it to the oceans. Farms and factories pollute the rivers with
pesticides and chemicals. This is harmful to the marine animals and the humans …(45)… drink
it.

At the opposite ends of the worlds, Arctic and the Antarctic are freezing cold. So cold that they
are …(46)… covered in ice. The fuel we use makes the climate …(47)… It melts the ice and the
water level rises which …(48)… to floods.

Out planet is …(49)…. We must conserve it by using as much renewable sources of energy as
possible. Every individual must play his part to …(50)… to the protection of the environment.

41. (A) concept


(B) observation
(C) facts
(D) experience
(E) struggles

42. (A) take


(B) acquire
(C) display
(D) filter
(E) produce

43. (A) down


(B) lots
(C) much
(D) through
(E) thousands

44. (A) throw


(B) dispenses
(C) merges
(D) carry
(E) combines

45. (A) which


(B) who
(C) whom
(D) that
(E) those

46. (A) never


(B) once
(C) most
(D) equally
(E) always

47. (A) colder


(B) uncomfortable
(C) heated
(D) warmer
(E) burning

48. (A) leads


(B) results
(C) leading
(D) grows
(E) develop

49. (A) indestructible


(B) intense
(C) precious
(D) expensive
(E) lavish

50. (A) help


(B) contribute
(C) afford
(D) result
(E) leading

Answers :
1. (A) 2. (E) 3. (D) 4. (B) 5. (A) 6. (D) 7. (D) 8. (C) 9. (C) 10. (B)
11. (D) 12. (E) 13. (B) 14. (C) 15. (A) 16. (C) 17. (A) 18. (D) 19. (B) 20. (E)
21. (A) 22. (B) 23. (D) 24. (C) 25. (E) 26. (C) 27. (E) 28. (B) 29. (D) 30. (A)
31. (B) 32. (A) 33. (C) 34. (C) 35. (E) 36. (D) 37. (C) 38. (C) 39. (E) 40. (A)
41. (C) 42. (E) 43. (A) 44. (D) 45. (B) 46. (E) 47. (D) 48. (B) 49. (C) 50. (A)
Canara Bank Probationary Officers Exam., 2009
(Held on 15-3-2009)
English : Solved Paper
Directions—(Q. 1–15) Read the following passage carefully and answer the questions given
below it. Certain words/phrases are printed in bold to help you to locate them while answering
some of the questions.

John Maynard Keynes, the trendiest dead economist of this apocalyptic moment, was the
godfather of government stimulus. Keynes had the radical idea that throwing money at
recessions through aggressive deficit spending would resuscitate flatlined economies- and he
wasn’t too particular about where the money was thrown. In the depths of the Depression, he
suggested that the Treasury could “fill old bottles with banknotes, bury them at suitable depths in
disused coal mines” then sit back and watch a money-mining boom create jobs and prosperity.
“It would, indeed, be more sensible to build houses and the like, “he wrote, but “the above would
be better than nothing.”

As President-elect Barack Obama prepares to throw money at the current downturn-a stimulus
package starting at about $800 billion, plus the second $350 billion chunk of the financial
bailout-we all really do seem to be Keynesians now. Just about every expert agrees that pumping
$1 trillion into a moribund economy will rev up the ethereal goods-and-services engine that
Keynes called “aggregate demand” and stimulate at least some shortterm activity, even if it is all
wasted on money pits. But Keynes was also right that there would be more sensible ways to
spend it. There would also be less sensible ways to spend it. A trillion dollars’ worth of bad
ideas-sprawl-inducing highways and bridges to nowhere, ethanol plants and pipelines that
accelerate global warming, tax breaks for overleveraged McMansion builders and burdensome
new long-term federal entitlements-would be worse than mere waste. It would be smarter to buy
every American an iPod, a set of Ginsu knives and 600 Subway foot-longs.

It would be smarter still to throw all that money at things we need to do anyway, which is the
goal of Obama’s upcoming American Recovery and Reinvestment Plan. It will include a mix of
tax cuts, aid to beleaguered state and local governments; and spending to address needs ranging
from food stamps to computerized health records to bridge repairs to broadband network to
energy-efficiency retrofits, all designed to save or create 3 million to 4 million jobs by the end of
2010. Obama has said speed is his top priority because the faster Washington injects cash into
the financial bloodstream, the better it stands to help avert a multiyear slump with double-digit
unemployment and deflation. But he also wants to use the stimulus to advance his long-term
priorities : reducing energy use and carbon emissions, cutting middle-class taxes, upgrading
neglected infrastructure, reining in health-care costs and eventually reducing the budget deficits
that exploded under George W. Bush. Obama’s goal is to exploit this crisis in the best sense of
the word to start pursuing his vision of a greener, father, more competitive, more sustainable
economy.

Unfortunately, while 21st century Washington has demonstrated an impressive ability to spend
money quickly, it has yet to prove that it can spend money wisely. And the chum of a 1 with 12
zeros is already creating a feeding frenzy for the ages. Lobbyists for shoe companies, zoos,
catfish farmers, mall owners, airlines, public broadcasters, car dealers and everyone else who can
afford their retainers are lining up for a piece of the stimulus. States that embarked on raucous
spending and tax cuttng sprees when they were flush are begging for bailouts now that they’re
broke. And politicians are dusting off their unfunded mobster museums, waterslides and other
pet projects for rebranding as shovel-ready infrastructure investments. As Obama’s aides
scramble to assemble something effective and transformative as well as politically achievable,
they acknowledge the tension between his desires for speed and reform.

1. Obama’s upcoming American Recovery and Reinvestment Plan focuses on which of the
following ?
(1) Recovery of all debts from the debtors in a phased manner.
(2) Pumping money very liberally in projects that are mandatory.
(3) Investing money recklessly in any project regardless of its utility.
(A) (1) only
(B) (2) only
(C) (3) only
(D) (2) and (3) only
(E) All (1), (2) & (3)
Ans : (D)

2. John M. Keynes was advocate of which of the following suggestions ?


(A) Spending money recklessly during recessions is suicidal
(B) Exorbitant spending during recessions is likely to boost economy
(C) Aggressive deficit spending is likely to be fatal for economic meltdown
(D) Government stimulus to economy may not help because of red-tapism
(E) None of these
Ans : (B)

3. Obama desires to accelerate the process of pumping money with utmost rapidity as he believes
that it would—
(1) Help create reasonably high employment opportunities
(2) Avoid deflation
(3) Inject cash into the already troubled economy
(A) (1) and (2) only
(B) (2) and (3) only
(C) (1) and (3) only
(D) All (1) (2) and (3)
(E) None of these
Ans : (A)

4. Which of the following is TRUE about Keynes’ philosophy ?


(A) Actual spending money during meltdown is more important than where and on what it is
spent
(B) Government should be selective in approach for spending money during recession
(C) Filling old bottles with banknotes and burying them is an atrocious proposal
(D) Creating jobs and prosparity during recessions is almost an impracticable proposal
(E) None of these
Ans : (B)

5. What, according to Keynes, is the “aggregate demand” ?


(A) Goods and Services Sector
(B) Stimulation of a short-term activity
(C) Attempting to rev up the sluggish economy
(D) Pumping one trillion dollars into economy
(E) None of these
Ans : (A)

6. According to the author of the passage, food stamps, bridge repairs, etc. are the projects that—
(A) Do not warrant urgent spending as they have a lower utility value
(B) Need the least investment and priority as compared to building houses for the needy
(C) May not have any favourable impact on attempts to counter recession
(D) Have lower value in terms of returns but require major investments
(E) None of these
Ans : (E)

7. The author of the passage calls Barack Obama and his team as “Keynesians” because—
(A) Barack Obama has been reluctant to follow Keynes’ philosophy
(B) His team is advising Barack to refrain from Keynes’ philosophy
(C) Barack Obama and his team have decided to fil old bottles with banknotes
(D) Building houses has been under the active consideration of Barack Obama and his team
(E) None of these
Ans : (C)

8. Highways, bridges, ethanol plants, etc. are considered by the author as—
(A) Reasonably appropriate propositions to spend money on
(B) Measures that affect the environment adversely
(C) Imprudent proposals to waste money on
(D) Tax saving schemes bestowed on builders
(E) None of these
Ans : (B)

9. Which of the following is/are corrective measure(s) as part of the long term priorities of
Obama that was an outcome of his predecessor’s regime ?
(1) Countering recession through immediate rescue operations.
(2) Reining the budget deficit.
(3) Creating a more sustainable economy.
(A) (1) & (2) only
(B) (2) & (3) only
(C) (1) & (3) only
(D) (2) only
(E) None of these
Ans : (E)
Directions—(Q. 10–12) Choose the word which is most OPPOSITE in meaning of the word
printed in bold as used in the passage.

10. Raucous
(A) Strident
(B) Harsh
(C) Rough
(D) Unprecedented
(E) Soft
Ans : (E)

11. Beleaguered
(A) Carefree
(B) Harassed
(C) Stressful
(D) Uneventful
(E) Evaporating
Ans : (A)

12. Moribund
(A) Declining
(B) Waning
(C) Thriving
(D) Pessimistic
(E) Glorifying
Ans : (C)

Directions—(Q. 13–15) Choose the word which is most nearly the SAME in meaning as the
word given in bold as used in the passage.

13. Frenzy
(A) Passion
(B) Expression
(C) Succession
(D) Habit
(E) Manifestation
Ans : (A)

14. Apocalyptic
(A) Unwelcome
(B) Disastrous
(C) Risk-free
(D) Joyous
(E) Ceremonious
Ans : (B)
15. Resuscitate
(A) Melt down
(B) Devastate
(C) Mislead
(D) Save
(E) Deactivate
Ans : (E)

Directions—(Q. 16–20) In each of these questions four words are given denoted by (1), (2), (3)
and (4). Two of these words may be either synonyms or antonyms. Find out the correct pair in
each question.

16. (1) Tranquility


(2) Loyalty
(3) Calamity
(4) Uproar
(A) 1–3
(B) 2–4
(C) 2–3
(D) 3–4
(E) 1–4
Ans : (E)

17. (1) Exorbitant


(2) Expeditious
(3) Quick
(4) Quest
(A) 3–4
(B) 1–2
(C) 1–4
(D) 3–2
(E) None of these
Ans : (C)

18. (1) Opaque


(2) Translucent
(3) Transverse
(4) Transvestite
(A) 1–4
(B) 2–4
(C) 3–1
(D) 2–1
(E) None of these
Ans : (D)
19. (1) Ecstasy
(2) Depression
(3) Intoxication
(4) Compression
(A) 1–2
(B) 2–4
(C) 2–3
(D) 3–4
(E) None of these
Ans : (A)

20. (1) Vilification


(2) Nullification
(3) Denigration
(4) Falsification
(A) 1–2
(B) 2–3
(C) 1–3
(D) 2–4
(E) None of these
Ans : (B)

Directions—(Q. 21–30) Which of the phrases (A), (B), (C) and (D) given below should replace
the phrase given in bold in the following sentence to make the sentence grammatically correct. If
the sentence is correct as it is and ‘No correction is required’, mark (E) as the answer.

21. Radha’s three children, Shantana, Manu and Meera are talented, but the latter excels the
other two.
(A) the last excels
(B) latter excel
(C) the latter excelling
(D) the last excelling
(E) No correction required
Ans : (A)

22. Students are not abandoning helmets, but some avoiding use of helmets while riding
motorbikes.
(A) some avoid the used
(B) some avoid of the use
(C) some are avoiding of use
(D) some are avoiding use
(E) No correction required
Ans : (D)

23. We must treat any statement as a rumour until they are confirmed with proof.
(A) till they are confirmed
(B) until they are confirming
(C) until it is confirmed
(D) until it is confirming
(E) No correction required
Ans : (C)

24. The officer appreciated his subordinate’s many attempt to bravely confront the miscreants.
(A) many attempting brave confronts
(B) many brave attempts to confront
(C) repeated attempts to brave confront
(D) many attempts of brave confront
(E) No correction required
Ans : (B)

25. We admire him attempting to climb the summit in such a bad weather.
(A) his attempting to climb
(B) his attempt of climb
(C) him for attempt of climb
(D) his for attempt to climbing
(E) No correction required
Ans : (E)

26. Being a successful businessman demands hard work, honesty, persuasive skills and sound
market knoweldge.
(A) To be a successful business who demands
(B) Being a successfully demanding businessman
(C) To be a successful businessman demanding
(D) For being a successful demanding businessman
(E) No correction required
Ans : (E)

27. Was it they who were accused of stealing the neighbour’s car ?
(A) Were it they who were
(B) Was it they who had
(C) Were they who
(D) Were it they who
(E) No correction required
Ans : (A)

28. The doctor has advised him to lay in bed at least for two weeks.
(A) that he lay in bed
(B) that he lays in bed
(C) to lie in bed
(D) to be laid in bed
(E) No correction reequired
Ans : (C)
29. He is the only one of the members who have paid all the dues.
(A) Member who has paid
(B) Members who have been paying
(C) Member who has been paid
(D) members who has paid
(E) No correction required
Ans : (D)

30. Neither any of the members of the society nor the Chairman were present for the annual
meeting.
(A) were present at
(B) was present for
(C) have been present
(D) has been present for
(E) No correction required
Ans : (B)

Directions—(Q. 31–35) In each sentence below one word has been printed in bold. Below the
sentence, five words are suggested, one of which can replace the word printed in bold, without
changing the meaning of the sentence. Find out the appropriate word in each case.

31. She came in utter disrepute due to her vindictive act.


(A) revengeful
(B) ungrateful
(C) unpardonable
(D) uneventful
(E) derogatory
Ans : (A)

32. His attempts to equivocate the subject under discussion were thwarted.
(A) balance
(B) defend
(C) mislead
(D) adjust
(E) reconcile
Ans : (C)

33. He was conceptually clear about the problem and therefore could provide a pragmatic
solution.
(A) diplomatic
(B) practical
(C) analytical
(D) tolerable
(E) acceptable
Ans : (B)
34. They returned after their ignominious defeat in the foreign country.
(A) irreversible
(B) impertinent
(C) indecisive
(D) impulse
(E) disgraceful
Ans : (E)

35. The minister’s clandestine meeting with the opposition party leader was severely criticized.
(A) unofficial
(B) authentic
(C) periodical
(D) secret
(E) purposeful
Ans : (D)

Directions—(Q. 36–40) Rearrange the following six sentences (1), (2), (3) (4), (5) and (6) in the
proper sequence to form a meaningful paragraph, then answer the questions given below them.
(1) In all varieties of humour, especially the subtle ones it is therefore what the reader thinks
which gives extra meaning to these verses.
(2) But such a verse may also be enjoyed at the surface level.
(3) Nonsense verse is one of the most sophisticated forms of literature.
(4) This fulfils the author’s main intention in such a verse which is to give pleasure.
(5) However the reader who understands the broad implications of the content and allusion finds
greater pleasure
(6) The reason being it requires the reader to supply a meaning beyond the surface meaning.

36. Which of the following is the THIRD sentence ?


(A) (1)
(B) (2)
(C) (6)
(D) (3)
(E) (4)
Ans : (A)

37. Which of the following is the SECOND sentence ?


(A) (1)
(B) (5)
(C) (6)
(D) (2)
(E) (3)
Ans : (C)

38. Which of the following is the FIFTH sentence ?


(A) (4)
(B) (5)
(C) (2)
(D) (3)
(E) (1)
Ans : (A)

39. Which of the following is the FIRST sentence ?


(A) (5)
(B) (1)
(C) (6)
(D) (4)
(E) (3)
Ans : (E)

40. Which of the following is the SIXTH (LAST) sentence ?


(A) (6)
(B) (5)
(C) (4)
(D) (1)
(E) (3)
Ans : (B)

Directions—(Q. 41–50) In the following passage, there are blanks, each of which has been
numbered. These numbers are printed below the passage and against each five words are
suggested, one of which fits the blank appropriately. Find out the appropriate word in each case.

The U.S. is in the …41… of a cleanup of toxic financial waste that will …42… taxpayers
hundreds of billions of dollars, at the very least. The primary manufacturers of these hazardous
products …43… multimillion- dollar paychecks for their efforts. So why shouldn’t they …44…
to pay for their mopup ? This is, after all, what the U.S. Congress …45… in 1980 for …46… of
actual toxic waste. Under the Superfund law …47… that year, polluters …48… for the messes
they make. Environmental lawyer E. Michael Thomas sees no …49… lawmakers couldn’t
demand the same of financial polluters and …50… them to ante up some of the bank bailout
money.

41. (A) essence


(B) debate
(C) range
(D) depth
(E) midst
Ans : (E)

42. (A) facilitate


(B) save
(C) benefit
(D) cost
(E) earn
Ans : (D)

43. (A) spent


(B) pocketed
(C) donated
(D) demanded
(E) dwindled
Ans : (A)

44. (A) have


(B) admit
(C) hesitate
(D) come
(E) defy
Ans : (C)

45. (A) admonished


(B) visualized
(C) decreed
(D) refrained
(E) commented
Ans : (B)

46. (A) producers


(B) users
(C) consumers
(D) advocates
(E) exponents
Ans : (A)

47. (A) abandoned


(B) enacted
(C) revoked
(D) forced
(E) squshed
Ans : (B)

48. (A) demand


(B) consider
(C) regain
(D) claim
(E) pay
Ans : (E)

49. (A) issue


(B) wonder
(C) practice
(D) reason
(E) compensation
Ans : (D)

50. (A) dupe


(B) follow
(C) force
(D) plead
(E) appeal
Ans : (C)

You might also like